Addiction Psychiatric Medicine
Addiction Psychiatric Medicine
Addiction Psychiatric Medicine
Addiction Psychiatric
Medicine
A COMPREHENSIVE BOARD REVIEW
LEILA M. VAEZAZIZI, MD
Clinical Instructor and Addiction Psychiatrist,
Montefiore Medical Center/Albert Einstein College of Medicine, New York, New York
Faculty, New York University School of Medicine, New York, New York
CONTENTS
v
Elsevier
1600 John F. Kennedy Blvd.
Ste 1800
Philadelphia, PA 19103-2899
No part of this publication may be reproduced or transmitted in any form or by any means, electronic or
mechanical, including photocopying, recording, or any information storage and retrieval system, without
permission in writing from the publisher. Details on how to seek permission, further information about the
Publisher’s permissions policies and our arrangements with organizations such as the Copyright Clearance
Center and the Copyright Licensing Agency, can be found at our website: www.elsevier.com/permissions.
This book and the individual contributions contained in it are protected under copyright by the Publisher (other
than as may be noted herein).
Notice
Practitioners and researchers must always rely on their own experience and knowledge in evaluating and
using any information, methods, compounds or experiments described herein. Because of rapid advances in
the medical sciences, in particular, independent verification of diagnoses and drug dosages should be made.
To the fullest extent of the law, no responsibility is assumed by Elsevier, authors, editors or contributors for
any injury and/or damage to persons or property as a matter of products liability, negligence or otherwise, or
from any use or operation of any methods, products, instructions, or ideas contained in the material herein.
Printed in India.
1
2 ADDICTION PSYCHIATRIC MEDICINE: A COMPREHENSIVE BOARD REVIEW
■ In a controlled environment: When a person who tolerance described earlier. Conditioned tol-
previously met diagnostic criteria for SUD is absti- erance refers to the presence of conditioned
nent as a result of no longer having access to the compensatory responses in response to using
substance when in an environment where access a substance in a novel environment, rather
to the substance is restricted (such as correctional than their familiar place of use. Conditioned
detention or residential rehabilitation programs). tolerance can be seen for example when a per-
son with opioid use disorder experiences an
Physiological dependence refers to the state result-
opioid overdose when they use heroin in a new
ing from repeated use of a substance marked by toler-
environment.
ance and/or withdrawal symptoms:
3. Incentive salience refers to yet another phenom-
■ Tolerance to a given substance is a phenom- enon. With incentive salience, specific environ-
enon marked by requiring a higher dose of mental sensory or experiential cues become as-
the substance to achieve the same intoxicating sociated with using a given substance (such as
effect (alternatively, it can be conceptualized as smells, places of use, or persons with whom one
experiencing a reduced effect if the individual uses). This leads to a physical state of expecta-
consumed their usual dose). tion of substance use when encountering these
■ Withdrawal is a syndrome occurring following dis- cues. For example, someone using cocaine at a
continuation or reduction of substance use marked nightclub will experience a strong urge to use co-
by symptoms opposite to the expected effects of caine again every time they go to the nightclub.
that substance. Resuming substance use would 4. Incentive salience is driven by operant condi-
reverse the withdrawal and relieve these symptoms. tioning (conditioning using positive or nega-
tive reinforcement) or classical conditioning
The 4 C’s is another commonly used working descr
(Pavlovian conditioning).
iption of the components of SUD.
Working Definitions of Addiction:
■ Compulsion SUD SEVERITY AND PATIENT
■ Loss of Control PLACEMENT CRITERIA
■ Consequences
■ Craving The American Society of Addiction Medicine (ASAM)
patient placement criteria (PPC-2R) offer a standard-
Know This: ized approach to connect a person’s SUD severity and
characteristics with the treatment level they require.
1. Rebound symptoms and pseudo-withdrawal Using the PPC-2R, persons with SUD are assessed for
are two phenomena that are distinct from sub- their treatment needs in six dimensions (each scored
stance withdrawal. Rebound symptoms refer 0–4 based on the associated complication risks):
to situations in which symptoms preceding
the drug use affected by the use worsen after ■ Dimension 1: Intoxication and withdrawal pote-
discontinuation (e.g., persons with insomnia ntial
using benzodiazepines to sleep might experi- ■ Dimension 2: Biomedical conditions and com-
ence worsening insomnia after benzodiaz- plications
epines discontinuation). Pseudo-withdrawal ■ Dimension 3: Emotional, behavioral, or cogni-
refers to placebo-type symptoms experienced tive complications
when a person using a substance considers or ■ Dimension 4: Readiness to change (trans-theo-
expects the substance’s discontinuation. These retical model of change or stages of change)
symptoms resemble symptoms that would like- ■ Dimension 5: Relapse or continued use potential
ly occur with drug discontinuation. ■ Dimension 6: Recovery environment (including
2. Conditioned (or learned) tolerance refers to a social, legal, vocational, educational, financial,
different phenomenon than the physiological and housing factors)
1 General Principles of Substance Use Disorders 3
Treatment type needs for every dimension are 2. Do not confuse placement matching and mo-
determined and classified by level: dality matching. “Placement matching” refers
to the required intensity of treatment resources
■ Level 0?
as identified in the PPC-2R, whereas “modal-
■ Level 0.5: Early intervention
ity matching” refers to whichever clinical ap-
■ Level I: Outpatient treatment
proach might be optimal in treating a patient’s
■ Level II.1: Intensive outpatient
problems (such as using contingency manage-
■ Level II.5: Partial hospitalization
ment for stimulant use disorders, buprenor-
■ Level III.1: Clinically managed low-intensity res-
phine for opioid use disorder, or dialectical
idential services
behavioral therapy [DBT] for borderline per-
■ Level III.3: Clinically managed medium-inten-
sonality disorder).
sity residential treatment
Child and adolescent levels of care utilization ser-
■ Level III.5: Clinically managed high-intensity
vices (CALOCUS) is a model similar to PPC-2R that
residential treatment
is specific for identifying necessary levels of care for
■ Level III.7: Medically monitored intensive inpa-
adolescent SUD. Basic services or prevention repre-
tient treatment
sent the least restrictive level of care in the CALOCUS
■ Level IV: Medically managed intensive inpatient
model (Level 0), whereas a secure 24-hour medical
treatment
management program is the most restrictive level
When applicable, subspecifiers are used to denote (Level 6).
treatment types further:
■ D: Detoxification
■ OMT: Opioid maintenance treatment STATISTICS AND EPIDEMIOLOGICAL
■ BIO: Capable of managing complex medical TIDBITS
comorbidity
It is not uncommon for the boards to ask a couple of
■ AOD: Alcohol or drug treatment only
basic statistical questions, most commonly addressing
■ DDC: Dual diagnosis capable (the treatment facility
sensitivity, specificity, positive and negative predictive
can identify co-occurring psychiatric problems and
values, or the types of biases in scientific research.
refer to outside mental health treatment centers)
■ DDE: Dual diagnosis enhanced, capable on-site Test positive Test negative
of managing patients with co-occurring psychi- Condition True positive False negative
atric problems present (TP) (FN)
Condition False positive True negative
The PPC-2R model is presented as a matrix grid in absent (FP) (TN)
which illness dimensions are listed on the Y-axis and
Sensitivity and specificity present the means to assess
treatment types on the X-axis.
how accurate a diagnostic test demonstrates the pres-
For example, a patient with alcohol use disorder
ence or absence of a condition. They address the likeli-
and depression seeking treatment might require initial
hood of a given test outcome for persons who have or do
care in a Level IV-D (inpatient detoxification) followed
not have the condition. A test with high specificity will
by a II.5-DDE (partial hospital program that is dual
help Rule-In the condition (Specificity – IN → SPIN),
diagnosis enhanced)
whereas a high sensitivity test will help Rule-Out the
Know This: condition (Sensitivity – OUT → SNOUT). For example,
gamma-glutamyl transferase is a highly specific but not
1. The role of coercion in treatment or the impact very sensitive biomarker of alcohol use disorder.
of one’s experienced adverse consequences of In contrast, positive predictive value (PPV) and
substance use affects their motivation for treat- negative predictive value (NPV) address the likelihood
ment and is assessed in Dimension 4 (readiness of having a condition for persons who test positive
to change). or negative. As such, a test with high PPV will have a
4 ADDICTION PSYCHIATRIC MEDICINE: A COMPREHENSIVE BOARD REVIEW
large number of TPs among all those who test positive, 4. Cannabis use disorder is the most common
whereas a test with high NPV will have a large number SUD in adolescents (AUD is the second most
of TNs among all those who test negative. Note that common SUD in adolescents). In contrast,
PPV and NPV are based on the assumption that the AUD is the most common SUD in adults.
prevalence in the sample is representative of the preva- 5. Tobacco use and tobacco use disorder preva-
lence in the general population. lence decreased over the past decade.
6. Telescoping refers to the phenomenon whereby
Sensitivity = TP/(TP + FN)
SUD severity progresses faster for women than
Specificity = TN/(TN + FP)
for men, with women more likely to seek SUD
PPV = TP/(TP + FP)
treatment and be in recovery faster than men.
NPV = TN/(TN + FP)
7. PPV and NPV are conditional on the assump-
Statistic and demographic data on SUD derives mainly tion that the sample prevalence reflects the
from a few national surveys, including Monitoring the prevalence in the general population.
Future (MTF), the National Survey of Drug Use and
Health (NSDUH), and the National Epidemiologic Sur- Marijuana is the illicit drug with the highest new
vey on Alcohol and Related Conditions (NESARC). drug initiation rates, followed closely by prescription
MTF is conducted in schools and targets the youth pain medications.
with a sampling frame including only grades 8, 10, or The board examinations often ask about SUD prev-
12. NSDUH, on the other hand, is a household survey. alence in different racial and ethnic groups. Overall, it
is important to remember that racial and ethnic iden-
Know This: tity are not found to be risk factors for SUDs when
adjusted for socioeconomic factors. With that in mind,
1. In general, remember that alcohol use disorders Native Americans have the highest SUD rates in the
(AUD) are three to five times more common United States (lifetime prevalence of AUD is 44% and
than illicit drug use disorders. The numbers illicit drug use disorders are 18%).
below reflect an easy-to-memorize approxima- Racial and ethnic minorities are disproportion-
tion. Alcohol use disorder in the United States: ately affected by serious adverse health consequences
■ Past year prevalence: 10% of SUD, have less access to treatment, are less likely to
■ Lifetime prevalence: 30% to 44% achieve recovery, and more likely to engage in high-
Illicit drug use disorders in the United States: risk drug use behaviors, such as intravenous injecting.
■ Past year prevalence: 2% Roughly half of high school students experiment
■ Lifetime prevalence: 10% with the illicit drug by the time of graduation, and a
2. SUDs are roughly twice as prevalent in men third of those develop a SUD later in life. Risk factors in
than in women. Adolescents and the elderly children for future SUD:
are exceptions:
■ In adolescents, prevalence rates are compa-
■ Poor school performance
■ Poor social skills
rable between genders.
■ In the elderly, the gender gap increases with
■ Aggressive behavior and lack of self-control
■ Anxiety disorders
age, for example, men over 65 are six times
more likely than women to have AUD. SUD rates in physicians are similar to those in
3. Caffeine is the most commonly used psychoactive the general population. However, recovery rates are
substance, alcohol is the most commonly use ad- significantly higher for physicians, likely due to the
dictive substance, marijuana is the most common- fear of losing one’s license. AUD is the most common
ly used illicit substance, and opioid pain medica- SUD in physicians, and marijuana is the most com-
tions are the most commonly abused prescription monly used illicit drug. The age at which physicians
drug (except in high school students who most present to physician health programs has decreased
commonly abuse prescribed amphetamines). in recent years.
1 General Principles of Substance Use Disorders 5
Tobacco is thought of as the substance with the high- ■ Tetrahydrocannabinol (THC) is metabolized by
est addictive potential, followed by opioids, cocaine, CYP 2C9 and CYP 3A4 into 11-hydroxy-Δ9-
alcohol, benzodiazepines, and marijuana consecutively. THC, excreted in feces.
This is assessed by examining what percentage of those
who used the substance once develop an SUD: Know This:
■ 33% of those who use tobacco once develop 1. Board examinations frequently ask questions
tobacco use disorder about substance metabolism. We review here
■ 25% of those who use heroin once develop opioid some important concepts:
use disorder ■ Alcohol metabolism follows a zero-order kinet-
■ 17% of those who use cocaine once develop ic model meaning that a constant amount of
cocaine use disorder alcohol is cleared per unit time. As such, the
■ 15% of those who use alcohol once develop AUD drug dose (amount of alcohol consumed) is
■ 10% of those who use benzodiazepine once the most important factor for determining the
develop a sedative use disorder time it takes to reach a steady state.
■ 10% of those who use cannabis once develop ■ In contrast, cocaine follows a first-order ki-
Screener and Opioid Screens for aberrant Short version of Screens for AUD in the
Assessment for Patients opioid behaviors in MAST-G (SMAST-G) elderly
with Pain (SOAPP-R) patients with chronic NIAAA single-question One question: “Do
pain alcohol screen you sometimes drink
Pain Medication Screens for aberrant alcohol? (in the past
Questionnaire (PMQ) opioid behaviors in year).” Persons who
patients with chronic screen positive should
pain then receive an in-depth
Opioid Risk Tool (ORT) Screens for aberrant assessment to allow
opioid behaviors in for the determination
patients with chronic of the risk level. Brief
pain intervention and
providing advice
Recovery Attitude and Screens for resistance to
follow (per the
Treatment Evaluatory treatment
SBIRT – Screening,
(RAATE)
Brief Intervention and
URICA Screens for motivation Referral to Treatment
level for treatment model)
Alcohol Use Disorders Three questions: Tolerance, Annoyed, Screens for alcohol
Identification 1. How often do you Cut down, Eye opener
Test – Consumption have alcohol? (T-ACE)
(AUDIT-C) 2. How many SDU do TWEAK Screens for AUD in
you drink in a typical pregnant women
day? Cut down, Annoyed, Screens for alcohol
3. How often do you Guilty, Eye Opener
have six or more (CAGE)
drinks on one
CAGE-AID Screens for alcohol and
occasion?
drugs
A score of 4 or more for
men or 3 or more for SAFE-T Not SUD specific,
women is considered assesses suicide risk
positive. factors
Addiction Severity Index Assesses SUD severity PCL Not SUD specific,
(ASI) in seven potential assesses PTSD
problem areas: medical symptoms
status, employment
and support, drug use, THE NATIONAL INSTITUTE
alcohol use, legal status, ON ALCOHOL ABUSE AND
family and social status, ALCOHOLISM DRINKING
and psychiatric status. GUIDELINES
Michigan Alcohol Screens for AUD in the
■ No-risk drinking: 4 or fewer drinks per day and
Screen-Geriatrics elderly
14 or fewer drinks per week for men; 3 or fewer
(MAST-G)
drinks per day and 7 or fewer drinks per week for
women.
1 General Principles of Substance Use Disorders 7
■ At-risk drinking: More than 4 drinks per day ■ Feeling restless or irritable when attempting to
and more than 14 drinks per week for men; more cut down or stop gambling.
than 3 drinks per day and more than 7 drinks per ■ Making repeated unsuccessful efforts to control,
week for women. cut back, or stop gambling.
■ In men and women over age 65, NIAAA recom- ■ Being often preoccupied with gambling (e.g.,
mends no more than 7 drinks per week and no having persistent thoughts of reliving past gam-
more than 3 drinks per day. bling experiences, handicapping or planning the
next venture, thinking of ways to get money with
One standard alcoholic beverage or standard drink-
which to gamble).
ing unit contains 12 to 14 g of ethanol (0.6 oz of pure ■ Often gambling when feeling distressed (e.g.,
ethanol, or one 12 oz beer, one 5 oz glass of wine, one
helpless, guilty, anxious, depressed).
1.5 oz of 80-proof liquor). ■ After losing money gambling, often returning
another day to get even (“chasing” one’s losses).
BEHAVIORAL ADDICTION ■ Lying to conceal the extent of involvement with
gambling.
Beginning in the DSM-5, behavioral addictions were ■ Having jeopardized or lost a significant relation-
included in the same chapter as SUDs. Behavioral
ship, job, or educational or career opportunity
addictions are characterized by a recurrent pattern
because of gambling.
of behavior within a specific domain (gambling, ■ Relying on others to provide money to relieve des-
eating, shopping, video gaming) that ultimately
perate financial situations caused by gambling.
interferes with functioning in other domains (fam-
ily, society, work). Unlike SUDs, there are no medi- Not unlike other addictive disorders, the telescop-
cally dangerous withdrawal states from behavioral ing phenomenon has been studied in gambling disor-
addictions. ders. This refers to the finding that gambling disorder
Gambling addiction is the only diagnosable behav- in women has a later age of initiation and shorter times
ioral addiction using the DSM-5 framework. Gam- from the gambling initiation to having the disorder.
bling disorder is the most studied behavioral addiction. There are no U.S. Food and Drug Administration-
Gambling was referred to as pathological gambling in approved pharmacotherapeutic treatments for gam-
the DSM-III under the impulse-control disorders cat- bling disorder. However, naltrexone has demonstrated
egory. It was then renamed to gambling disorder in the efficacy in managing gambling urges and behavior
DSM-5 and moved to the new category of “Addictions compared to placebo in some studies. The mainstay of
and Related Disorders.” Gambling disorder usually treatment for gambling disorder involves psychothera-
starts in the early stages of life, and males tend to start pies, cognitive-behavioral therapy utilizing the relapse
during childhood or adolescence years. Its lifetime prevention model, and motivational interviewing. Par-
prevalence is estimated to be between 0.1% and 7.6% ticipation in 12-step based mutual support groups has
of the adult population in the United States. The preva- also been found to support persons with behavioral
lence of adolescent gambling disorder is higher than addictions to maintain states of recovery effectively.
that of adults. Twin studies have shown the role of a
genetic etiology as a perpetuating factor for gambling SUGGESTIONS FOR FURTHER READING
activities. People with gambling disorders are at higher Calado, F., & Griffiths, M. D. (2016). Problem gambling worldwide:
rates of comorbidity with alcohol, tobacco, and mood An update and systematic review of empirical research (2000–
2015). Journal of Behavioral Addictions, 5(4), 592–613.
disorders.
Diazgranados, N., & Goldman, D. (2020). The assessment and treat-
A diagnosis of gambling disorder is made when ment of addiction: Best practices in a direct-to-consumer age.
persons meet at least four of the following diagnostic In B. A. Johnson (Ed.), Addiction medicine: Science and practice,
criteria: (2nd ed., pp. 167–172). Elsevier.
Gowing, L. R., Ali, R. L., Allsop, S., Marsden, J., Turf, E. E., West, R.,
■ Needing to gamble with increasing amounts of & Witton, J. (2015). Global statistics on addictive behaviours:
money in order to achieve the desired excitement. 2014 status report. Addiction, 110(6), 904–919.
8 ADDICTION PSYCHIATRIC MEDICINE: A COMPREHENSIVE BOARD REVIEW
Grant, J. E., Odlaug, B. L., & Mooney, M. E. (2012). Telescoping phenom- Teesson, M., & Mewton, L. (2015). Epidemiology of addiction. 2015.
enon in pathological gambling: association with gender and comor- In M. Galanter, H. D. Kleber, & K. T. Brady (Eds.), The Ameri-
bidities. Journal of Nervous and Mental Disease, 200(11), 996–998. can Psychiatric Publishing textbook of substance abuse treatment
Mee-Lee, D., & Shulman, G. D (2018). The ASAM criteria and (5th ed., pp. 47–58). American Psychiatric Publishing.
matching patients to treatment. In S. Miller, D. Fiellin, R. Zajicek, A., & Karan, L. D. (2018). Pharmacokinetic, pharmacody-
Rosenthal, & R. Saitz (Eds.), The ASAM principles of addiction namic and pharmacogenomic principles. In S. Miller, D. Fiellin,
medicine (6th ed., pp. 433–447). Wolters Kluwer. R. Rosenthal, & R. Saitz (Eds.), The ASAM principles of addic-
American Psychiatric Association. (2013). Substance-related and tion medicine (6th ed., pp. 93–106). Wolters Kluwer.
addictive disorders. In The diagnostic and statistical manual for
mental disorders (5th ed., pp. 481–590).
REVIEW QUESTIONS
1. Which of the following are accurate risk factors E. As a result of later onset of SUD in women
for the development of alcohol use disorder? compared to men, women use substances
A. Male gender, younger, low socioeconomic for fewer years than men.
status (SES), married
4. Which of the following is true when thinking
B. Male gender, younger, low SES, single
about treating SUD in females?
C. Male gender, older, high SES, married
A. Attending women only AA groups leads to
D. Female gender, younger, low SES, married
better clinical outcomes than call and groups.
E. Female gender, older, high SES, single
B. Women respond better to medications for
2. In understanding the ASAM patient placement the treatment of SUD than men.
criteria (PPC) dimensions, encouraging one’s C. Pregnancy and breastfeeding are contrain-
spouse to attend Al-Anon is an example of seek- dications for the use of medications for the
ing to address problems in which dimension? treatment of SUD.
A. Dimension 1 D. Treating co-occurring psychiatric disorders
B. Dimension 3 leads to improved SUD outcomes.
C. Dimension 4 E. Women are treatment-resistant more often
D. Dimension 5 than men.
E. Dimension 6
5. A 36-year-old married man who enjoyed day
3. Which of the following best represents the tele- trading stocks since college started to move his
scoping effects seen in females with SUDs? money to an online casino phone application.
A. Women get intoxicated with alcohol faster Over the last 13 months, he has lost most of
as a result of higher levels of gastric aldehyde his family savings. His sleep has been affected,
dehydrogenase. as he is always thinking about going back online
B. Women get intoxicated with alcohol faster as to recover the money loss; he stopped going to
a result of body water and fat distribution. the gym, his performance at work has decreased,
C. The risk of developing alcohol use disor- and he stopped visiting his mother. What phar-
der is lower in women than it is in a man macotherapies should be considered a first-line
only when traditional gender roles are treatment for pathologic gambling?
followed. A. Naltrexone
D. Women with substance use disorders B. Lithium
experience significantly more severe adverse C. Buprenorphine
medical and psychological consequences D. Bupropion
than men. E. Topiramate
1 General Principles of Substance Use Disorders 9
6. A 68-year-old female presents to your office sleep and mood. She has had two episodes of
with her son who is worried about his mother’s bankruptcy in 2 years. What phenomenon are we
late-night visits to the casino. His father suffered observing in this patient?
from gambling “issues,” but his mother was nev- A. Rapid cycling
er like that until a year ago. He describes that his B. Interpersonal phenomenon
mother has been increasing the time at the ca- C. Telescoping phenomenon
sinos, impairing her finances, and spending less D. Early initiation
time with the family. The behavior is affecting her E. Middle-life crisis
2 COMORBIDITIES OF SUBSTANCE
USE DISORDERS
11
12 ADDICTION PSYCHIATRIC MEDICINE: A COMPREHENSIVE BOARD REVIEW
4. Pentoxifylline is a steroid medication used as a disease, cirrhosis, and hepatocellular carcinoma. Risk
part of the management of hepatitis. Its use is factors for worse prognoses include alcohol use, HCV
contraindicated in patients with gastrointesti- genotype 1, older age at the time of infection, male sex,
nal bleeding, renal insufficiency, or active infec- co-occurring HBV, and higher viral load at the time
tious processes. of treatment initiation. The medical management of
HCV includes ensuring that the person is vaccinated
Other substances can also cause direct hepatotox- against hepatitis A virus/HBV and limiting alcohol
icity, including cocaine, heroin, 3-4 methylenedioxy- consumption. Historically, HCV treatment consisted
methamphetamine (MDMA), phencyclidine (PCP), of interferon and ribavirin. Newer interferon-free
and androgenic steroids. Indirect substance-related HCV antivirals have been developed and are becom-
hepatotoxicity can be caused by drug contaminants ing more easily accessible. These include sofosbuvir,
or viral hepatitis associated with drug and alcohol use simeprevir, and ledipasvir.
discussed later.
Know This:
Infectious Comorbidities of Substance Use
Injecting drugs (with contaminated needles, syringes, 1. HIV patients should continue taking HAART
spoons, cotton swabs) or using intranasal instruments during pregnancy, although the medication
is associated with contracting HIV and hepatitis C regimen should exclude efavirenz because of
virus (HCV). Toxic shock syndrome caused by group its teratogenicity (associated with renal tube
A beta-hemolytic Streptococcus is also seen as a result defects).
of injecting drugs and causes a generalized rash, hypo- 2. Efavirenz is the antiretroviral medication most
tension, tachycardia, murmurs, abdominal pain, and commonly associated with psychiatric adverse
diarrhea. Similarly, drug and alcohol use are associ- effects, including anxiety, depression, suicidal
ated with high-risk sexual behaviors, which are also ideation, confusion, and hallucinations.
associated with contracting HIV or hepatitis B virus
(HBV). Finally, excessive alcohol use is associated with Renal Comorbidities of Substance Use
an increased risk of contracting various infectious pro-
cesses, postoperative infections, along with impaired Acute renal failure can be caused by chronic hyperten-
wound healing. sion (from alcohol or cocaine use) or rhabdomyolysis
HIV-related immune suppression and AIDS cause from seizures or volume depletion in persons using
opportunistic infections, including Pneumocystis cari- alcohol, cocaine, heroin, or bath salts. Contaminants
nii pneumonia (PCP), toxoplasmosis, or tuberculo- found in heroin can cause nephropathy. Additionally,
sis. Similarly, HIV infection and AIDS are associated chronic and excessive alcohol use can cause hepatore-
with other complications, including Kaposi sarcoma nal syndrome (renal failure in patients with acute and
and other neoplasms, HIV-associated nephropathy, severe liver failure from decompensated cirrhosis or
cardiovascular disease, and neurocognitive decline. acute fulminant hepatitis, causing jaundice, coagu-
Highly active antiretroviral therapy (HAART) is rec- lopathy, and death from hepatic and renal failure or
ommended for all HIV-positive patients and slows or variceal bleeding). Finally, infectious processes seen in
stops the progression of HIV infections, suppresses injection drug users, such as HBV or HCV or HIV, can
HIV viral loads, increases CD4 counts, and prevents cause nephrotic syndrome.
the above-noted complications. Medications used in
HAART have multiple interactions with commonly Know This:
prescribed psychiatric medications and certain drugs. 1. Rhabdomyolysis symptoms include fever, mus-
HCV is the most common chronic bloodborne cle pain, and malaise. It is diagnosed by finding
infectious agent, four times more prevalent than HIV. red blood cells in urine and myoglobin casts
Up to 60% to 80% of injection drug users are HCV pos- on sediment analysis (urine myoglobin analysis
itive. Untreated HCV can lead to progressive hepatic can take several days, so use urine blood and
2 Comorbidities of Substance Use Disorders 15
Saitz, R. (2018). Medical and surgical complications of addiction. Youdelis-Flores, C., Goldsmith, R. J., & Ries, R. K. (2018). Substance-
In S. Miller, D. Fiellin, R. Rosenthal, & R. Saitz (Eds.), The induced mental disorders. In S. Miller, D. Fiellin, R. Rosenthal,
ASAM principles of addiction medicine (6th ed.). Wolters & R. Saitz (Eds.), The ASAM principles of addiction medicine
Kluwer. (6th ed.). Wolters Kluwer.
REVIEW QUESTIONS
1. A 46-year-old woman with a long history of an the morning. When meeting with him, he tells
AUD, hepatitis C, and cirrhosis is referred to you that he drinks alcohol to “self-medicate his
your clinic for treatment. She expresses a de- anxiety and panic attacks.” Which of the fol-
sire to discontinue her alcohol consumption. lowing is the most likely explanation for this pa-
In addition to psychotherapeutic interventions, tient’s anxiety and panic attacks?
which of the following pharmacologic agents is A. Cocaine intoxication
the medication of choice for treating this pa- B. Alcohol intoxication
tient’s condition? C. Alcohol withdrawal
A. Disulfiram D. Generalized anxiety disorder
B. Gabapentin E. Panic disorder without agoraphobia
C. Ondansetron
4. In patients with comorbid SUDs and depres-
D. Acamprosate
sion, which of the following factors is most
E. Naltrexone
strongly associated with improved response to
2. A 59-year-old man with opioid and AUD is cur- antidepressant treatments?
rently hospitalized for an abscess. He reports A. Using selective serotonin reuptake inhibi-
depression and sleep disturbances. Which of tors (SSRI) antidepressants
the following would be the best time to perform B. Diagnosis of depression made while patient
psychiatric evaluation to determine whether he is actively drinking
would benefit from treatment for an underlying C. Diagnosis of depression made at least
depressive pathology? 1 week after patient discontinues drinking
A. His treatment should be started in the D. Using antidepressant medications without
emergency room, as soon as he arrives at the psychosocial interventions to treat depres-
hospital. sive symptoms
B. His treatment should be started after com- E. Greater initial severity of the depressive
pleting medically assisted detoxification but symptoms
before starting maintenance treatment.
5. A 42-year-old man with severe AUD is admitted
C. He should be screened for depression
into a combined program, including medically
on admission and reassessed frequently
supervised alcohol detoxification followed by a
thereafter.
long-term residential rehabilitation program.
D. He benefits only after being sober from all
He reports that he has been drinking 8 to 10
drugs and alcohol for at least 1 month.
alcoholic drinks daily for the past 6 years. Also,
E. Never, this man’s depression results from
he describes symptoms of major depressive dis-
his heroin and alcohol use.
order for the past 7 months. Which of the fol-
3. A 34-year-old man with severe AUD, cocaine lowing would be the most appropriate strategy
and ketamine use is referred to you for chronic to treat his depressive symptoms?
and severe anxiety and frequent panic attacks. A. His alcohol use causes this person’s depres-
The panic attacks are associated with signifi- sion. Abstinence during the residential
cant avoidance behaviors leading to him to skip rehabilitation program will be curative for
work often. The panic attacks often happen in his depression.
2 Comorbidities of Substance Use Disorders 17
B. Because of the comorbid AUD, antidepres- 1 month ago, he attempted suicide by inten-
sant medications are not indicated. This tionally overdosing on heroin. Which of the fol-
patient’s depression should be treated lowing is the most appropriate next step of this
using intensive cognitive behavioral therapy evaluation?
alone. A. Order stat laboratory testing for HIV and
C. This patient’s depression has lasted more HCV.
than 6 months. This is consistent with a B. Assess the severity of the patient’s depres-
diagnosis of major depressive disorder, and sion and current suicidal thoughts.
he should be prescribed antidepressant C. Inform the patient that he is not eligible
medication immediately. for treatment with buprenorphine because
D. Start an antidepressant medication after this he has a history of obtaining it from illicit
patient completes the residential rehabilita- sources.
tion program if he remains symptomatic. D. Inform the patient that he is not eligible for
E. Start an antidepressant medication after treatment with buprenorphine because of
this patient completes a medically super- his recent suicide attempt.
vised alcohol detoxification if he remains E. Prescribe fluoxetine.
symptomatic.
8. A patient with severe opioid use disorder who
6. A 42-year-old woman is brought to the emer- injects heroin is referred to your office for an
gency room by emergency medical services for evaluation. The patient tells you that he is
chest pain and dyspnea. ST depressions are interested in being prescribed medication to
seen on electrocardiogram. The patient admits treat opioid use disorder. The patient tells you
to using cocaine regularly, including snorting 1 that his opioid use began 10 years ago when
g of cocaine on the day of admission. Assum- he was prescribed opioid pain medications for
ing that her drug use played a causal role in her chronic and severe back pain. He continued
current presentation, what is cocaine-induced using illicitly obtained opioid pain medica-
ischemia’s physiological mechanism? tion after his physician “cut him off ” and ulti-
A. Increased cardiac workload and systemic mately transitioned to heroin use. The patient
vascular resistance reports more than seven suicide attempts by
B. Dilated cardiomyopathy intentional opioid overdose but denies any
C. Superficial venous thrombosis current suicidal thoughts or depression. The
D. Severe hypotension following a vasovagal patient is hopeful that treatment will improve
episode his condition. During your evaluation, the pa-
E. Coronary thromboembolism tient appeared to be in insignificant distress
7. A 34-year-old man with severe opioid use dis- as a result of opioid use disorder. What is the
order and comorbid major depressive disorder most appropriate next step?
comes into the clinic to evaluate and consider A. The patient should be assessed for suicide
treatment with buprenorphine. The patient tells risk and consideration for inpatient hospi-
you that he has been injecting heroin at least talization given prior suicide history.
four times daily on and off for the past 15 years. B. Prescribe clonidine, ibuprofen, and loper-
He also acknowledges using illicitly obtained amide for his withdrawal symptoms and
buprenorphine on multiple occasions in the schedule him for a follow-up appointment
past in an attempt to treat his opioid use dis- in 1 week.
order on his own. During your evaluation, the C. Perform a naloxone challenge test. If no
patient admits that his father died of an opi- evidence of withdrawal is seen, prescribe
oid overdose a month ago, and as a result, he oral naltrexone in preparation for starting
has been feeling very depressed. He adds that long-acting injectable naltrexone.
18 ADDICTION PSYCHIATRIC MEDICINE: A COMPREHENSIVE BOARD REVIEW
D. Send the patient to the emergency room 11. Which of the following is accurate regarding
to be admitted for a medically supervised the effects of chronic excessive alcohol use?
detoxification. A. Brain imaging studies reveal frontal lobe hy-
E. Assess the severity of his opioid withdrawal pertrophy, and neuropsychological testing
and begin initiation with buprenorphine. reveals cognitive deficits.
B. Neuropsychological testing of cognitive
9. A 17-year-old boy presents for an intake eval-
function does not reveal any impairments.
uation. He tells you that he has been using
C. No changes are noted on brain imaging,
heroin and cocaine together by injection for
and neuropsychological testing reveals cog-
6 months and admits to sharing needles. He
nitive deficits.
mentions that he has been experiencing chest
D. Brain imaging studies reveal a loss of brain
discomfort, low energy, and generalized fatigue
tissue, and cognitive impairments are noted
during the examination. You find him febrile,
on neuropsychological testing.
tachycardic, and normotensive on physical ex-
E. Elevated concentration of serotonin is
amination. What is the most appropriate next
found in the cerebrospinal fluid.
step?
A. Refer him to a methadone clinic. 12. Which of the following personality disorders
B. Send him to the emergency room. has the highest comorbid SUD rates?
C. Obtain laboratory testing for HIV and HCV. A. Borderline personality disorder
D. Refer him to a medically supervised inpa- B. Obsessive-compulsive personality disorder
tient detoxification program. C. Dependent personality disorder
E. Initiate treatment with buprenorphine as D. Schizoid personality disorder
soon as he presents with moderate symp- E. Antisocial personality disorder
toms of opioid withdrawal.
13. The FDA approves several medications for the
10. A 27-year-old woman with AUD and a his- treatment of opioid use disorder. Which of
tory of cocaine and heroin use by injection is the following is an FDA-approved medication
referred to your methadone clinic. On metha- for treating co-occurring PTSD and opioid use
done initiation, she discontinues all illicit opi- disorder?
oid use but continues to drink alcohol exces- A. Buprenorphine
sively and inject cocaine. Laboratory testing B. Naltrexone
reveals that she is HIV positive. When is the C. Methadone
most appropriate time for her to begin treat- D. Baclofen
ment for HIV using highly active antiretroviral E. None of the above
treatment (HAART)?
A. Only if she develops Kaposi sarcoma
B. Only if she just continues all injection drug
use
C. Only if she discontinues all alcohol use
D. When her CD4 count is less than 200
E. Immediately
3 NEUROBIOLOGY OF ADDICTION
GENERAL TERMS AND CONCEPTS in animal models by pairing substance use with
an adverse consequence or progressive-ratio re-
Following initiation of substance use, using is typically inforcement schedule.
impulsive and a positively reinforcing experience. The ■ Delayed discounting: It is the tendency to
chronic and repeated use of it leads to possible tolerance choose immediate, smaller rewards over larger,
accompanied by diminishing returns on the substance’s delayed rewards. Higher rates of discounting
positively reinforcing effects. In parallel, there is an are associated with substance use disorders
increase in the negative reinforcing effects, such as using (SUDs) and many negative life outcomes.
the substance to avoid withdrawal symptoms. Over ■ Positive reinforcement: A reward or punish-
time, substance use becomes less impulsive and reward- ment is paired with behavior in an additive
ing and more compulsive and habitual. There is strong fashion, making it more likely that the behav-
evidence of how the brain changes in substance use that ior will be repeated or extinguished. Example:
explains long-term changes in behavior and motivation, Experiencing euphoria following heroin use,
the key points of which will be reviewed in this chapter. or withdrawal symptoms after substance use
discontinuation.
Know This: ■ Negative reinforcement: A stimulus is removed
19
20 ADDICTION PSYCHIATRIC MEDICINE: A COMPREHENSIVE BOARD REVIEW
THE ADDICTION CYCLE AND ■ The “reward pathway” refers to the mesolimbic
RELATION TO ANIMAL MODELS dopamine pathway, which includes dopami-
nergic cell bodies in the ventral tegmental area
The current body of human and animal research pro- (VTA; midbrain) that project to the nucleus
vides robust support for a conceptual model of addic- accumbens (NAc) in the ventral striatum (part of
tion, first described by Dr. George Koob that involves the basal ganglia).
three recurring stages: (1) binge/intoxication, (2) with- ■ The acute effects of drugs are generally mediated
drawal/negative affect, and (3) preoccupation/antici- through one of several mechanisms: they can
pation. The addiction cycle is an organizing principle directly increase extracellular dopamine release in
to understand the neurochemical and neurobiological the reward pathway (e.g., amphetamines, cocaine),
changes that a person undergoes as their SUD intensi- activate or inhibit channels (e.g., phencyclidine,
fies, for example, as they move from impulsive to com- ketamine, or ethanol), or mimic neurotransmit-
pulsive substance use. Definitions of these stages are ters by activating receptors (opioids, cannabis,
as follows: nicotine). Table 3.1 summarizes substances of
1. Binge/intoxication: The stage in which a person abuse and affected neurotransmitter systems.
uses an intoxicant and experiences pleasure and
reward (positively reinforcing effect). For exam- TA B L E 3 .1
ple, a person experiences a pleasurable high Common Drugs of Abuse and Primary
from heroin or cocaine. Receptor Signaling Mechanism
2. Withdrawal/negative affect: The stage in which
Molecular Target Receptor Signaling
a person experiences the negative emotional Drug and Action Mechanism
and physiological effects in the absence of the
Nicotine Nicotinic Ligand-gated
intoxicant and removes these negative effects on acetylcholine channel
resumption of substance use (negatively rein- receptor agonist
forcing effect). For example, opioid or alcohol Alcohol GABA-A receptor Ligand-gated
withdrawal. agonist, NMDA channel
3. Preoccupation/anticipation: The stage in which receptor antagonist
a person seeks out an intoxicant after a period of Cannabinoids CB1 and CB2 Gia
abstinence (stress-, cue-, or drug-induced rein- receptor agonist
statement). For example, a relapse of alcohol Opioids Mu-, delta- and Gi
after decades of abstinence following a stressful kappa-opioid
receptor agonist
life event.
Cocaine Inhibits dopamine Gi and Gs
In the subsequent sections, we will describe each transporter,
stage in more detail and the relevant animal models, increasing
neuroanatomy, neurocircuitry, and neurotransmit- extrasynaptic DA
ter systems that are frequently tested on in board Amphetamine Stimulates dopamine Gi and Gs
examinations. release, increasing
extrasynaptic DA
Phencyclidine NMDA glutamate Ligand-gated
Stage I: Binge/Intoxication receptor antagonist channels
In this stage, an individual consumes a substance that Ketamine NMDA glutamate Ligand-gated
is acutely experienced as rewarding and positively rein- receptor antagonist channels
forcing. This stage’s neurobiological processes involve Hallucinogens 5-HT2A agonist Gaq
the acute effects of substances, activation of reward cir- aG
ireceptors couple D2-like receptors and Gs receptors couple
cuitry, development of incentive salience circuits, and D1-like receptors, both of which are important for their
reinforcing effects.
drug-seeking habits. Key points to remember regard- 5-HT2A, 5-Hydroxytryptamine 2A; DA, dopamine; GABA,
ing this stage: γ-aminobutyric acid; NMDA, N-methyl-D-aspartate.
3 Neurobiology of Addiction 21
■ “Incentive salience” refers to the phenomenon of the specific biological effects of substances on the
cue learning, in which associated stimuli become reward pathway:
conditioned with a substance’s rewarding effect ■ Opioids inhibit γ-aminobutyric acid (GABA)-
to the point in which the associated stimuli alone ergic interneurons in the VTA, which disinhibit
can induce motivation to drug-seek and dopa- VTA dopaminergic neurons, thus increasing
mine release. Incentive salience is regulated by dopaminergic neurotransmission. They also act
glutamatergic projections from the prefrontal on opioid receptors directly in the NAc.
cortex (PFC) to dopamine neurons in the VTA, ■ Nicotine activates VTA dopamine neurons
■ The dorsal striatum (part of the basal ganglia) is believed to inhibit GABA-ergic nerve terminals
key for habit strengthening and development of in the VTA, thus disinhibiting VTA dopamine
compulsive substance use. neurons and increasing dopaminergic neuro-
transmission. Alcohol also activates endog-
Know This: enous opioid pathways.
■ PCP and ketamine act by inhibiting postsynap-
Stimulants are the main drugs of abuse that directly tic N-methyl-D-aspartate (NMDA) glutamate
increase dopaminergic neurotransmission in the receptors in the NAc.
NAc; other drugs of abuse increase dopaminergic ■ Cannabis activates CB1 receptors on NAc cell
neurotransmission indirectly. Fig. 3.1 summarizes the bodies, as well as on glutamatergic and GABA
acute effects of substances on the reward pathway. ergic nerve terminals in the NAc.
These are some key features to remember regarding
Nicotine
alcohol +
Opioid
Opiates peptides Glutamate inputs
_ (e.g., from cortex)
VTA Alcohol
interneuron Opiates ?
GABA _ PCP
_
Alcohol ? Stimulants
+
+
DA DA
Nicotine
Cannabinoids _
+
Glutamate
inputs VTA NAc
(e.g., from
amygdala
PPT/LDT)
Fig. 3.1 ■ Scheme of acute effects of substances on the nucleus accumbens and ventral tegmental area. (From Nestler, E. J.
(2005). Is there a common molecular pathway for addiction? Nature Neuroscience, 8(11), 1445–1449. https://doi.org/10.1038/
nn1578).
22 ADDICTION PSYCHIATRIC MEDICINE: A COMPREHENSIVE BOARD REVIEW
TA B L E 3 . 2
Summary of the Most Important Animal Models Used to Study Addiction, Categorized by Relevant
Stage of the Addiction Cycle
Stage of Addiction Animal Models Description
I. Binge/intoxication Self-administration The animal receives a drug dose by performing a discrete response, such as pushing
a lever. The pattern of response required for a dose depends on the reinforcement
schedule (e.g., fixed or variable-interval schedule).
Conditioned place A drug is paired with one specific environment, whereas placebo is paired with
preference another. The choice of the animal to spend time in one environment or another is a
measure of conditioned place preference.
Brain stimulation ICSS, e.g., the classic Olds and Milner (1954) study in which animals pressed a
reward lever to self-administer a stimulus delivered via brain-implanted electrode.
II. Withdrawal/negative Conditioned place Animals are exposed to the effects of drug withdrawal in one environment and
affect aversion are not in withdrawal in another. The choice of the animal to spend time in one
environment or another is a measure of conditioned place aversion.
Brain stimulation ICSS as described above; animals in withdrawal show increased ICSS thresholds
reward (e.g., decreased ICSS reward).
Elevated maze test A maze is placed approximately 2 feet above ground and an animal is placed at
(anxiety) the intersection of a cross, where two arms are dark and walled off, and the other
two are open and well lit. An anxious animal will generally spend more time in
the “safer” region (the dark, walled off arms), and an animal’s behavior in this
paradigm is very sensitive to drug withdrawal.
Defensive burying An animal is placed in a box filled with woodchip bedding and an electrified probe
(anxiety) protruding into the box. After touching the probe and receiving a mild shock, the
active response is generally to bury the probe with the woodchips; observers will
then measure the time to burying (latency), the height of the woodchip mound,
total time spent burying, and number of burying acts.
III. Preoccupation/ Drug-induced Exposure to a reinforcing drug can reinstate drug use after the behavior has been
anticipation reinstatement extinguished.
Cue-induced Cues previously paired to drug use act as a conditioned stimuli and lead to
reinstatement reinstatement of drug self-administration after the behavior has been extinguished.
Stress-induced Reinstatement of drug self-administration provoked by stress; first shown with
reinstatement intermittent foot shock stressor in rats leading to reinstatement of substance use.
ICCC, Intracranial self-stimulation.
3 Neurobiology of Addiction 23
environment, whereas substances that have aver- perturbations in withdrawal. Implicated in this
sive effects will cause animals to prefer the pla- are perturbations in the HPA axis, CRF, dynor-
cebo-paired environment. phin, norepinephrine, and NPY.
■ Withdrawal from all substances leads to acti-
that withdrawal is an aversive stimulus for animals self-awareness, attention, interoception, and
and that they will avoid cues associated with it. salience attribution. Deficits in PFC function-
■ Elevated maze test: A maze is elevated and an ing are associated with more negative outcomes,
animal is placed at the intersection of a cross and chronic substance use can damage the PFC’s
where two arms are dark and walled off, and the selective regions.
other two are open and well lit. An anxious ani- ■ The anterior cingulate cortex and orbitofrontal
mal will generally spend more time in the “safer” cortex also modulate activity in the VTA-NAc
(dark and walled off) arms. An animal’s behav- through glutamatergic projections. Reduced corti-
ior in this paradigm is very sensitive to drug cal activity in these areas has also been linked to
withdrawal. increased impulsive and compulsive substance use.
■ Defensive burying test: The animal is placed in ■ In the presence of frontal lobe deficits, excessive
a box filled with woodchip bedding and a pro- incentive salience to drug-associated cues can
truding electrified probe. After receiving a shock lead to relapse and uncontrolled substance use.
from the probe, the animal generally buries the ■ During abstinence, an important risk factor for
probe with woodchips; observers will measure relapse is cue-induced drug cravings. Cravings’
variables related to burying including the time to development involves multiple cellular processes,
burying, the height of the woodchip mound, and but a key pathway involves AMPA-mediated syn-
time spent burying. aptic remodeling of the medium spiny neurons
of the NAc.
Know This:
Withdrawal from substances produces higher anxi- Know This:
ety-like responses in the elevated maze and defensive In the preoccupation/anticipation stage, hypofrontality in
burying test, and both of these responses can be re- the PFC, orbitofrontal cortex, and anterior cingulate
versed by administration of CRF antagonists. cortex contributes to the inability to modulate the
drive to use drugs despite the presence of negative
Stage III: Preoccupation/Anticipation consequences.
The preoccupation/anticipation stage of addiction best
captures the concept of addiction as a chronic, relaps- Preoccupation/Anticipation: Animal Models
ing, and remitting disease. It describes the stage of Animal models of “craving” are divided into reward
addiction in which one seeks a substance following cravings and relief cravings. Reward cravings are
a period of abstinence. The construct of drug “crav- provoked by stimuli previously coupled with drug
ing” is generally linked to this stage as well, although self-administration, and stressful situations provoke
it has proven difficult to study in humans and does not relief cravings. Animal models of reward craving
always correlate with relapse. Key alterations relevant include drug-induced reinstatement and cue-induced
to the preoccupation/anticipation stage include (1) reinstatement; the animal model most relevant to relief
deficits in PFC functioning leading to loss of executive cravings is stress-induced reinstatement. Among ani-
control, (2) excessive incentive salience for a substance mals for whom drug self-administration was estab-
when relevant cues are presented to the individual, and lished and subsequently extinguished, reinstatement of
(3) the development of drug cravings. drug-seeking behavior can occur following exposure
■ The PFC is the key brain region implicated in the to a reinforcing drug (drug-induced reinstatement)
loss of control over substance use that character- or a cue previously paired to drug use (cue-induced
izes this stage of addiction. The PFC regulates the reinstatement). On the other hand, relief cravings are
limbic reward region and is critical for higher- induced by stressful stimuli, for example, foot shocks
order executive functioning, for example, self- or social defeat experiments. Animal models of stress-
control, emotional regulation, inhibitory control, induced reinstatement demonstrate that stress can
3 Neurobiology of Addiction 25
lead to reinstatement among animals where drug self- have substantive diagnostic value and treatment impli-
administration was previously extinguished. cations for SUDs. In this section, we review:
1. Heritability of SUDs and current estimates.
Know This:
2. Contribution of genetic linkage and association
The board examination may test you on the brain studies.
regions involved in drug, cue, and stressed-induced 3. Key candidate genes.
reinstatement. In summary: 4. Epigenetic contributions.
■ Drug-induced reinstatement is localized to the 5. Genetic overlap with psychiatric conditions.
glutamatergic circuit involving the medial pre- The heritability of a trait or disorder refers to the pro-
frontal cortex and ventral striatum. portion of the variability attributed to genetic factors and
■ Cue-induced reinstatement involves the basolat-
is typically estimated through family, adoption, and twin
eral amygdala with hypothesized feed-forward studies. Based on national surveys of adult twins, herita-
inhibition of the prelimbic prefrontal cortex. bility is lowest for hallucinogen abuse (0.39) and highest
■ Stress-induced reinstatement depends on acti-
for cocaine abuse (0.72), placing SUDs among the most
vation of CRF and norepinephrine in the amyg- highly heritable of psychiatric disorders (Fig. 3.2).
dala and VTA. The finding of moderate to high heritability for
SUDs is a precondition for searching for relevant
The three stages of addiction and their key features genetic polymorphisms using molecular genetic
are summarized in Table 3.3. approaches. Two approaches used to identify genetic
causes of variability in SUDs are (1) genetic linkage
GENETICS OF ADDICTION and (2) genetic association studies.
SUDs are heritable conditions comparable to other ■ Genetic linkage studies search for chromo-
highly heritable psychiatric disorders, such as schizo- somal regions co-inherited with the phenotype
phrenia and autism. With the development of rapidly of interest. The major aim is to identify risk
improving genetic techniques and research studies genes within these regions on follow-up studies.
with unprecedented sample sizes, we are approach- Genetic linkage studies have identified important
ing an era in which knowing individual genotypes will susceptibility loci for a number of SUDs, such as
TA B L E 3 . 3
Summary of the Three Stages of Addiction
Stage of Addiction Definition Key Brain Regions Key Neurotransmitters
Binge/intoxication Stage of addiction in which the person uses an Nucleus accumbens Dopamine (directly or indirectly
intoxicant and experiences the rewarding and Ventral tegmental area for all drugs)
reinforcing effects. Opioid system (opioids, alcohol,
Dorsal striatum
cannabis)
Serotonin (alcohol, stimulants)
Endogenous cannabinoid system
GABA (alcohol)
Withdrawal/ Stage of addiction in which the person Extended amygdala Corticotrophin-releasing factor
negative affect experiences the negative physiological Habenula Dynorphin
and emotional state in the absence of the Neuropeptide Y
substance.
Norepinephrine
Preoccupation/ Stage of addiction in which one seeks the Prefrontal cortex Glutamate
anticipation substance after a period of abstinence. Orbitofrontal cortex Dopamine
Anterior cingulate cortex
26 ADDICTION PSYCHIATRIC MEDICINE: A COMPREHENSIVE BOARD REVIEW
1.0
0.8
− range
0.6
h2 +
0.4
0.2
Mean
0
59)
97)
)
570
212
659
758
206
,62
,89
,49
3,3
6,9
(4,
(2,
(7,
(4,
(2,
(10
l (9
s (3
g(
e(
ens
nts
bis
ne
oho
ive
iate
lin
ng
n
ffei
cai
nna
la
mb
g
oki
dat
Alc
mu
Op
ino
Co
Ca
Ca
Ga
Sm
Se
lluc
Sti
Ha
the ADH and GABRA2 gene cluster, which are protein. CREB and ΔFosB are the two main transcrip-
relevant for risk of alcohol use disorder. tion factors relevant for drug-induced neuroplasticity and
■ Genome-wide association studies (GWAS) have long-term changes in addictive behavior caused via epi-
largely replaced genetic linkage studies in the genetic modification. The actions of CREB and ∆FosB can
search for candidate genes. GWAS is typically be summarized as follows:
performed by querying the genome with microar- ■ CREB rapidly increases in the NAc with opioids,
rays of thousands to millions of genetic markers
cocaine, and alcohol and leads to upregulation of
to examine whether particular alleles are com-
dynorphin.
mon among individuals with the disease versus ■ CREB rapidly decreases in the central nucleus of
controls. GWAS for SUDs have largely replicated
the amygdala with alcohol and nicotine and leads
prior candidate gene findings, and have identified
to reductions in levels of NPY.
novel genes such as CHRNA5/A3/B4 on chromo- ■ ∆FosB accumulates slowly with substance use and
some 15 (encoding neuronal nicotinic acetylcho-
remains stably high for months following absti-
line receptor subunits) that are associated with
nence. It increases the positive rewarding effects
tobacco, alcohol, and cocaine use disorders.
of substances and susceptibility to addiction.
Notable genes that may be tested are presented in Other important discoveries include the prominent
Table 3.4. genetic overlap between substance use disorders and
Epigenetic modification is another mechanism by several psychiatric illnesses. Epidemiological studies
which substance use is believed to alter gene expression have established the significant comorbidity between
and result in longer-term changes in neuralplasticity and substance use disorders and most psychiatric illnesses,
behavior. Epigenetics refers to DNA modification that does and this overlap has been supported by genetic studies
not involve changes in DNA base pair sequence, but rather that have identified overlapping risk alleles. Examples of
involves changes in gene expression through transcription this are ADHD and tobacco use disorders. Both have
factors and histone modification. The best-known tran- been associated with monoamine regulating genes,
scription factors relevant for drug-induced neuroplasticity including the dopamine D4 and D5 receptor genes
are ∆FosB and cAMP response element-binding (CREB) (DRD4 and DRD5), the dopamine transporter (DAT1)
3 Neurobiology of Addiction 27
TA B L E 3 . 4
Identified Genetic Variants Relative to Specific Substance Use Disorders
Substance Gene Relevance
Nicotine
CYP2A61B Involved in metabolism of nicotine; if homozygous, more likely to be a heavier smoker
GABRA2 Impulsivity and external behavior problems in nicotine and alcohol use disorder
GABRA6 Implicated in flushing reaction, as well as impulsivity
CHRNA5 Increased risk of nicotine use disorder
DRD2 Associated with increased risk of smoking and reward deficiency syndrome
Alcohol
AUTS2 Prefrontal cortex
SLC6A4 Implicated in activation of amygdala and linked to psychiatric issues in alcohol use
OPRM1 Moderates naltrexone response; mediates greater reduction of heavy drinking in alcohol users
prescribed naltrexone
ALDH1A1 Linked to epigenetic changes that occur in patients who suffer childhood adversity leading to later
alcohol abuse
ADH1B Associated with upper GI cancers and alcohol consumption
ADH2 Increases risk for alcohol use disorder
Opioids
ALDH5A1 Associated with nonresponsiveness in patients who receive maintenance therapy for opioid use
disorder
ORPD1 Associated with treatment efficacy with long-term use of methadone
CYP2D6 Associated with risk of respiratory depression particularly among European Whites who abuse
codeine
OPRM1 Variants A118G and C17T SNPs are most important: A118G variant associated with increased risk of
heroin addiction and alcohol abuse
Cocaine
CAMK4 Associated with higher susceptibility to cocaine addiction
PER1 Associated with changes in circadian rhythm and cocaine addiction
VNTR Associated with cocaine-induced paranoia
H3K9E3 Associated with cocaine use
Marijuana
FAAH Associated with striatal activation to marijuana cues and linked to withdrawal and cravings in
patients who use marijuana
CRN1 Encodes CB1 receptor
AKT1 Associated with psychosis in cannabis users
COMT Associated with schizophrenia in cannabis users
GI, Gastrointestinal; SNPs, single nucleotide polymorphisms.
Here we provide examples of medications used for norepinephrine, and brain-derived neurotrophic fac-
SUDs that follow each of these principles and are fre- tor. Glutamatergic input from the prefrontal cortex,
quently asked about on board examinations. Subse- hippocampus, and amygdala is essential in modulating
quent chapters on individual substances of abuse will dopamine release in the NAc and VTA. Medications
review treatment approaches in greater depth. with glutamatergic actions are of interest in addiction
Examples of medications that block drug targets treatment; some examples of medications with gluta-
include receptor antagonists, for example, naltrexone (an matergic actions include acamprosate, topiramate,
opioid receptor antagonist used for opioid and alco- and N-acetylcysteine.
hol use disorder) and rimonabant (an inverse agonist
that acts as an antagonist at the cannabinoid 1 receptor; REFERENCE
researched but not approved for cannabis use disorder). Olds, J., & Milner, P. (1954). Positive reinforcement produced by
Other research efforts are focused on developing vac-
##
REVIEW QUESTIONS
1. Which of the following animal models would be 2. Which of the following is believed to be a key neu-
most appropriate in studying the aversiveness (i.e., romodulator for reducing stress and counteracting
negative reinforcing effects) of opioid withdrawal? the effects of corticotropin-releasing factor (CRF)
A. Conditioned place preference A. Dynorphin
B. Conditioned place aversion B. Neuropeptide Y
C. Self-administration C. Norepinephrine
D. Drug-induced reinstatement D. Adrenocorticotropic hormone
E. Cue-induced reinstatement E. Glutamate
3 Neurobiology of Addiction 29
Evidence of alcohol consumption has been noted as ■ Alcohol undergoes first-pass metabolism in the
far back as 9,000 years ago by Neolithic farmers in stomach due to alcohol dehydrogenase (ADH) in
Northern China, and recipes for producing alcohol the gastric mucosa.
have been discovered worldwide for thousands of ■ Alcohol is metabolized by ADH to acetaldehyde,
years. Dr. Benjamin Rush is believed to have been the which is toxic and a known carcinogen. Acetal-
first person to characterize excessive alcohol use as a dehyde is then metabolized by aldehyde dehy-
disorder in 1784. By the late 1800s, temperance move- drogenase (ALDH) to acetate, which is converted
ments sprung up across the United States, inspired to water and carbon dioxide for easy elimination
mainly by religious groups who considered drunk- (Fig. 4.1). CYP2E1 also metabolizes alcohol at
enness a national threat. In 1920, Prohibition was higher blood concentrations and produces reac-
enacted in the form of the 18th Amendment, which tive oxygen species as a byproduct.
prohibited the manufacture and sale of alcohol. This ■ ADH and ALDH are responsible for metaboliz-
was later repealed in 1933 because of its unpopular- ing a range of alcohols encountered in the body,
ity. In 1935, Alcoholics Anonymous (AA) was founded including ethanol, methanol, ethylene glycol, and
by Bill Wilson and Dr. Robert Smith as a peer-based isopropyl alcohol (Fig. 4.1).
fellowship to provide group support to other recover- ■ Alcohol metabolism follows zero-order kinetics
ing alcoholics when no other treatment options were because ADH saturates at very low serum levels of
available. Since then, enormous advances have been ethanol, leading to constant levels of elimination.
made in understanding alcohol use disorder (AUD), ■ Alcohol is metabolized at roughly 0.015 g/dL per
including the biological effects of alcohol, treatment hour. The rate of alcohol metabolism is depen-
approaches for alcohol intoxication and withdrawal, dent on factors including age, sex, body weight,
development of medications, and establishment of and differences in expression and enzyme iso-
evidence-based therapies for AUD of which will be types of ADH and ALDH. Women have a higher
reviewed in this chapter. blood alcohol concentration than men for an
equivalent amount of alcohol use because of
PHARMACOLOGY OF ALCOHOL lower levels of gastric ADH and first-pass metab-
olism and lower total body water. Lower body
Chemical Structure and Metabolism weight and increased age are both associated
Fig. 4.1 shows the chemical structure and metabolism with a slower metabolism. Gastric ADH levels
of ethanol and several other alcohols that are clinically decline with age.
relevant to know about and which you may be asked ■ The peak effect of ethanol occurs 30 minutes
about on the board examination. after consumption.
31
32 ADDICTION PSYCHIATRIC MEDICINE: A COMPREHENSIVE BOARD REVIEW
TA B L E 4 .1
Methanol, Ethylene Glycol, and Isopropyl Alcohol
Alcohol Byproducts Clinical Findings Unique Laboratory Findings Treatment
Methanol Formic acid (toxic) Blindness, visual blurring, Anion gap metabolic acidosis, Fomepizole +/– dialysis,
and central scotomata elevated lactate Ethanol
Ethylene glycol Glycolic acid (toxic) Hematuria, oliguria, flank Anion gap metabolic acidosis, Fomepizole +/– dialysis,
pain, fluorescent urine elevated lactate, urine calcium
oxalate crystals
Isopropyl alcohol Acetone (nontoxic) Fruity breath, intoxication No anion gap, ketosis without Supportive
syndrome similar to ethanol metabolic acidosis
include nausea and vomiting, tremor, sweating, anxiety, ■ Barbiturates such as phenobarbital can be used
agitation, tactile disturbance, auditory disturbances, to treat alcohol withdrawal but are far less com-
visual disturbances, headache, and clouding of sen- monly employed than benzodiazepines because
sorium. Scores greater than 15 indicates severe with- of their narrow therapeutic index.
drawal. CIWA scores are also used to guide the need for ■ Other medications that can be used for treat-
medication dosing. Individuals with a CIWA score less ing alcohol withdrawal include anticonvulsants
than or equal to 8 generally do not require medications (such as carbamazepine, gabapentin, levetirace-
to manage alcohol withdrawal symptoms. tam and valproic acid) and adrenergic agents
Administration of the CIWA-Ar relies on patient’s (such as clonidine and dexmedetomidine).
ability to communicate and should only be used if ■ Benzodiazepines-sparing protocols for the treat-
other etiologies of the patient’s condition have been ment of alcohol withdrawal (such as protocols
excluded (e.g., delirium, dementia, acute psychosis, using anticonvulsant medications) are particu-
opioid withdrawal). If the patient is unable to commu- larly useful for treating alcohol withdrawal in
nicate, the CIWA-Ar is not an appropriate assessment outpatient settings.
tool. Other assessment tools such as the Richmond ■ For patients with refractory DTs, treatment options
Agitation-Sedation Scale (RASS) can be used to man- include phenobarbital or propofol. Propofol is a
age withdrawal in patients who are intubated or in the sedative-hypnotic that acts as a GABA-A receptor
intensive care setting. agonist and NMDA receptor antagonist. Intubation
is frequently necessary if phenobarbital or propofol
Know This: are given.
The CIWA does not include scored items for vital signs. Two commonly employed strategies are fixed mul-
The decision not to include vital signs was based on tiple daily dosing and symptom-triggered treatment for
data showing that pulse and blood pressure did not individuals requiring medications to manage alcohol
correlate with the severity of alcohol withdrawal than withdrawal. In fixed multiple daily dosing schedules,
the other signs and symptoms included in the CIWA-Ar. patients are typically placed on a gradual, tapering
benzodiazepine schedule once their withdrawal symp-
toms are stabilized. In symptom-triggered treatment,
Treatment of Withdrawal benzodiazepines are only administered if patients are
The mainstay of pharmacological treatment for alcohol showing sufficient symptoms of alcohol withdrawal.
withdrawal are benzodiazepines; commonly used ben- The current evidence favors symptom-triggered treat-
zodiazepines are those with longer half-lives, including ment, as it reduces the average length of stay, the total
diazepam, chlordiazepoxide, or clonazepam. Some key dosage of administered benzodiazepines, and the level
points regarding the treatment of alcohol withdrawal of patient sedation. Despite this, the fixed-dose sched-
include the following: ule is still widely used. CIWA-Ar can be used to guide
the need for symptom-triggered medication and ancil-
■ Benzodiazepines are cross-tolerant with alcohol lary rescue medication if a patient is on a fixed-dose
and are considered the first line for the treatment schedule.
of alcohol withdrawal.
■ Benzodiazepines are typically metabolized by Know This:
hepatic oxidation followed by hepatic glucuroni-
dation. For patients with liver damage, alterna- For the board examination, you may be asked whether
tive benzodiazepines that do not undergo hepatic a patient in alcohol withdrawal can be safely treated
oxidation include lorazepam (L), oxazepam (O), in the inpatient versus outpatient setting. The Ameri-
and temazepam (T); use the LOT mnemonic to can Society of Addiction Medicine (ASAM) patient
remember safer benzodiazepines in patients with placement criteria provide a guide to help you deter-
liver damage. mine which patients are appropriate for outpatient
36 ADDICTION PSYCHIATRIC MEDICINE: A COMPREHENSIVE BOARD REVIEW
care (see Chapter 1). In general, inpatient treatment Important screening instruments for alcohol use
is preferred for patients who: include the National Institute on Alcohol Abuse
Present in more severe withdrawal (CIWA >15) and Alcoholism (NIAAA) single-question alcohol
A history of severe or complicated withdrawal screen Alcohol Use Disorders Identification Test
(history of seizures or delirium tremens) (AUDIT), The Alcohol Use Disorders Identification
Have high levels of recent alcohol consumption Test-Concise (AUDIT-C), (CAGE) stand for Cut,
Have severe co-occurring medical or psychiatric Annoyed, Guilty, and Eye questionnaire, Michigan
illness that may complicate treatment Alcohol Screening Test (MAST), Michigan Alcohol-
Lack an adequate support network ism Screening Test- Geriatric Version (MAST-G),
Patients who have only mild or moderate with- TWEAK is an acronym for Tolerance (T1 number
drawal, no significant medical or psychiatric illness, of drinks to feel high; T2, number of drinks one can
can come to the clinic daily, and have social support hold), Worry about drinking, Eye-opener (morning
to assist through the detoxification process are good drinking), Amnesia (blackouts), and Cut down on
candidates for outpatient detoxification. drinking (K/C) and Screening, Brief Intervention,
and Referral to Treatment (SBIRT), all of which have
been covered in Chapters 1 and 2.
You will frequently encounter the DSM-IV defini-
AT-RISK DRINKING AND AUD tions of alcohol dependence and alcohol abuse. In the
DSM-5, these distinct diagnoses were replaced by the
Screening for At-Risk Alcohol Use
single diagnosis of AUD. The DSM-5 also removed
There are a number of screening tests designed to the DSM-IV criterion of “legal problems” and added
identify unhealthy alcohol use or AUD. When patients the criterion of “cravings.” According to the DSM-5,
are identified as having harmful or unhealthy alcohol the diagnosis of AUD is established based on the diag-
use, they should be assessed for an AUD and referred nostic criteria listed in Chapter 1.
to treatment if warranted.
Some important terminology:
AUD Typologies
■ Standard drink: Definition varies by country; in A number of early research studies have led to the idea
the United States, one standard drink contains that there are typologies of individuals with alcohol
approximately 12 to 14 g of ethanol and is equiv- dependence, and these typologies are frequently ref-
alent to 12 oz beer, 5 oz wine, and 1.5 oz 80-proof erenced in clinical practice. The most well-known of
liquor. these are the Cloninger type I vs. II and Babor type A
■ At-risk drinking (unhealthy alcohol use): vs. B. These typologies are summarized in Table 4.2.
According to the National Institute on Alcohol
Abuse and Alcoholism (NIAAA), unhealthy alco-
hol use is defined for women as more than seven EPIDEMIOLOGY OF ALCOHOL USE
standard drinks per week and more than three
In terms of the overall epidemiology of alcohol use,
standard drinks per day. For men, it is defined as
some key facts to remember are:
more than 14 standard drinks per week and more
than 4 standard drinks per day. Drinking above ■ Alcohol use, heavy alcohol consumption, and
these levels increases the likelihood of social and lifetime AUD among adults 18 and older have
health consequences. increased in the United States over the past 2
■ Binge drinking: Pattern of drinking that brings decades.
BAC to 0.08 mg/dL; this is approximately five or ■ Alcohol-related deaths in the United States have
more drinks for men and four or more drinks for doubled from 1999 to 2017; in 2017, alcohol-
women consumed on one occasion. related mortalities accounted for nearly 3% of
■ Heavy alcohol use: Defined as binge drinking on all mortalities predominantly from liver disease
5 or more days during the past 30 days. (31%) and followed by overdoses (18%).
4 Alcohol Use Disorder 37
TA B L E 4 . 2
Typologies of Alcohol Dependence
Cloninger Typologies of Alcohol Dependence
Type I Type II
Genetics and environment both contribute to risk Predominantly genetic; environment has limited contribution
Late onset (after age 25 years) Early onset (before age 25 years)
Affect men and women equally Predominantly men
Ability to abstain temporarily Inability to abstain temporarily
Harm avoidance high Harm avoidance low
Drinking to self-medicate (e.g., to relieve anxiety) History of antisocial behaviors (e.g., drinking followed by aggressive behaviors)
Typically respond better to treatment Drinking to induce euphoria
Babor Typologies of Alcohol Dependence
Type A Type B
Late onset Early onset
Fewer childhood risk factors Greater childhood risk factors
Less severe dependence Family history of alcohol use
Fewer problems associated with alcohol More severe dependence
Less psychopathology More serious consequences of alcohol use
Greater psychopathology
More frequent polysubstance use
More chronic treatment history
■ Prevalence of lifetime DSM-IV alcohol abuse ■ Sex: Males are approximately twice as likely to
and/or dependence among those 18 and older have an AUD compared with females. During
was 30% between 2001 and 2002 and 44% pregnancy, AUD is less prevalent than in nonpreg-
between 2012 and 2013. nant females, and alcohol use has been declining
■ In 2018, 25% of individuals aged 12 years and in pregnant adult females over the past 2 decades.
over were current binge alcohol users, and 6.1% ■ Race: Lifetime prevalence of AUD varies by race/
were heavy alcohol users. However, among ado- ethnicity. From highest to lowest: Native Ameri-
lescents aged 12 to 17 years, binge alcohol use has can (43%), White (33%), Hispanic (23%), Black
been declining over the past 2 decades. (22%), and Asian/Pacific Islander (15%) accord-
■ The financial burden of excessive alcohol use in ing to data collected between 2012 and 2013.
the United States (nearly $250 billion in 2010) ■ Low income is also a risk factor for having an AUD.
is predominantly from drinking-related crimes
and binge drinking.
■ Keep in mind that more than 50% of individu- RISK FACTORS FOR AUD
als will remit from AUD without any treatment, This section will cover the key genetic, psychopatho-
depending on the severity of the problem. logical, and personality factors that increase the risk
for developing an AUD.
In terms of other demographic risk factors:
■ Age: Adults aged 18 to 25 years old have the Genetics of AUD
highest prevalence of past year and lifetime AUD The heritability of AUD is estimated to be approxi-
than other age groups (National Epidemiologic mately 50% based on twin and adoption studies, indi-
Survey on Alcohol and Related Conditions-III cating that positive family history is a strong risk factor
[NESARC-III] 2012–2013). for developing AUD. Genetic factors may explain some
38 ADDICTION PSYCHIATRIC MEDICINE: A COMPREHENSIVE BOARD REVIEW
diagnostic criteria: (1) two or more characteristic ■ Risk of motor vehicle accidents increases by a
facial features (short palpebral fissures, thin ver- factor of 5 with a BAC greater than 0.08 (the legal
million border, or smooth philtrum), (2) growth limit in most U.S. states), and by a factor of 27 if
retardation, (3) evidence of brain involvement, younger than 21 years of age with a BAC greater
and (4) neurobehavioral impairments. A docu- than 0.08.
mented alcohol exposure in utero is not required ■ Alcohol is estimated to be involved in over 50%
for the diagnosis. of homicides and violent assaults, as well as 25%
of sexual assaults.
■ Partial fetal alcohol syndrome (pFAS): Individu-
■ Nearly 50% of patients admitted to trauma cen-
als with pFAS do not meet full criteria for FAS
ters have a positive BAC.
but have some or most of its features. If in utero ■ Individuals with AUD have a higher odd of suicidal
alcohol exposure is documented, the diagnosis
ideation (odds ratio [OR] = 1.9), suicide attempts
requires the presence of two or more characteristic
(OR = 3.1), and completed suicide (OR = 2.6).
facial features and neurobehavioral impairments;
if no such exposure is documented, the diagnosis
Know This:
requires the presence of two or more characteris-
tic facial features, neurobehavioral deficits, growth The U.S. Federal Aviation Administration has estab-
retardation, or evidence of brain involvement. lished a BAC offense level of 0.04 mg/dL or more
■ Alcohol-related neurodevelopmental disorder owning to evidence that even a low BAC can impair
(ARND): Diagnosis requires neurobehavioral flying ability.
impairments with documented prenatal alcohol
exposure; diagnosis cannot be made in children
younger than 3 years of age.
■ Alcohol-related birth defects (ARBD): Requires TREATMENTS FOR AUD
alcohol exposure documentation with at least Among persons with AUD, fewer than one-third
one major malformation associated with alcohol receive any treatment, and less than 10% receive any
exposure. medication. Here we review pharmacologic and psy-
■ Neurobehavioral disorder associated with pre- chosocial treatments for AUD, as well as research stud-
natal alcohol exposure (ND-PAE): ND-PAE is ies that you should know for the board examination.
listed in the appendix of the DSM-5 under “con-
ditions for further study.” Requires documenta- Pharmacotherapy
tion of alcohol exposure with neurobehavioral Currently available pharmacologic interventions for
impairments and onset in childhood. Facial fea- AUD typically function by producing aversive effects
tures, growth retardation, and evidence of brain when alcohol is consumed or by reducing the reinforc-
involvement are not necessary for the diagnosis. ing effects of alcohol through actions on the reward
system. The four available U.S. Food and Drug Admin-
Know This: istration (FDA)-approved medications are naltrexone
The diagnoses of FAS and pFAS do not require docu- (oral and long-acting intramuscular [IM] formula-
mentation of prenatal alcohol exposure. tions), acamprosate, and disulfiram.
1. Naltrexone (oral):
■ Antagonist at μ- and κ-opioid receptors, and
Accidents, Trauma, Suicides, and Homicides to a lesser extent at δ-opioid receptors.
Alcohol is associated with an increased risk of acci- ■ Blocks opioid pathways projecting to the ven-
dents, traumatic injuries, suicides, violent deaths, tral tegmental area and nucleus accumbens,
and interpersonal violence. Globally, 29% of alcohol- reducing alcohol-induced dopamine release
attributable deaths were caused by injuries such as traf- and thereby reducing the positive reinforcing
fic accidents, self-harm, or violence. effects of alcohol.
4 Alcohol Use Disorder 41
■ Reduces cravings, number of drinks per day, therefore, safe in patients with liver disease
heavy alcohol use days, and rate of relapse to but should be avoided in patients who have
heavy drinking. renal impairment.
■ Can be started while the patient is still drinking. ■ Because of low bioavailability, acamprosate
■ Undergoes extensive first-pass metabolism, is has to be taken three times daily, which may
hepatically metabolized through a non-CYP negatively affect medication adherence.
dehydrogenase and conjugated with glucuronide. ■ Its side effects include diarrhea, nausea, anxi-
Naltrexone and its metabolites are excreted in ety, depression, and suicidal thinking. Its use
urine and should be used with caution in patients is contraindicated in patients with a creatinine
with renal impairment. clearance (CrCl) less than 30 mL/min.
■ Side effects include headache, anorexia, diar- 4. Disulfiram:
rhea, and nausea. More serious side effects ■ Produces a sensitivity to alcohol by irrevers-
include acute hepatitis, eosinophilic pneumo- ibly inhibiting ALDH, resulting in the build-
nia, depression, and suicidality. Contraindi- up of acetaldehyde and leading to a highly
cations include acute hepatitis, liver failure, unpleasant disulfiram–alcohol reaction.
physiological dependence on opioids, current ■ The disulfiram–alcohol reaction can begin
opioid use, or anticipated need for opioids. minutes following alcohol ingestion and last
■ Once-daily dosing can present issues for for several hours or until there is no more
patient adherence. alcohol in the body. Symptoms include cuta-
2. Naltrexone (IM): neous flushing, head throbbing, tachycardia,
■ For administration in gluteal muscles only. palpitations, nausea, vomiting, and respira-
■ Administered once every 4 weeks for tory difficulties. Severe reactions have been
improved adherence. reported including death from cardiorespira-
■ In addition to the side effects of oral naltrex- tory failure.
one, side effects unique to IM route of naltrex- ■ Patients must be warned against consuming
one include injection site reactions. alcohol in any form (e.g., in sauces, vinegars,
cough syrups, or aftershave), and informed
that a disulfiram–alcohol reaction is possible
Know This: for up to 14 days after ingesting disulfiram.
Polymorphisms of the OPRM1 (μ-opioid receptor) The half-life of disulfiram is variable (∼60–120
gene have been found to moderate naltrexone re- hours) and may lead to disulfiram–alcohol
sponse in some but not all studies. reactions for as long as several weeks after
ingesting disulfiram.
■ Disulfiram should only be started at least 12 hours
ity and increase GABAergic activity during headache and a metallic or garlic taste. More
early abstinence and can counteract the symp- rare side effects include peripheral neuropa-
toms of protracted withdrawal experienced in thy, optic neuritis, cholestatic or fulminant
early abstinence. hepatitis, rash, and psychosis. Owning to risk
■ Acamprosate is most effective when initiated of drug–induced hepatic toxicity, baseline
in patients who have established abstinence. and follow-up liver function tests are recom-
■ Acamprosate reduces rate of relapse to drinking. mended for patients on disulfiram. The rate
■ It is not metabolized by the body and is of disulfiram–induced fatal hepatitis is 1 in
excreted unchanged by the kidneys. It is, 30,000 patients.
42 ADDICTION PSYCHIATRIC MEDICINE: A COMPREHENSIVE BOARD REVIEW
■ Contraindications include severe myocardial number of heavy drinking days. It has low bio-
disease or coronary occlusion, psychosis, or availability and requires 3 times per day dosing.
patients who are receiving alcohol or alcohol- ■ Ondansetron: A 5-HT3 receptor antagonist that
containing products. Relative contraindications also regulates release of dopamine. Ondanse-
include hepatic cirrhosis or liver impairment. tron has been found to reduce drinking days and
■ Medication compliance is frequently an issue drinks per day in patients with early-onset AUD.
with disulfiram, and administration is opti- ■ Sertraline: Selective serotonin reuptake inhibi-
mally supervised (e.g., by a family member) tors have overall shown little efficacy for treatment
in patients who want to maintain abstinence. of AUD except for patients with a concomitant
depressive disorder. In one study, sertraline was
Know This: found to be helpful in patients with Babor type A
Disulfiram also inhibits dopamine-B-hydroxylase, (lower risk/lower severity), but not type B AUD.
the enzyme that metabolizes dopamine, leading to In fact, sertraline appeared to worsen alcohol use
higher basal levels of dopamine in the CNS. For this among patients with type B AUD.
reason, disulfiram is associated with neuropsychiat-
ric side effects, including psychosis. Know This:
For pregnant patients with AUD, pharmacologic
Other non-FDA-approved medications frequently treatments are generally not recommended unless it
used off-label for treatment of AUD include baclofen, is for treatment of acute alcohol withdrawal or a co-
topiramate, gabapentin, and ondansetron. Here we occurring condition.
summarize key aspects about these medications:
■ Baclofen: An antispasmodic and GABA-B ago-
nist with mixed evidence regarding efficacy in Psychosocial Interventions for AUD
AUD. It has limited hepatic metabolism and is The majority of patients who are treated for AUD
largely excreted renally as an unchanged mol- receive psychosocial interventions without medication
ecule. For this reason, it may be useful in patients management. The primary psychosocial modalities
with hepatic cirrhosis. for treatment of AUD are reviewed in greater depth
■ Topiramate: An antiepileptic medication with in Chapter 13 (Psychosocial Approaches to Substance
several mechanisms of action including poten- Use Disorder Management) and include the following:
tiation of currents through the GABA-A receptor ■ Alcoholics Anonymous (AA)
and inhibition of AMPA and kainate glutamate ■ Twelve-step facilitation (TSF)
receptors. By antagonizing glutamate, topiramate ■ Motivational interviewing (MI)
is thought to reduce dopamine release in the ■ Motivational enhancement therapy (MET)
nucleus accumbens. It reduces drinking days and ■ Cognitive behavioral therapy (CBT)
alcohol cravings. Common side effects include
anorexia, word-finding difficulties, metabolic Project MATCH (Matching Alcoholism Treatment
acidosis, nephrolithiasis, angle-closure glau- to Client Heterogeneity): Recall the results of Project
coma, and increased intraocular pressure. MATCH reviewed in Chapter 2 (“Matching alcoholism
■ Gabapentin: A structural analog of GABA that treatments to client heterogeneity,” 1998). It is a multisite
reduces neuronal excitability through inhibi- clinical trial comparing CBT, MET, and TSF for treatment
tion of alpha-2-delta-1 subunit of voltage-gated of AUD. The main findings were that no method had a
calcium channels. It does not act directly on distinct advantage. Subgroup analysis found that MET
the GABA receptor or modulate GABA activity. was superior for angry patients and those with lower initial
Gabapentin is not metabolized in the liver and motivation, CBT was superior for patients with less severe
is renally excreted. It has demonstrated efficacy AUD, and TSF was superior for patients with little social
for increasing rates of abstinence and reducing support for abstinence and those with more severe AUD.
4 Alcohol Use Disorder 43
Combined Pharmacotherapies and Behavioral A., LoCastro, J. S., Longabaugh, R., Mason, B. J., Mattson, M. E.,
Interventions (COMBINE) study: This is a multi- Miller, W. R., Pettinati, H. M., Randall, C. L., Swift, R., Weiss, R.
D., Williams, L. D., & Zweben, A. COMBINE Study Research
center, randomized placebo-controlled trial comparing Group. (2006). Combined pharmacotherapies and behavioral
different combinations of three interventions for AUD interventions for alcohol dependence: the COMBINE study: a
(Anton et al., 2016). Two of the interventions were randomized controlled trial. JAMA, 295(17), 2003–2017.
medication management (MM) with naltrexone and/ Matching alcoholism treatments to client heterogeneity. (1998).
or acamprosate, and the third was a combined behav- Project MATCH three-year drinking outcomes. Alcoholism,
Clinical and Experimental Research, 22(6), 1300–1311.
ioral intervention (CBI). Patients received 16 weeks of Sullivan, J. T., Sykora, K., Schneiderman, J., Naranjo, C. A., & Sell-
treatment and were followed for 1 year; the primary out- ers, E. M. (1989). Assessment of alcohol withdrawal: the revised
comes were percent of days abstinent and time to first clinical institute withdrawal assessment for alcohol scale (CIWA-
heavy drinking day. Patients were randomized to one of Ar). British Journal of Addiction, 84(11), 1353–1357.
nine groups evaluating different combinations of MM
and CBI. Drinking was reduced in all groups. Acampro- SUGGESTIONS FOR FURTHER READING
sate was not associated with reduced drinking compared Grant, B. F., Goldstein, R. B., Saha, T. D., Chou, S. P., Jung, J.,
with placebo when given alone or in combination with Zhang, H., Pickering, R. P., Ruan, W. J., Smith, S. M., Huang,
naltrexone and/or CBI. The group performing most B., & Hasin, D. S. (2015). Epidemiology of DSM-5 alcohol use
poorly received CBI without MM (including placebo). disorder: results from the National Epidemiologic Survey on
The best treatment outcomes were observed among alcohol and related conditions III. JAMA Psychiatry, 72(8),
757–766.
patients receiving MM with naltrexone with or without Reus, V. I., Fochtmann, L. J., Bukstein, O., Eyler, A. E., Hilty, D. M.,
CBI, and among patients receiving CBI and MM with Horvitz-Lennon, M., Mahoney, J., Pasic, J., Weaver, M., Wills,
placebo. At 1-year follow-up, there were no discernible C. D., McIntyre, J., Kidd, J., Yager, J., & Hong, S. H. (2018).
differences between any of the treatment groups. The American Psychiatric Association practice guideline for
the pharmacological treatment of patients with alcohol use
disorder. American Journal of Psychiatry, 175(1), 86–90.
REFERENCES Woodward, J. J. (2019). The ASAM principles of addiction medicine.
Anton, R. F., O’Malley, S. S., Ciraulo, D. A., Cisler, R. A., Couper, D., In S. C. Miller, D. A. Fiellin, R. N. Rosenthal, & R. Saitz (Eds.),
Donovan, D. M., Gastfriend, D. R., Hosking, J. D., Johnson, B. Pharmacology of Alcohol (6th ed., pp. 107–124). Wolters Kluwer.
REVIEW QUESTIONS
1. You are seeing a new patient for intake. She is a C. Her alcohol use is not considered at-risk
32-year-old woman with no significant medical because she has no medical or functional
history. You ask her about her alcohol use. She sequelae of her current alcohol use.
tells you that each week she consumes about 10 D. Her alcohol use is not considered at-risk
standard drinks and that at most she will have because she is not exceeding the weekly
three drinks at a time, typically on weekends accepted amount for women.
when she is out with friends. She has no medi-
cal or functional sequelae of alcohol use. Based 2. A 50-year-old man with well-controlled hyper-
on the NIAAA guidelines, you tell her that: tension and type 2 diabetes presents to your
A. Her alcohol use is considered at-risk outpatient clinic for alcohol detoxification. He
because she is drinking more than the ac- reports he has been drinking for the past 2 years
cepted weekly amount for women. and currently drinks a 6-pack of beer daily. He
B. Her alcohol use is considered at-risk drink- has a history of mild alcohol withdrawal symp-
ing because she is binge-drinking, although toms but denies past alcohol-related seizures or
her weekly amount is within the accepted DTs. He lives alone in a single-room occupancy
range for women. and has no family in the area. He has a heart rate
44 ADDICTION PSYCHIATRIC MEDICINE: A COMPREHENSIVE BOARD REVIEW
A. Production of calcium oxalate crystals, nausea, and discomfort when she consumes even
leading to renal injury small amounts of alcohol. Her reaction is likely
B. Production of formic acid, leading to dam- due to a genetic variant in the enzyme ________
age of the optic nerves that metabolizes __________ more slowly.
C. Production of acetaldehyde, leading to a A. ALDH; acetate
severe flushing B. ADH; acetate
D. Production of acetone, leading to ketosis~
C. ALDH; acetaldehyde
D. ADH; acetaldehyde
8. Which of the following biomarkers for alcohol
consumption is a direct measure of alcohol 12. The majority of individuals with AUD exhibit
(i.e., alcohol or one of its metabolites)? cognitive deficits with neuropsychological test-
A. Phosphatidylethanol (PEth) ing. Which of the following cognitive domains
B. Gamma-glutamyltransferase (GGT) show the earliest reversal upon establishment
C. Carbohydrate-deficient transferrin (CDT) of abstinence?
D. Ethyl glucuronide (EtG) A. Short-term memory
E. Aspartate aminotransferase (AST) B. Abstraction
C. Problem solving
9. On average, women have a higher BAC for D. Verbal learning
the equivalent amount of alcohol consump- E. Visuospatial abilities
tion compared with age and weight-matched
men. Which of the following best explains this 13. What is the most common cause of alcohol-
phenomenon? related mortality in the United States?
A. Women have a lower expression of ADH A. Suicide
and ALDH in the liver, reducing the rate of B. Cardiovascular disease
alcohol metabolism. C. Motor vehicle traffic injuries
B. Women have a lower expression of ADH D. Liver disease
in the gastric mucosa, reducing first-pass E. Malignant neoplasms
metabolism. 14. Which of the following medications for AUD
C. Women have a higher expression of an predominantly works by attenuating the posi-
ALDH isotype that metabolizes alcohol tively reinforcing effects of alcohol?
more slowly. A. Disulfiram
D. Women have a higher expression of an B. Naltrexone
ADH isotype that metabolizes alcohol more C. Acamprosate
quickly. D. Sertraline
10. The CIWA-Ar is a broadly used and well-vali- E. Gabapentin
dated scale measuring the severity of alcohol 15. Which of the following medications for AUD is
withdrawal. Which of the following sign or thought to work by reducing symptoms of pro-
symptom is not included among its 10 scored tracted withdrawal and thereby risk for relapse
items? in early abstinence?
A. Agitation A. Disulfiram
B. Elevated heart rate B. Naltrexone
C. Tremor C. Acamprosate
D. Auditory disturbances D. Sertraline
E. Clouding of sensorium E. Gabapentin
11. Elena is a 25-year-old woman of East Asian de- 16. Which of the following is true regarding the
scent who reports experiencing facial redness, mechanism of action of acamprosate?
46 ADDICTION PSYCHIATRIC MEDICINE: A COMPREHENSIVE BOARD REVIEW
B. It is a GABA-B agonist.
19. Which of the following are absolute contrain-
C. It is a positive allosteric modulator of gluta-
dications for using disulfiram for treatment of
matergic NMDA receptors.
AUD?
D. It inhibits the alpha-2-delta-1 subunit of
A. Migraine headaches
voltage-gated calcium channels.
B. Hepatic impairment
17. Gabapentin is frequently used as a second-line C. Coronary occlusion
agent for the treatment of AUD. Which of the D. Physiological dependence on opioids
following best describes its relevant mechanism E. Severe depression~
of action?
A. GABA-A receptor agonist 20. There are several personality factors that are
B. GABA-B receptor agonist associated with a higher risk of AUD. These in-
C. NMDA receptor antagonist clude:
D. Alpha-2-delta-1 subunit of voltage-gated A. High conscientiousness
calcium channels antagonist B. High neuroticism
E. Alpha-2 receptor agonist C. High extraversion
D. Low extraversion
18. A 10-year-old boy in foster care is brought to E. Low openness to experience
your clinic because of poor attention, hyperac- F. Low agreeableness
tivity, learning problems, and frequent fighting
~
in school. His foster parents confirm that his bi- 21. A 46-year-old woman with a long history of an
ological mother used alcohol heavily through- AUD, hepatitis C, and cirrhosis is referred to
out pregnancy. On examination, he has no your clinic for treatment. She expresses a desire
discernible facial dysmorphisms characteristic to discontinue her alcohol consumption. In ad-
of prenatal alcohol exposure, and he is in the dition to psychotherapeutic interventions, which
50th percentile for height, weight, and head cir- of the following pharmacological agents is the
cumference. Neurologic examination and brain medication of choice for treating this patient’s
imaging are normal. On neuropsychological condition?
testing, he has an IQ of 85 and clear deficits in A. Disulfiram
attention and executive functioning. Which of B. Gabapentin
the following is the most accurate diagnosis? C. Ondansetron
A. FAS D. Acamprosate
B. pFAS E. Naltrexone
5 SEDATIVES AND HYPNOTICS
47
48 ADDICTION PSYCHIATRIC MEDICINE: A COMPREHENSIVE BOARD REVIEW
■ Z-drugs, a.k.a. nonbenzodiazepines (e.g., zaleplon, ■ Potency refers to the number of milligrams to
zolpidem, eszopiclone, zopiclone): Bind GABA-A obtain a given action; for example, alprazolam 1
at the same sites as benzodiazepines. Z-drugs have mg is considered equivalent to diazepam 10 mg,
greatest effect at alpha-1 subunits and weaker activ- indicating that alprazolam has a higher potency.
ity at alpha-2 and -3 subunits, lending to its potent Approximate dose equivalencies for commonly
hypnotic effects and lesser anxiolytic effects. used benzodiazepines are shown in Table 5.1.
■ Gamma-hydroxy-butyrate (GHB): GABA-B ■ Elimination half-life: The half-lives of com-
receptor agonist used for narcolepsy. monly used benzodiazepines are also shown in
■ Baclofen: GABA-B receptor agonist used for Table 5.1. Benzodiazepines with longer half-lives
muscle spasticity. can be dosed less frequently.
■ Lipophilicity: The more lipophilic a benzodiaz-
Know This: epine is, the more quickly it will both enter and
leave the CNS. High-lipophilicity benzodiazepines
Z-drugs have greatest activity at the alpha-1 recep- have a more rapid onset and include diazepam and
tors; benzodiazepines have greatest activity at the alprazolam; low-lipophilicity benzodiazepines
alpha-1 through -3 and -5 subunits. Because Z-drugs include lorazepam and chlordiazepoxide.
primarily bind to the alpha-1 subunit of the GABA-A
receptor, they have limited anxiolytic and anticonvul- Knowing just the elimination half-life of a benzodi-
sant activity. azepine is not a good predictor of how quickly or how
Z-drugs are cross-tolerant for alcohol but less so long a patient will feel the medication’s effect: you must
than benzodiazepines and barbiturates. also consider its lipophilicity and potency. For example,
although the half-life of alprazolam is 11 to 15 hours,
patients typically experience the acute anxiolytic effects
Pharmacokinetics of alprazolam for approximately 2 to 4 hours; this is
The pharmacokinetic effects of the various sedative/ because alprazolam is highly lipophilic and both enters
hypnotics can help to predict their effectiveness, dura-
tion of action, and differential liabilities for addiction.
In general, medications with faster onsets of action are TA B L E 5 .1
considered more addictive. Commonly Used Benzodiazepines,
Approximate Half-Lives and Equipotent
Onset and Duration of Action Dosages
Dose
The structure of prototypical sedative/hypnotics is Elimination Active Equivalency
shown in Fig. 5.1. To predict the actions of benzodiaz- Benzodiazepines Half-Life (hr) Metabolite (mg)a
epines, you should know the relative potency, lipophi- Triazolam 2–5 Inactive 0.5
licity, and elimination half-life.
Lorazepam 10–14 Inactive 1
Temazepam 8–15 Inactive 10
Alprazolam 11–15 Inactive 0.5
Chlordiazepoxide 5–30 Activeb 10
Clonazepam 18–50 Inactive 0.25–0.5
Oxazepam 5–15 Inactive 15–30
Diazepam 50–100 Activeb 5
aThese doses are approximate equipotencies and are not
Fig. 5.1 ■ Structure of prototypical sedative/hypnotics. Ben-
recommended for initiation or for conversion between
zodiazepines are composed of a benzene ring fused to a seven- medications.
membered diazepine ring; modification of the R-groups can bThe active metabolites of diazepam are oxazepam, temazepam,
change the compound’s characteristics. Barbituric acid is the and desmethyldiazepam; those of chlordiazepoxide are oxazepam
parent compound for all barbiturates. Zolpidem is shown as a and desmethyldiazepam. The presence of these active metabolites
prototypical nonbenzodiazepine or Z-drug. contributes to these compounds’ longer half-lives.
5 Sedatives and Hypnotics 49
and leaves the CNS rapidly. The same principles regard- Lorazepam and diazepam are prepared in solutions
ing lipophilicity and elimination half-life apply to other with propylene glycol when administered intrave-
sedatives/hypnotics. Concerning barbiturates, the nously. There have been cases of iatrogenic propylene
compound with the fastest onset of action is sodium glycol toxicity with repeated administration of intrave-
thiopental, which is highly lipophilic and used for anes- nous benzodiazepines.
thesia. It has an onset of action of less than 1 minute
and a duration of action between 5 and 10 minutes. TOXICOLOGY TESTING
Metabolism Toxicology testing for benzodiazepines is covered more
■ Benzodiazepines are generally metabolized extensively in Chapter 12. Recall that the standard tox-
by CYP3A4 oxidation followed by glucuro- icology test for benzodiazepines detects the presence
nide conjugation. The exceptions to this are the of compounds that are metabolized to oxazepam—this
“LOT” (lorazepam, oxazepam, and temazepam) includes diazepam, chlordiazepoxide, clorazepate, and
benzodiazepines, which undergo only gluc- temazepam. Several of the very commonly prescribed
uronidation and have no active metabolites. For benzodiazepines (clonazepam, alprazolam, and loraz-
this reason, they are preferred in patients with epam) have a different metabolic pathway and are fre-
hepatic impairment or to avoid drug–drug inter- quently missed by standard toxicology tests.
actions. Many of the common benzodiazepines
are metabolized to oxazepam (diazepam, clo-
SEDATIVE/HYPNOTIC MEDICAL
razepate, chlordiazepoxide, and temazepam).
Alprazolam, lorazepam, and clonazepam do not INDICATIONS AND ADVERSE EFFECTS
share this metabolic pathway. Sedative/Hypnotic Medical Indications
■ Barbiturates are generally metabolized by ■ Benzodiazepines: FDA-approved indications
CYP3A4, 3A5, and 3A7; they also induce CYP2D6, include panic disorder, generalized anxiety dis-
2C9, and 3A4 and can, therefore, decrease serum order, social phobia, insomnia, and status epilep-
levels of medications metabolized by these enzymes. ticus/seizures; non-FDA-approved uses include
■ Zolpidem and zopiclone are metabolized by agitation, alcohol withdrawal, muscle spasms,
CYP3A4; zaleplon is metabolized by alde- restless leg syndrome, and presurgical medica-
hyde oxidase. There are significant sex differ- tion. Benzodiazepines are classified by the Drug
ences in the metabolism of zolpidem. In 2003, Enforcement Agency (DEA) as Schedule IV con-
the U.S. Food and Drug Administration (FDA) trolled Substances.
required lower recommended doses of zolpi- ■ Barbiturates: Previously widely used for sleep
dem for women to reduce the risk of next-day and anxiety but largely replaced now by benzo-
activities that require alertness, such as driving. diazepines. They are still used as anticonvulsants,
Men metabolize the standard 10-mg formula- in general anesthesia, and physician-assisted sui-
tion approximately twice as quickly as women; cide. The majority of barbiturates are classified by
females have also been found to have higher peak the DEA as Schedule III or IV.
concentration of zolpidem than men. ■ Baclofen: FDA-approved for spasticity, and off-
label use for alcohol use disorder. Baclofen is not
Know This: classified as a controlled substance.
Most benzodiazepines are metabolized by CYP3A4 ■ GHB: FDA-approved for narcolepsy as sodium
enzymes. Any medication that induces or inhibits oxybate, the sodium salt of GHB. Currently
CYP3A4 (e.g., ketoconazole, macrolides, oral contra- classified by the DEA as Schedule III when pre-
ceptives) may, therefore, affect drug levels. Exceptions scribed as sodium oxybate, and Schedule I for
to this are lorazepam, oxazepam, and temazepam, any other use.
which do not undergo metabolism by CYP enzymes. ■ Z-drugs: FDA-approved for short-term treatment
of insomnia. Classified by the DEA as Schedule IV.
50 ADDICTION PSYCHIATRIC MEDICINE: A COMPREHENSIVE BOARD REVIEW
Barbiturate toxicity is far less common today than it vomiting, ataxia, and death. GHB has a steep dose-
was during the 1960s and 1970s when they were more response curve, and the LD50 is approximately
commonly prescribed. Barbiturates have largely been five times the typical recreational dose. Treatment
replaced by benzodiazepines due to their dangerous- includes ABCs, atropine for bradycardia, and
ness in overdose and narrow therapeutic window. One potential intensive care unit admission.
unique complication of barbiturate-induced intoxica- ■ Flunitrazepam (brand name Rohypnol): a fast-
tion is bullous lesions, frequently seen on the hands. acting benzodiazepine that has also been used to
The differential diagnosis of patients present- facilitate date rape. It is no longer available in the
ing with suspected sedative/hypnotic intoxication is United States, the United Kingdom, or Canada. It
broad. Many of the sedative/hypnotics share similar responds to flumazenil in overdose.
features in overdose. If patients present with altered
mental status, other life-threatening medical etiolo- Know This:
gies should be ruled out such as hypoglycemia, car-
Flumazenil competitively antagonizes the GABA-A
bon monoxide exposure, stroke, encephalitis, or head
receptor at the benzodiazepine binding site. Given
trauma. Remember that benzodiazepine overdose
its mechanism of action, it can reverse both benzodi-
in isolation rarely causes life-threatening respiratory
azepine and Z-drug overdose but not barbiturate or
depression or coma, so evaluation for co-ingestion of
alcohol poisoning.
substances should always be performed.
■ Nonnative cultural origin use disorders. In patients presenting with sedative mis-
■ Longer duration of benzodiazepine use use, it is especially important to evaluate for other psy-
■ Higher dose of benzodiazepine chiatric conditions. Common co-occurring conditions
■ Psychiatric illness include panic disorder, mood disorders, and personal-
■ Other substance use disorders ity disorders. In patients with a current substance use
disorder and a psychiatric illness, it is preferable not
As mentioned previously, approximately 10% of
to initiate treatment with a benzodiazepine given their
individuals who use benzodiazepines once will develop
higher risk of developing a use disorder.
a substance use disorder. However, the majority do not
To choose an appropriate alternative agent to a ben-
find them reinforcing and do not develop problematic
zodiazepine, one should perform a thorough psychiat-
use. Benzodiazepine abuse is rarely the sole or primary
ric evaluation, determine and treat the patient’s specific
substance of abuse; most benzodiazepine abuse is along
subtype of anxiety disorder or other psychiatric condi-
with other substances, most commonly opioids. One
tion. Patients should be educated about the onset of
study conducted by NIDA found that 73% of heroin users
action of antidepressants or other medications that are
use benzodiazepines more often than weekly. When
utilized and that effects will not be felt as immediately
used illicitly, benzodiazepines are reported to “boost” the
as with benzodiazepines. Therapy should be integrated
effects of opioids, alleviate withdrawal syndromes from
into the treatment plan as needed.
other substances, and manage stimulant highs.
Some key points:
Signs of a burgeoning use disorder include rapid
development of tolerance with frequent requests for ■ SSRIs and tricyclic antidepressants work about
dose increases, requests for early refills, using a higher as well for generalized anxiety disorder as benzo-
dose and using more often than prescribed, escalating diazepines; buspirone also has proven efficacy in
use despite negative consequences, using medication multiple controlled trials.
for euphoric high, using medication when feeling upset, ■ Alpha- and beta-adrenergic agents (such as
preoccupation with medication, reported inability to clonidine and beta-blockers) and second-gen-
function without it, and use of other drugs or unhealthy eration antipsychotics (risperidone, quetiapine,
use of alcohol. olanzapine) have reported efficacy in treatment
of acute anxiety in patients in which one wishes
Know This: to avoid benzodiazepines.
■ There are many nonbenzodiazepine agents that
You may be asked about “pseudoaddiction” on the can be used for treatment of insomnia including
board examination, which is the notion that patients trazodone, amitriptyline, doxepin, and ramelteon.
treated with inadequate doses may repeatedly re- Treatment of insomnia should also include psy-
quest increases in dosages; these patients may in- choeducation around sleep hygiene.
correctly be seen as drug-seekers. Pseudoaddiction
has most widely been discussed in pain management
and opioid medications but applies to benzodiaz- REFERENCES
epines as well. In the context of benzodiazepines and Dodds, T. J. (2017). Prescribed benzodiazepines and suicide risk:
treatment of anxiety, these patients’ approaches in- a review of the literature. The Primary Care Companion for
clude increasing the dose or frequency of dosing or CNS Disorders, 19(2), 16r02037. https://doi.org/10.4088/PCC.
considering a longer-acting agent. 16r02037.
Vicens, C., Fiol, F., Llobera, J., Campoamor, F., Mateu, C., Alegret,
S., & Socías, I. (2006). Withdrawal from long-term benzodiaz-
epine use: randomised trial in family practice. British Journal
Treatment of Co-Occurring Psychiatric Illness of General Practice: The Journal of the Royal College of General
Practitioners, 56(533), 958–963.
Patients with a sedative/hypnotic use disorder have
Weich, S., Pearce, H. L., Croft, P., Singh, S., Crome, I., Bashford, J., & Frisher,
high rates of psychiatric comorbidity. Approximately M. (2014). Effect of anxiolytic and hypnotic drug prescriptions on
half of patients have a psychiatric illness, which is mortality hazards: retrospective cohort study. BMJ (Clinical research
higher than the comorbidity rates for other substance ed.), 348, g1996. https://doi.org/10.1136/bmj.g1996.
54 ADDICTION PSYCHIATRIC MEDICINE: A COMPREHENSIVE BOARD REVIEW
SUGGESTIONS FOR FURTHER READING of substance abuse treatment (5th ed.). American Psychiatric As-
sociation Publishing.
Bachhuber, M. A., Hennessy, S., Cunningham, C. O., & Starrels, J. L.
Olfson, M., King, M., & Schoenbaum, M. (2015). Benzodiazepine
(2016). Increasing benzodiazepine prescriptions and overdose
use in the United States. JAMA Psychiatry, 72(2), 136–142.
mortality in the United States, 1996-2013. American Journal
https://doi.org/10.1001/jamapsychiatry.2014.1763.
of Public Health, 106(4), 686–688. https://doi.org/10.2105/
Soyka, M. (2017). Treatment of benzodiazepine dependence. New
AJPH.2016.303061.
England Journal of Medicine, 376(12), 1147–1157. https://doi.
Bisaga, A., & Mariani, J. J. (2015). Benzodiazepines and other seda-
org/10.1056/NEJMra1611832.
tives and hypnotics. In M. Galanter, H. D. Kleber, & K. Brady
(Eds.), The American Psychiatric Association Publishing textbook
REVIEW QUESTIONS
7. In 2019, the FDA placed a black box warn- attempted to taper the lorazepam. Which of
ing on three Z-drugs: zolpidem, zaleplon, and the following options is the next best step?
eszopiclone. The black box warning indicates A. Transition patient from lorazepam to clon-
that these Z-drugs carry a risk of: azepam and perform a slow taper
A. Mortality when combined with opioids B. Transition patient from lorazepam to alpra-
B. Complex sleep-related behaviors zolam and perform a slow taper
C. Suicidal ideation C. Start a SSRI while performing a rapid taper
D. Arrhythmia of lorazepam
E. Respiratory depression D. Start a SSRI while performing a slow taper
8. A 42-year-old woman who has been prescribed of lorazepam
alprazolam 2 mg three times daily for 5 years. E. Start a SSRI and discontinue lorazepam
Which of the following side effects of alprazol- 10. A 37-year-old man with a history of posttrau-
am is she least likely to build tolerance to? matic stress disorder is brought to the emergency
A. Anticonvulsant effects room following a suicide attempt by overdos-
B. Sedation ing on prescription alprazolam. On arrival, the
C. Memory impairment patient is obtunded and is unable to provide a
D. Falls history. Respiratory rate is slowed, and he has
E. Impaired driving an oxygen saturation of 86%. Administration of
9. A 49-year-old man who has been prescribed which of the following medications or proce-
lorazepam 0.5 mg three times daily for the past dures would be the immediate next best step?
6 years for generalized anxiety disorder. He A. Flumazenil
wants to discontinue taking lorazepam because B. Flurazepam
of concerns about long-term side effects but C. Naloxone
has experienced intolerable anxiety, irritability, D. Activated charcoal
and sleeplessness whenever his previous doctors E. Hemodialysis
6 OPIOID USE DISORDER
BRIEF HISTORY OF THE CURRENT 1995, the American Pain Society launched a campaign
OPIOID EPIDEMIC to standardize the treatment of pain symptoms and pro-
moted pain as “the fifth vital sign.” The Federation of
Beginning in the early 1800s, opium and morphine use State Medical Boards, the Drug Enforcement Agency,
became widespread; opiate-containing “patent medi- and the Joint Commission also issued statements pro-
cines” were heavily marketed for several medical condi- moting pain assessments and analgesic treatment. Phy-
tions, especially toward women. Morphine prescribing sicians were now pressured to provide aggressive pain
peaked in 1890, and in 1898, Bayer started marketing control, as hospital administrations were under pressure
heroin as a treatment for morphine addiction. In 1914, to assess and treat pain to receive federal health care
changes in domestic and international policy, spurred funds. Starting in 1996, OxyContin, an extended-release
in part by the Opium Wars in China, led to the passage oxycodone, was heavily marketed by the pharmaceuti-
of the Harrison Act (discussed in Chapter 12). This act cal industry and its use for the treatment of noncancer
banned physicians from prescribing opioids for the chronic pain skyrocketed. The United States subse-
treatment of opioid use disorder (OUD) and made any quently saw an extraordinary increase in the prescrip-
medicinal use of heroin illegal. tion and consumption of opioid medications.
The current opioid epidemic is not the first in this
nation’s history. The roots of the current opioid crisis Know This:
began in the 1990s, when prescription opioid prescrib-
ing was greatly expanded along with pharmaceutical One important feature of the current opioid epidemic
marketing of these agents as “minimally addictive.” is that many new opioid users initially used prescrip-
Opioid prescribing reached a zenith in 2010 when tion opioid medications (prescribed to oneself or to
there were an astounding 81 opioid prescriptions others) and then transitioned to the use of heroin. This
per 100 people. Other causal factors for the current transition was largely driven by reduced access and
opioid epidemic include declining economic oppor- restrictiveness around opioid medication prescribing
tunities and the rise in self-reported psychological dis- following the early rise of opioid-related deaths and
tress. Because of the alarming rates of opioid-related OUDs, and higher cost compared to heroin.
overdoses, the United States declared a public health
emergency because of the opioid epidemic in 2017.
One critical factor in the current opioid epidemic EPIDEMIOLOGY OF OPIOID USE
was the change in clinical practice toward the treat- Key facts to remember:
ment of pain (see Chapter 11). Treatment of chronic
pain became an important research focus, particularly ■ The rate of drug overdoses has increased more
pain caused by cancer. Several articles and small retro- than threefold between 1999 and 2017 from 6.1
spective studies were widely circulated in support of the per 100,000 standard population to 21.7, accord-
claim that treatment of chronic pain with opioids rarely ing to data from the Centers for Disease Control
led to the development of a substance use disorder. In and Prevention (CDC).
57
58 ADDICTION PSYCHIATRIC MEDICINE: A COMPREHENSIVE BOARD REVIEW
■ In 2017, drug overdoses were the leading cause of KEY REGULATIONS, POLICIES, AND
accidental death among people aged 15 to 64 years. GUIDELINES
■ In 2018, the prevalence of past-year illicit drug use
from prescription pain relievers in people aged 12 Narcotic Addiction Treatment Act of 1974
years and older was 3.6%; it was the second most This act legitimizes and regulates the use of methadone
commonly used drug following marijuana. Of peo- to treat OUD and states that it must be dispensed in
ple aged 12 years and older, 2.1% have used heroin opioid treatment programs (OTPs) registered with
at some point in their lives (NSDUH, 2018). the Drug Enforcement Administration, the federal
■ According to the CDC, the overdose rate from Substance Abuse and Mental Health Administration
synthetic opioids (including fentanyl and its ana- (SAMHSA), and the home state’s methadone agency.
logues) was 12 times higher in 2019 than in 2013.
Between 2018 and 2019, overdose deaths from Drug Addiction Treatment Act of 2000
synthetic opioids increased by 16%. (DATA 2000)
■ Close to half of people on chronic opioid therapy DATA 2000 permits physicians who meet the quali-
meet the criteria for an OUD. fications to treat up to 30 patients with OUD in an
■ Age: The average age of first heroin use is in the office-based setting with buprenorphine. Before enact-
early 20s, with the 25 to 34 age group experienc- ing DATA 2000, the use of opioid medications to
ing the most opioid overdose deaths in 2019. treat OUD was permissible only in federally approved
■ Sex: Even though woman are prescribed opioid treatment programs, such as a methadone clinic. The
medication more frequently than men, men have DATA 2000 requires prescribers to complete an 8-hour
1.5-fold the risk of prescription opioid misuse training course to receive an X-waiver to prescribe
and threefold the risk of heroin use. Between buprenorphine for OUD.
1999 and 2010, overdoses from prescription opi-
oid medication increased four times in women Ryan Haight Online Pharmacy Consumer
and over two times in men according to data Protection Act of 2008
from the CDC; however, men are still more likely The Drug Enforcement Administration is responsible for
to die from a prescription opioid overdose. the implementation and enforcement of this act. Under
■ Race: Among racial/ethnic groups, the highest this act, a health professional cannot prescribe a con-
opioid overdose rates are among Native Ameri- trolled substance without seeing that patient in-person.
cans and non-Hispanic Whites. Approximately This requirement for in-person visits was waived until
90% of new heroin users in the past decade are the end of 2021 because of the COVID-19 pandemic to
non-Hispanic Whites. enable continued treatment of patients with OUD.
■ Geography: Between 1999 and 2015, although the
greatest proportion of drug overdoses occurred Comprehensive Addiction and Recovery
in the metropolitan areas, the rate of increase in Act (CARA) of 2016
overdose deaths has been greatest in nonurban This act was signed into law in response to the opioid
areas (nonurban: 325%; urban: 198%). crisis. It allocates grants for the expansion of preven-
tion and treatment programs, especially evidence-
Know This: based treatments for OUD. It temporarily authorizes
nurse practitioners and physician assistants to become
The current opioid epidemic is demographically dis- trained and waivered to prescribe buprenorphine.
tinct from prior epidemics in that it has predominantly
impacted non-Hispanic Whites, a greater proportion
SUPPORT for Patients and Communities
of women, and individuals living in rural and suburban
Act of 2018 (SUPPORT Act)
areas. Previous opioid epidemics between the 1950s
and 1980s were centered in urban minority communi- The Substance Use-Disorder Prevention Promotes
ties, predominantly heroin use among young men. Opioid Recovery and Treatment for Patients and Com-
munities (SUPPORT) for Patients and Communities
6 Opioid Use Disorder 59
Act of 2018 (SUPPORT Act) expands the ability to “opiates” are frequently used interchangeably. “Opiates”
treat up to 100 patients (from 30 patients under DATA refers specifically to natural or semisynthetic opioids
2000) in the first year of waiver receipt if practitio- derived from the poppy plant, which include opium,
ners satisfy one of the following two conditions: the morphine, heroin, and codeine. “Opioids” refers to all
practitioner holds a board certification in addiction natural, synthetic, or partly synthetic molecules, such as
medicine or addiction psychiatry, and the practitioner hydrocodone, fentanyl, and methadone that act on opioid
provides medication-assisted treatment in a qualified receptors. Synthetic and semisynthetic molecules, such as
practice setting under the act. oxycodone, hydrocodone, hydromorphone, methadone,
tramadol, and fentanyl, are not found in nature.
Federal Guidelines for OTPs Opioids exert their effects through several molecular
OTPs dispense opioid agonists on-site for the treat- pathways on the level of both the peripheral and central
ment of OUD and provide a number of other medical nervous system. The family of opioid receptors consist
and psychosocial services. OTPs are expected to follow of three G protein-coupled receptors: the mu, delta, and
federal standards as outlined in Title 42 of the Code of kappa receptors. The mu-opioid receptors mediate the
Federal Regulations Part 8 (42 CFR § 8). Naltrexone, analgesic properties of opioids and their euphoric prop-
an opioid antagonist used for treatment of OUD, can erties, as well as tolerance and physical dependence.
be offered in an OTP and it is not subject to the same The role of delta-opioid receptors is linked to reducing
regulations as methadone or buprenorphine. Certifica- chronic pain, drug reward regulation, inhibitory control,
tion of these programs is overseen by SAMHSA. and learning. Kappa receptors are the most abundant
opioid receptors in the brain and have functions related
Know This: to the modulation of stress, anxiety, and emotional reac-
tions. These effects are mediated by dynorphins and not
Methadone can only be dispensed in an OTP when by enkephalins, which predominantly bind to delta- and
prescribed for the treatment of OUD. Buprenorphine the mu-opioid receptors.
can be offered in either an office-based opioid treat- Opioid intoxication causes the following physiolog-
ment clinic or an OTP; if used in an OTP, it must be ical and psychological effects:
dispensed by the clinic similar to methadone.
Physiological Psychological
Prescription Drug Monitoring Programs ■ Meiosis ■ Poor decision making
■ Head nodding ■ Lowered motivation
Prescription drug monitoring programs (PDMPs) are
electronic databases that track controlled substance ■ Sedation ■ Anxiety attacks
prescriptions in a state in real-time. Pharmacies have ■ Hypokinesis ■ Slurred speech
to report to the PDMP whenever a prescription for a ■ Hypothermia ■ Sleeping more and
controlled substance is dispensed to a patient. Some ■ Hypotension feeling tired
states have policies requiring the provider to check the ■ Mood swings
■ Bradycardia
state’s PDMP before prescribing controlled substances.
■ Abdominal pain ■ Euphoria or dysphoria
PDMPs enable state health departments to better
■ Irritability
understand physician and patient behavior and evalu-
ate interventions to reduce inappropriate prescribing
of controlled substances. Opioid use is associated with adverse effects including:
■ Respiratory depression: This is the most concerning
opioid side effect and the hallmark of opioid over-
OPIOID PHARMACOLOGY
dose. Sedation occurs before the onset of significant
In this section, we review the pharmacology of opioid respiratory depression and, therefore, is a warning
medications used to treat pain. A note on terminol- sign. It is caused by the opioid’s effects on the med-
ogy: although they are distinct, the terms “opioids” and ullary respiratory center and is marked by impaired
60 ADDICTION PSYCHIATRIC MEDICINE: A COMPREHENSIVE BOARD REVIEW
■ The initial course of treatment should be viewed patients already taking opioid medications for
as short term (<60 days) pain
■ Start low and titrate cautiously ■ Screener and Opioid Assessment for Patients
■ Do not start extended-release or long-acting for- with Pain-Revised (SOAPP-R): Used before the
mulations in opioid naïve patients initiation of long-term opioid therapy
■ Avoid daily doses of more than 90 mg morphine In the next section, we review some commonly
milligram equivalent (MME) encountered (and assessed in board examination) pre-
■ Individualize opioid selection and dosing based scribed opioids.
on the patient’s age and health status
■ Consider patients’ previous exposure to opioids Morphine
and their risks for opioid tolerance Morphine is a mu-, kappa-, and delta-opioid recep-
tor agonist, with strongest affinity for the mu-opioid
Know This:
receptor. It is glucuronidated to two metabolites with
The doses of pain medications, especially opioids, potentially significant differences in efficacy, clear-
should be lower in patients with renal failure, hepatic ance, and action: morphine-6-glucuronide (M6G)
failure, or age older than 65 years. and morphine-3-glucuronide (M3G). The M3G
metabolite of morphine lacks analgesic activity but
When prescribing opioids, the following measures has neuroexcitatory effects and may be responsible
can be taken to mitigate the associated risks: for opioid-induced hyperalgesia. The M6G metabolite
has analgesic effects but less than the parent molecule.
■ Safe storage and disposing of the prescriptions Morphine is not recommended for use in patients with
(e.g., storage in a lockbox) advanced chronic kidney disease.
6 Opioid Use Disorder 61
opioid discontinuation, cAMP signaling increases and methadone, tramadol, and buprenorphine) or alpha2-
leads to noradrenergic overactivation in the locus coe- adrenergic receptor agonists (e.g., clonidine, tizanidine,
ruleus and the classic symptoms of opioid withdrawal. and lofexidine). Adrenergic agents are effective for the
On discontinuation of opioid use, the onset of opioid treatment of opioid withdrawal symptoms by targeting
withdrawal symptoms depends on the half-life of the spe- noradrenergic hyperactivity in locus coeruleus neurons
cific opioid used. In persons using short-acting opioids, but may cause hypotension.
such as heroin, oxycodone, morphine, hydrocodone, and
fentanyl, withdrawal symptoms typically appear within 8 Know This:
to 12 hours with a peak in symptoms within 1 to 3 days
■ COWS are commonly used during buprenor-
and can persist for up to 7 days. Withdrawal symptoms
phine inductions. Patients must be experienc-
from long-acting opioids, such as methadone, typically
ing moderate opioid withdrawal before ad-
first appear 1 to 2 days following opioid discontinuation
ministration of the first buprenorphine dose,
and can persist for more than 14 days.
equivalent to a COWS score of 8 to 11.
The Clinical Opiate Withdrawal Scale (COWS) is the
■ Transcutaneous electrical acupoint stimulation
most commonly used tool for the assessment and moni-
(electrical acupuncture) has been used for opi-
toring of opioid withdrawal in inpatient or outpatient
oid detoxification because of reduced cravings
settings. It is based on the Clinical Institute Withdrawal
via endorphin release.
Assessment of Alcohol (CIWA-A) Scale used to mea-
■ Detoxification is not a treatment for OUD.
sure alcohol withdrawal. Alternatively, the Finnegan
Without subsequent maintenance treatment, it
scoring system can be used to assess opioid and nono-
carries a risk of overdose and decreased reten-
pioid withdrawal.
tion rates to care.
Opioid withdrawal syndrome is marked by the follow-
ing physiological and psychological signs and symptoms:
with doses greater than 80 mg per day are associated see Chapter 4). It is available in daily oral tablets or
with superior clinical outcomes. Methadone mainte- as a long-acting injectable. It has been associated with
nance leads to marked reductions in illicit opioid use, hepatocellular injury particularly when used in doses
including injection opioid use, fewer HIV or hepatitis C much higher than 50 mg daily, although it may still be
infections, increased employment rates, and decreased used in patients with hepatic impairments in consulta-
likelihood of criminal justice involvement. Data are tion with the patient’s gastroenterologist. Recent stud-
mixed regarding the effect of methadone maintenance ies of naltrexone have focused on pharmacogenetic
on the use of illicit drugs other than opioids. predictors of treatment response, namely the mu1-
opioid receptor (OPRM1) gene.
Know This:
Know This:
Patients receiving methadone maintenance should
receive an electrocardiogram at the onset of treat- Naltrexone produces some pupillary constriction by
ment and within 30 days because methadone causes an unknown mechanism.
QTc prolongations.
Signs and symptoms of NAS occur 48 to 72 hours opioid-agonist medication administered to treat
post-birth following discontinuation of prenatal opi- NAS in methadone-exposed neonates was three
oid exposure, and the syndrome duration depends on times greater than for buprenorphine-exposed
the half-life of the opioids used. NAS is treated using neonates. Similarly, the length of hospitalization
deodorized tincture of opium or oral morphine. Barbi- was shorter for buprenorphine-exposed than for
turates can be used as second-line treatment for severe methadone-exposed neonates.
withdrawal, including withdrawal seizures, or if maxi-
mum safe doses of morphine or deodorized tincture of
opium have already been used. REFERENCE
Women who are on treatment for OUD and on a Center for Behavioral Health Statistics and Quality. (2019). 2018
National Survey on Drug Use and Health Final Analytic File Co-
stable dose of methadone or buprenorphine should be debook. Substance Abuse and Mental Health Services Adminis-
encouraged to breastfeed unless specifically contraindi- tration, Rockville, MD.
cated. A study of neonates prenatally exposed to metha-
done or buprenorphine for a minimum of 30 days found SUGGESTIONS FOR FURTHER READING
that breastfed neonates had significantly shorter lengths American Psychiatric Association. (2013). Diagnostic and statistical
of hospital stay and need for pharmacotherapy for NAS. manual of mental disorders (5th ed., p. 541). American Psychi-
atric Association.
Know This: American Society of Addiction Medicine. (2016). Sample office-
based opioid use disorder policy and procedure manual. Ameri-
■ The Maternal Opioid Treatment: Human Ex- can Society of Addiction Medicine.
perimental Research (MOTHER) study was a Faggiano, F., Vigna-Taglianti, F., Versino, E., & Lemma, P. (2003).
Methadone maintenance at different dosages for opioid dence.
large-scale trial comparing maternal and neo- Cochrane Database Syst Rev, (3), CD002208.
natal outcomes in pregnant women with OUD Food and Drug Administration. (2017). Appropriate use checklist:
randomly assigned to methadone or buprenor- Buprenorphine-containing transmucosal products for opioid de-
phine pharmacotherapy during pregnancy. It pendence. Food and Drug Administration.
showed that OUD treatment using buprenor- Jones, H. E., Johnson, R. E., Jasinski, D. R., O’Grady, K. E., Chisholm,
C. A., Choo, R. E., Crocetti, M., Dudas, R., Harrow, C., Hues-
phine is associated with shorter treatment dura- tis, M. A., Jansson, L. M., Lantz, M., Lester, B. M., & Milio, L
tion, less medication needed to treat NAS symp- (2005). Buprenorphine versus methadone in the treatment of
toms, and shorter hospitalizations for neonates. pregnant opioid-dependent patients: effects on the neonatal ab-
The buprenorphine-exposed group of neonates stinence syndrome. Drug and Alcohol Dependence, 79(1), 1–10.
showed less motor activity suppression and a Kosten, T. R., & Baxter, L. E. (2019). Review article: Effective manage-
ment of opioid withdrawal symptoms: a gateway to opioid depen-
longer movement duration. The methadone- dence treatment. American Journal on Addictions, 28(2), 55–62.
exposed fetuses were found to have a higher in- Substance Abuse and Mental Health Services Administration.
cidence of intrauterine growth restriction than (2015). Clinical use of extended-release injectable naltrexone in
buprenorphine-exposed fetuses. They were also the treatment of opioid use disorder: A brief guide. Substance
more likely to have an abnormal nonstress test Abuse and Mental Health Services Administration. HHS Pub-
lication No. (SMA) 14-4892R.
and fewer fetal heart rate accelerations. Substance Abuse and Mental Health Services Administration.
■ The Pregnant and the Reduction of Opiates:
(2016). Pocket guide: Medication-assisted treatment of opioid use
Medication Intervention Safety and Efficacy disorder. Substance Abuse and Mental Health Services Admin-
(PROMISE) study found that the amount of istration. HHS Publication No. (SMA) 16-4892PG.
66 ADDICTION PSYCHIATRIC MEDICINE: A COMPREHENSIVE BOARD REVIEW
REVIEW QUESTIONS
67
68 ADDICTION PSYCHIATRIC MEDICINE: A COMPREHENSIVE BOARD REVIEW
Norepinephrine Arousal; poor Decreased heart smoke, at least 250 are known to be harmful, in-
appetite rate cluding hydrogen cyanide, carbon monoxide, and
Dopamine Pleasure; poor Increased appetite; ammonia. At least 69 of these can cause cancer.
appetite craving ■ Tobacco affects sleep during withdrawal but not
■ Women who smoke have an increased risk for ■ The Fagerström Test for Nicotine Use Disorders
hip fractures. is a standard instrument for assessing the in-
■ Nicotine concentration can be twice as high in tensity of physical dependence to nicotine. The
breast milk compared to blood concentration in higher the total Fagerström score, the more in-
lactating women. tense is the patient’s physical dependence on
nicotine. The results correlate with smoking
Secondhand Smoke cessation outcomes.
■ Many of the chemicals that smokers inhale ■ The 5 A’s of smoking cessation: Ask, Advise,
REVIEW QUESTIONS
1. A 41-year-old woman who smokes close to greatest efficacy in achieving abstinence in in-
15 cigarettes per day presents to your office. dividuals with tobacco use disorder?
Recently, a family member was diagnosed with A. Buspirone
lung cancer. This event has motivated her to B. Varenicline
quit smoking. She has tried several times to C. Clonidine
quit without success. Which of the following D. Bupropion
treatment options has been shown to have the E. NRT
7 Tobacco and Other Nicotine Products 71
2. Which of the following is a symptom of nicotine B. Apply the patch once a day, ideally at differ-
withdrawal? ent times each day
A. Abdominal cramps C. Wear the patch over clothing
B. Hypersomnia D. Use two patches if the patient smokes more
C. Decreased appetite than five cigarettes per day
D. Decreased heart rate E. Use the same patch for more than 24 hours
E. Tremor
5. Which of the following Nicotine Replacement
~
3. A 21-year-old woman was brought to your office Therapy option has a higher risk of being abused?
by her mother. She has been smoking close to 10 A. Inhalers
cigarettes a day. You discussed NRT to which she B. Nasal spray
agrees. What dose of nicotine patch would you C. Lozenges
prescribe to the patient? D. Gum
A. 7 mg every 12 hours E. Patch
B. 14 mg every 12 hours
6. Which one of the following medications can in-
C. 14 mg every 24 hours
crease the odds of quitting smoking?
D. 21 mg every 24 hours
A. Paroxetine
E. 2 patches of 21 mg every 12 hours
B. Duloxetine
4. Which of the following is recommended to de- C. Nortriptyline
crease side effects when using nicotine patches D. Fluoxetine
as NRT? E. Desvenlafaxine
A. Not wearing the patch during nighttime
8 CANNABIS USE DISORDER
73
74 ADDICTION MEDICINE COMPREHENSIVE REVIEW
Rimonabant is a CB1 receptor antagonist and has been The combination of cannabinoids with anticholin-
studied to treat cannabis use disorder and weight loss. ergic drugs can lead to marked tachycardia.
TA B L E 8 . 2
Cannabis Route of Administration
Route of
Administration Smoking (Joints, Pipes, Blunts) Vaporization Mucosal (Oils) Edible
Toxicity Combustion at high heat Utilizes moderate heat— No toxic byproducts or No toxic byproducts or
leads to production of toxic some production of pulmonary symptoms pulmonary symptoms
byproducts toxic byproducts
Onset of action Rapid onset of action (5 min), Rapid onset (5 min), Rapid onset of action Onset of action
peak at 30 min, short duration short duration (2–4 h) (15–30 min) much longer and very
(2–4 h) unpredictable (1–3 h);
duration of action 6–8 h
Side effects Pulmonary symptoms Pulmonary symptoms No pulmonary No pulmonary symptoms
Cough E-cigarette or vaping symptoms
Bronchitis product use associated
lung injury (EVALI)
Frequently mixed with tobacco
8 Cannabis Use Disorder 77
burden of carbon monoxide and tar than tobacco. This In terms of other demographic risk factors:
effect is independent of the concentration of THC in ■ Age: The highest prevalence of past-year cannabis
the cannabis plant.
use by age is among 18- to 25-year-olds (33%); lowest
Cannabis Withdrawal Syndrome among 12- and 13-year-olds (0.5% and 2.8%, respec-
tively), and those aged 65 years and older (3.3%)
Cannabis withdrawal is a relatively recently identi-
(NSDUH, 2016).
fied syndrome following abrupt cessation of frequent ■ Sex: Men are nearly twice as likely than women to
cannabis use. Cannabis withdrawal is not as danger-
use cannabis (11.3% versus 6.7%; NSDUH, 2016).
ous as alcohol/sedative-hypnotic withdrawal or as ■ Race: Prevalence of cannabis use varies by race/
uncomfortable as opioid withdrawal but can lead to
ethnicity. From highest to lowest: Mixed race
distressing symptoms that make it difficult to quit. The
(17.7%), Native American (13.6%), Black/African
symptoms of cannabis withdrawal are fairly nonspe-
American (11.1%), non-Hispanic Whites (9.0%),
cific and include anxiety, difficulty sleeping, decreased
Hispanics (7.7%), Pacific Islanders (8.6%), > Asians
appetite, irritability, and restlessness. Most symptoms
(3.3%) (NSDUH, 2016).
have onset within 24 to 72 hours, peak within 1 week, ■ Pregnancy: Although pregnant women are still less
and last for approximately 1 to 2 weeks.
likely to use cannabis compared with nonpregnant
women, cannabis use has been steadily increasing.
Know This:
From 2009 to 2016, self-report in past-month cannabis
Specific polymorphisms of the fatty acid amide hydrolase use among pregnant women aged 15 to 44 increased
(FAAH) gene, which encodes the enzyme metabolizing from 4.2% to 7.1% according to the NSDUH.
AEA, are linked with more severe cannabis withdrawal. ■ Low income and low education are also risk fac-
There is currently a FAAH-inhibitor in clinical trials for tors for increased cannabis use.
the treatment of cannabis use disorder.
Know This:
EPIDEMIOLOGY OF CANNABIS USE Make sure to remember the main surveys used to es-
timate the prevalence of substance use in the United
In terms of the overall epidemiology of cannabis use, States: National Survey on Drug Use and Health
some key facts to remember are: (NSDUH), National Epidemiologic Survey on Alco-
■ Cannabis is by far the most commonly used illicit hol and Related Conditions (NESARC), and Moni-
substance both in the United States and globally. toring the Future (MTF) studies.
■ In 2016, the prevalence of past-year cannabis use
was 13.9%, and past-month prevalence was 8.9% There is an inverse relationship between perceived
based on a nationally representative epidemio- risk and substance use; as the perceived risk of can-
logical survey (National Survey on Drug Use and nabis has been declining among high schoolers, can-
Health [NSDUH], 2016). nabis use has increased. The converse is true regarding
■ Nationally, cannabis use has been rapidly increas- tobacco use among high schoolers. Tobacco use has
ing; from 2007 to 2013, past-month use increased reduced substantially over the past decade.
from 5.8% to 7.5% of the population; the use of
most other illicit drugs over the same time frame LONG-TERM EFFECTS OF CANNABIS
has stabilized or declined (NSDUH, 2016). USE
■ The perceived risk of cannabis has been declin-
ing since 2005 among 8th, 10th, and 12th graders Cannabis Use Disorder
and has been accompanied by an increase in can- Among individuals who have tried cannabis once,
nabis use among these age groups (Monitoring about 9% will eventually develop a cannabis use disor-
the Future, 2013). der. This makes cannabis rank among one of the lesser
78 ADDICTION MEDICINE COMPREHENSIVE REVIEW
addictive substances compared to heroin (24%) and data, cannabis use (not cannabis use disorder) was not
cocaine (15%). associated with the later development of mood or anxi-
The diagnosis of cannabis use disorder is estab- ety disorders, again bringing the question of causality to
lished based on the American Psychiatric Association’s the forefront. Schizophrenia is discussed separately.
Diagnostic and Statistical Manual, 5th Edition diagnos-
tic criteria listed in Chapter 1. Association of Cannabis Use With Schizophrenia
There is no question that there is a strong positive
Know This: association between cannabis use and schizophrenia.
However, a debate has raged in the medical literature
The heritability of lifetime cannabis use is estimated questioning the direction of causality between cannabis
at 45% based on a meta-analysis of twin studies. use and schizophrenia. Does cannabis cause schizophre-
nia, or do genetic risk factors increase the risk for both
The CNR1 gene encodes the cannabinoid type I disorders? This debate has taken on greater urgency
receptor. There is an AAT repeat in an unexpressed as cannabis (and particularly high-potency cannabis)
area of this gene, and individuals with increased num- becomes more widely available to young adults because
ber of AAT repeats may have a higher risk for cannabis of large-scale changes in policy and cultural norms.
use disorder. Here are key points to remember regarding the
association between cannabis and schizophrenia:
Substance Use Comorbidities ■ There is a strong, dose-dependent association
Having a cannabis use disorder is associated with between use of high-potency cannabis use (>10%
increased odds of having a co-occurring alcohol, tobacco, THC concentration) and later development of
and any other drug use disorders. According to find- schizophrenia.
ings from two waves of survey data from the NESARC, ■ Individuals who have a family history of schizo-
cannabis use is also associated with increased odds of phrenia and those who begin using cannabis in
subsequently developing alcohol, tobacco or a drug use adolescence appear more vulnerable to develop
disorder. Keep in mind that even with these temporal psychosis.
findings, causality cannot be inferred between early can- ■ Patients with psychosis who continue to use can-
nabis use and later development of other substance use nabis have a worse prognosis (longer hospitaliza-
disorders; the relationship could alternately be explained tions and more frequent psychotic episodes).
by shared risk factors. ■ Polymorphisms of the
genes AKT1 and COMT are associated with
Know This: increased risk of schizophrenia with exposure to
cannabis; AKT1 is essential for cellular signaling
Among individuals who use cannabis, concurrent alco-
and dopamine transmission. COMT encodes the
hol use is reported at 75%. This has major public health
enzyme responsible for the degradation of cat-
implications for driving accidents related to co-use of
echolamines. There is a high-risk polymorphism
cannabis and alcohol, which is increasingly common.
of the AKT1 gene associated with a sevenfold
risk of developing psychosis with daily cannabis
Psychiatric Comorbidities use compared with those with low-risk polymor-
phism.
Cannabis use and use disorder is associated with signifi-
cant psychiatric comorbidity; cannabis use disorder is
Comorbid Medical Conditions
associated with increased odds of having major depres-
sion, bipolar disorder, schizophrenia, anxiety disorders, Cardiovascular
posttraumatic stress disorder, attention-deficit/hyper- The endocannabinoid system (including CB1 and CB2
activity disorder, and personality disorders. However, receptors) is important in regulating the cardiovas-
according to findings from two waves of NESARC survey cular system, and there are components of marijuana
8 Cannabis Use Disorder 79
smoke that cause cardiovascular damage on the cel- cohort study’s strengths are that it used an enormous
lular level (e.g., carbon monoxide and tar). However, sample (>600,000 women in Ontario, Canada, who gave
the largest study to date examining cardiovascular birth between 2012 and 2017) and compared mothers
outcomes with cannabis use, the CARDIA study (Reis who self-reported cannabis use during pregnancy to
et al., 2017) did not find an association between can- matched controls who did not self-report cannabis use.
nabis use and cardiovascular events. This study found that cannabis use in pregnancy was
associated with a significantly increased risk of preterm
Pulmonary births (10% versus 7%) and an increased risk of the
Pulmonary sequelae of cannabis use occur exclusively via infant requiring transfer to the neonatal intensive care
the smoked or vaped route of administration due to the unit and being small for gestational age.
release of toxic byproducts and chemical irritants, and
include cough and/or bronchitis (Table 8.2). An increas- Know This:
ingly popular route of administration is through vaping,
Remember the results of the Corsi et. al. study. Can-
which involves heating a liquid containing cannabinoids
nabis use during pregnancy is associated with an in-
via a battery-operated device. Compared with smok-
creased risk of preterm birth, infants who are small
ing cannabis through combustion, vaping releases fewer
for gestational age, or newborns who require trans-
toxic byproducts but is still not toxin-free. In the sum-
fer to the neonatal intensive care unit.
mer of 2019, cases of acute lung injury caused by vaping
were identified, called e-cigarette or vaping product use
associated lung injury (EVALI). The majority of affected Prospective cohort studies have not unequivocally
patients vaped THC that contained vitamin E acetate as or consistently shown that cannabis affects child/ado-
an additive and required hospitalization. In severe cases, lescent development. It is biologically plausible that
ventilation and/or lung transplantation were needed. there may be developmental effects because of the
importance of the endocannabinoid system to fetal
Cancer development.
No clear association has been found to date between Cannabinoids readily cross the placenta as well as
cannabis use and lung cancer, although effects can be into breast milk.
hard to separate because of common co-use of canna-
bis and tobacco. Cannabis and Driving
Pregnancy and Cannabis Use Cannabis intoxication increases the risk of car accidents
by a factor of 2 (for comparison, alcohol with a blood
Cannabis is the most widely used illicit substance in
alcohol level >0.08% increases the risk of car accidents by
pregnancy, and its use during pregnancy has been
a factor of 5); however, the compounded risk associated
steadily increasing. Many women report using can-
with cannabis and alcohol use is greater than either alone.
nabis to counteract nausea or treat hyperemesis gravi-
Cannabis intoxication leads to delayed reaction time,
darum. This increase in cannabis use occurs in parallel
poorer hand–eye coordination, and impairs automatic
with the reduction in perceived risk of cannabis; studies
driving. This loss of automaticity can be compensated
to date indicate that most women using cannabis dur-
with higher-order cognitive strategies, but this ability to
ing pregnancy perceive little to no harm in using can-
compensate is diminished with co-use of alcohol.
nabis. Research on the effects of cannabis use during
pregnancy has been limited because cannabis use in
this period is typically accompanied by the use of other Cannabis and Opioid Use
substances (most commonly tobacco), making it dif- Another major debate in the literature is whether can-
ficult to study the independent effects of cannabis on nabis use reduces the prevalence of opioid use, opioid
the developing fetus. In 2019, Corsi et al. published a use disorder, and opioid deaths. Given the current opi-
study in JAMA examining the association between can- oid crisis in the United States, this question has major
nabis use and neonatal outcomes. This retrospective health and policy ramifications.
80 ADDICTION MEDICINE COMPREHENSIVE REVIEW
A widely cited and influential study in this area is by therapy (CBT), and family-based treatments (see
Bachhuber et al. (2014), which analyzed state-level vari- Chapter 13).
ables between 1999 and 2010 and showed that states
that passed medical marijuana laws had fewer state-level Know This:
opioid overdoses compared with those that did not; the The most effective evidence-based psychosocial in-
authors hypothesize that individuals who are using medi- tervention for cannabis use disorder is abstinence-
cal marijuana may be substituting away from opioids. An based voucher CM with CBT; this combination is
updated study by a different group of authors (Shover more effective than either intervention alone.
et al., 2019), however, extended the same analysis to 2017
and found that the trends reversed. Ongoing research
studies with individual-level data on opioid consump- MEDICAL MARIJUANA
tion for individuals using medical marijuana for pain will
prove instrumental in answering this question. History and Policy
Under US federal law, cannabis is a Schedule I con-
Know This: trolled substance. However, individual states can have
medical and recreational cannabis programs because
Although the jury is still out on whether medical of the Justice Department’s Cole Memorandum in 2013,
marijuana reduces the risk of opioid overdose, medi- which stated that states pursuing state-legal cannabis
cal marijuana is approved for opioid substitution enterprises would not be prosecuted. In 2018, Attor-
and opioid use disorder in several states. ney General Jeff Sessions rescinded the Cole Memoran-
dum under President Donald Trump; despite this, in
practice, the Justice Department has generally adhered
TREATMENTS FOR CANNABIS USE to Cole Memorandum-era enforcement priorities.
DISORDER Timeline of cannabis in the United States:
Pharmacotherapy ■ 1700s–1890s: Hemp (from cannabis plants) is legal
Currently, there are no US Food and Drug Adminis- in most states and used to make fibers. Cannabis is
tration (FDA)-approved medications for cannabis use used widely in the United States and Europe.
disorder, although several commonly used psychiatric ■ 1910–1920s: Many states ban cannabis as part of
medications have been studied, are currently under Prohibition, but doctors still prescribe it.
study, or are used off-label. Agonists that have been ■ 1930s: Cannabis is renamed “marijuana” by the
studied for cannabis use disorder include dronabi- United States government as racial tensions
nol and nabilone. Antagonists include rimonabant, a against Mexicans grow during the Great Depres-
CB1 receptor antagonist, and a weight-loss drug whose sion; propaganda films (e.g., Reefer Madness)
use was limited due to the adverse effect of suicidality. suggest it causes insanity.
Rimonabant is no longer available in the United States. ■ 1937: The Marihuana Tax Act was passed, which
Other medications under active investigation include criminalized marijuana, created new taxes on
lofexidine (a centrally acting alpha-2 agonist recently manufacturing or selling cannabis, and created
approved by the FDA for the treatment of opioid with- additional barriers for prescribing doctors.
drawal), gabapentin, and N-acetylcysteine. There are ■ 1952–1956: The Boggs Act of 1952 and Narcotics
negative studies on many medications, including mir- Control Act of 1956 established mandatory sen-
tazapine, naltrexone, nefazodone, quetiapine, baclofen, tencing for cannabis possession.
divalproex, bupropion, and buspirone. ■ 1970s: The Controlled Substance Act is passed in
1970, creating mandatory sentences for marijuana
Psychosocial Interventions possession and use. President Nixon also declares
The primary evidence-based psychosocial treatments the “War on Drugs” and created the Drug Enforce-
include motivational enhancement therapy (MET), ment Agency (DEA) scheduling system, making
contingency management (CM), cognitive behavioral cannabis Schedule I.
8 Cannabis Use Disorder 81
■ 1996: California is the first state to legalize medi- Schedule II, dronabinol is Schedule III, and Epid-
cal marijuana for approved conditions. iolex is Schedule V.
■ 2012: Colorado is the first state to legalize recre-
ational marijuana.
■ As of 2020, 33 states and Washington, DC, allow
REFERENCES
medical cannabis.
Bachhuber, M. A., Saloner, B., Cunningham, C. O., & Barry, C. L.
(2014). Medical cannabis laws and opioid analgesic overdose
Know This: mortality in the United States, 1999-2010. JAMA Internal Medi-
cine, 174(10), 1668–1673.
Remember that a Schedule I drug is defined as a drug Reis, J. P., Auer, R., Bancks, M. P., Goff, D.C., Jr., Pletcher, M. J.,
with no currently accepted medical use, high abuse Rana, J. S., Shikany, J. M., & Sidney, S. (2017). Cumulative
potential, and lack of accepted safety. lifetime marijuana use and incident cardiovascular disease in
middle age: The coronary artery risk development in young
adults (CARDIA) study. American Journal of Public Health,
Clinical Uses for Medical Marijuana 107(4), 601–606.
Shover, C. L., Davis, C. S., Gordon, S. C., & Humphreys, K. (2019).
Cannabis has well-established antiemetic, appetite Association between medical cannabis laws and opioid over-
stimulant, antiepileptic, and analgesic properties. Qual- dose mortality has reversed over time. Proceedings of the Na-
ifying conditions for use of medical marijuana vary tional Academy of Sciences of the United States of America,
116(26), 12624–12626.
by state, but most states include the following: amyo-
trophic lateral sclerosis, HIV/AIDS, inflammatory
bowel disease, multiple sclerosis, severe chronic pain, SUGGESTIONS FOR FURTHER READING
cancer, and posttraumatic stress disorder. Blanco, C., Hasin, D. S., Wall, M. M., Flórez-Salamanca, L., Hoer-
tel, N., Wang, S., Kerridge, B. T., & Olfson, M. (2016). Canna-
Commonly used medicinal cannabinoids that you bis use and risk of psychiatric disorders: Prospective evidence
should remember for the board examination include: from a US national longitudinal study. JAMA Psychiatry,
73(4), 388–395.
■ Dronabinol: Synthetic delta-9-THC approved Brezing, C. A., & Levin, F. R. (2018). The current state of pharma-
for chemotherapy-induced nausea/vomiting and cological treatments for cannabis use disorder and withdrawal.
AIDS wasting Neuropsychopharmacology: Official Publication of the American
■ Nabilone: Synthetic THC analogue approved for College of Neuropsychopharmacology, 43(1), 173–194.
treatment of cancer-related nausea/vomiting Gates, P. J., Sabioni, P., Copeland, J., Le Foll, B., & Gowing, L.
(2016). Psychosocial interventions for cannabis use disor-
■ Epidiolex: Cannabis-derived CBD, which is approved der. The Cochrane Database of Systematic Reviews, 2016(5),
for treatment of Lennox-Gastaut syndrome CD005336.
■ Sativex: An oromucosal spray that is 1:1 delta- Piomelli, D. (2015). Neurobiology of marijuana. In M. Galanter,
9-THC:CBD that is approved in many countries H. D. Kleber, & K. Brady (Eds.), The American Psychiatric
but not currently in the United States; used for Publishing textbook of substance abuse treatment (5th ed.,
pp. 241–250). American Psychiatric Publishing.
treatment of spasticity in multiple sclerosis Welch, S. P., Smith, T. H., Malcolm, R., & Lichtman, A. H. (2019).
The pharmacology of cannabinoids. In S. C. Miller, D. A.
Know This: Fiellin, R. N. Rosenthal, & R. Saitz (Eds.), The ASAM principles
of addiction medicine (6th ed.). Wolters Kluwer.
Dronabinol, nabilone, and Epidiolex are the
only FDA-approved cannabinoids. Nabilone is
82 ADDICTION MEDICINE COMPREHENSIVE REVIEW
REVIEW QUESTIONS
The following vignette is linked to questions 1 and 2. E. Marijuana is no longer a federally controlled
A 20-year-old pre-med college student comes to you substance for individuals aged 18 and over.
because of trouble focusing and academic decline. He
4. Cannabis intoxication is characterized by which
was a straight-A student all through high school and
of the following symptoms?
his first year of college, but this semester he is getting Bs
A. Tachypnea
and Cs. He wonders if he has attention-deficit/hyper-
B. Bradycardia
activity disorder and if stimulant medication might be
C. Piloerection
helpful. He shares with you that he has smoked mari-
D. Hypersalivation
juana several times daily for the past 6 months.
E. Sensation of time “speeding up”
1. Which of the following is a well-known effect
5. A 22-year-old woman in a substance use treat-
of cannabis intoxication that is relevant to his
ment program for severe cannabis use disorder.
presenting problem?
Every week that she has a negative urine toxicol-
A. Long-term memory impairment
ogy, she gets a small prize such as a movie ticket
B. Short-term memory impairment
or a gift card for a coffee shop. This method is a
C. Respiratory depression
component of which evidence-based treatment
D. Distortion of time perception
for cannabis use disorder?
E. Early-onset dementia
A. CBT
2. His mother calls you several months after the B. CM
initial appointment and says that he was taken C. Motivational interviewing
to the emergency room by his roommate af- D. Alcoholics Anonymous
ter he was found at home hearing voices and E. Family-based therapy
paranoid that he was being chased by cops.
6. Your cousin calls you and is panicking because
She states that the roommate found an empty
his 18-year-old daughter told him that she tried
package of K2 on his table. Compared with
marijuana for the first time at a party last week-
plant-based cannabis, which of the following
end. His daughter is functioning well socially
properties of K2 best explains his symptoms?
and academically. Your cousin is terrified that
A. Lower affinity for the CB1 receptor
his daughter will now develop schizophrenia
B. Higher affinity for the CB1 receptor
and become addicted to marijuana and asks
C. Higher affinity for the CB2 receptor
you whether this is true. You tell him which of
D. Higher affinity for the D2 receptor
the following?
E. Lower affinity for the D2 receptor
A. There is a clear positive association between
~
3. The prevalence of marijuana use in the United cannabis use and schizophrenia, and it
States has been increasing over the past decade is well-established that cannabis use is a
among teens. Which of the following explana- causal factor for schizophrenia.
tions best accounts for this trend? B. There is a clear negative association between
A. The perceived risk of marijuana use has cannabis use and schizophrenia, and can-
declined among this age group. nabis use reduces the risk of schizophrenia.
B. The perceived risk of marijuana use has C. There is no clear association between can-
increased among this age group. nabis use and schizophrenia.
C. THC content in marijuana has declined, D. The majority of individuals who try cannabis
making it more palatable to naïve users. once will not develop a cannabis use disorder.
D. CBD content in marijuana has increased, E. The majority of young adults who try canna-
making it more palatable to naïve users. bis once will develop a cannabis use disorder.
8 Cannabis Use Disorder 83
■ Cocaine is an alkaloid that can be found in the (PER1) gene have abnormal locomotor sensiti-
leaves of the Erythroxylum genus of plants. zation and conditioned preference for cocaine.
■ Cocaine exists in two chemical forms: as a salt
TA B L E 9 .1
Mechanism of Action and Molecular Targets of Common Stimulants
Drug Mechanism of Action Molecular Target
Cocaine Stimulates CNS by inhibiting presynaptic reuptake DAT, neuronal plasma membrane transporters for
of dopamine and norepinephrine. Blocks neuronal norepinephrine and serotonin, and neuronal sodium
sodium channels, accounting for its local anesthetic channels
properties
Methamphetamine Increases release of monoamine neurotransmitters VMAT2
through its effects on VMAT2 in addition to some DAT, and neuronal plasma membrane transporters
inhibition of dopamine and norepinephrine reuptake for norepinephrine and serotonin
Amphetamine Similar to mechanism of action for VMAT2
analogues methamphetamine DAT, and neuronal plasma membrane transporters
for norepinephrine and serotonin
Methylphenidate Similar to mechanism of action for DAT, and neuronal plasma membrane transporters
methamphetamine, but methylphenidate does not for norepinephrine and serotonin
enhance dopamine release from synaptic vesicles
Khat and Cathinones Similar to mechanism of methamphetamine VMAT2
DAT, and neuronal plasma membrane transporters
for norepinephrine and serotonin
MDMA Promotes release of serotonin, dopamine, and VMAT2, TA1-receptor (TAAR1), serotonin (5-HT1A,
(3,4-Methylenedioxy norepinephrine into synaptic clefts; prevents 5-HT2A, 5-HT2B, 5-HT2C), dopamine (D1, D2), and
methamphetamine) serotonin reuptake; acts as agonist at serotonin, alpha- and beta-adrenergic receptors
dopamine, and alpha- and beta-adrenergic receptors
Pseudoephedrine Stimulates alpha- and beta-2-adrenergic receptors, alpha- and beta-2-adrenergic receptors
causing vasoconstriction of blood vessels and
relaxation of smooth muscle in airways
Phenylephrine Agonist at alpha-1-adrenoceptors, causing alpha-1-adrenergic receptors
vasoconstriction of blood vessels
Nicotine (discussed in Agonist at nicotinic acetylcholine receptors Nicotinic acetylcholine receptors
detail in Chapter 7)
Caffeine (discussed in Nonselective adenosine receptor antagonist and Adenosine-1, adenosine-2a, and adenosine-2b
detail in Chapter 10) phosphodiesterase inhibitor receptors
CNS, Central nervous system; DAT, dopamine transporter; MDMA, 3,4-methylenedioxymethamphetamine; VMAT2, vesicular monoamine
transporter-2.
STIMULANT WITHDRAWAL
SYNDROME ATTENTION-DEFICIT/
HYPERACTIVITY DISORDER AND
The abrupt discontinuation of stimulant use in individ-
uals with chronic and excessive stimulant use results in
STIMULANT USE DISORDER
a stimulant withdrawal syndrome marked by the signs Attention-deficit/hyperactivity disorder (ADHD) has a
and symptoms summarized in the table below. Note high prevalence in patients with substance use disorders
that some withdrawal symptoms occur within a few (SUDs) compared to the general population. Stimulant
hours and up to a few days after last use. Severity of medications (amphetamine analogues and methylphe-
symptoms is based on the amount of stimulants used. nidates) are highly effective in treating ADHD and are
currently the first-line pharmacologic treatment for
Physiological Psychological children and adults. For patients with active SUD and
Dehydration Dysphoric mood comorbid ADHD, the risks and benefits of treating
ADHD with stimulant medication must be weighed
Fatigue Cravings
carefully. In patients with active SUDs, FDA-approved
Psychomotor retardation Vivid dreams nonstimulant medications should be considered first-
before agitation Anxiety line; these include atomoxetine, guanfacine, and cloni-
Weight loss or anorexia Impaired memory dine. Other off-label medications used to treat ADHD
Insomnia followed by Anhedonia include bupropion, venlafaxine, and modafinil.
hypersomnia Irritability Individuals with SUDs and ADHD have an earlier
onset of substance use than those without ADHD.
Stimulant withdrawal syndrome is not medically Treating a patient with ADHD has not been directly
life-threatening but the risk of self-mutilating behav- linked to having higher risk of having a stimulant use
iors, suicide, and risk to others may be high during the disorder later in life. In fact, several studies have sug-
withdrawal period and management is recommended. gested that ADHD treatment using stimulant medica-
tions may reduce the risk of illicit stimulant use.
Know This:
Know This:
Chronic use of cocaine may lead to perforated nasal
septum due to vasoconstriction and resulting isch- Students with and without ADHD misuse prescrip-
emic necrosis. tion stimulants to:
■ Promote academic performance
■ Lose weight
Chronic cocaine use can damage many organs by ■ Increase energy and wakefulness
reducing blood flow with toxic effects on the cardio- ■ Induce euphoria
vascular, neurological, and gastrointestinal systems.
9 Stimulant Use Disorder 89
SUGGESTIONS FOR FURTHER READING Newton, T. F., Roache, J. D., De La Garza, R., 2nd, Fong, T., Wallace,
C. L., Li, S. H., Elkashef, A., Chiang, N., & Kahn, R. (2006). Bu-
Abarca, C., Albrecht, U., & Spanagel, R. (2002). Cocaine sensitiza-
propion reduces methamphetamine-induced subjective effects
tion and reward are under the influence of circadian genes and
and cue-induced craving. Neuropsychopharmacology: Official
rhythm. Proceedings of the National Academy of Sciences of the
Publication of the American College of Neuropsychopharmacol-
United States of America, 99(13), 9026–9030.
ogy, 31(7), 1537–1544.
Colfax, G. N., Santos, G. M., Das, M., Santos, D. M., Matheson, T.,
Sadler, B., Haller, G., Agrawal, A., Culverhouse, R., Bucholz, K.,
Gasper, J., Shoptaw, S., & Vittinghoff, E. (2011). Mirtazapine to
Brooks, A., Tischfield, J., Johnson, E. O., Edenberg, H., Schuck-
reduce methamphetamine use: A randomized controlled trial.
it, M., Saccone, N., Bierut, L., & Goate, A. (2014). Variants near
Archives of General Psychiatry, 68(11), 1168–1175.
CHRNB3-CHRNA6 are associated with DSM-5 cocaine use
Grant, J. E., Odlaug, B. L., & Kim, S. W. (2010). A double-blind,
disorder: Evidence for pleiotropy. Scientific Reports, 4, 4497.
placebo-controlled study of N-acetyl cysteine plus naltrexone
Schmitz, J. M., Lindsay, J. A., Green, C. E., Herin, D. V., Stotts, A.
for methamphetamine dependence. European Neuropsycho-
L., & Moeller, F. G. (2009). High-dose naltrexone therapy for
pharmacology: The Journal of the European College of Neuropsy-
cocaine-alcohol dependence. American Journal on Addictions,
chopharmacology, 20(11), 823–828.
18(5), 356–362.
Levin, F. R., Mariani, J. J., Pavlicova, M., Choi, C. J., Mahony, A. L.,
Trivedi, M. H., Walker, R., Ling, W., Dela Cruz, A., Sharma, G.,
Brooks, D. J., Bisaga, A., Dakwar, E., Carpenter, K. M., Naqvi, N.,
Carmody, T., Ghitza, U. E., Wahle, A., Kim, M., Shores-
Nunes, E. V., & Kampman, K. (2020). Extended release mixed
Wilson, K., Sparenborg, S., Coffin, P., Schmitz, J., Wiest, K.,
amphetamine salts and topiramate for cocaine dependence: A
Bart, G., Sonne, S. C., Wakhlu, S., Rush, A. J., Nunes, E. V.,
randomized clinical replication trial with frequent users. Drug
& Shoptaw, S. (2021). Bupropion and naltrexone in metham-
and Alcohol Dependence, 206, Article 107700.
phetamine use disorder. The New England Journal of Medi-
Ma, J. Z., Johnson, B. A., Yu, E., Weiss, D., McSherry, F., Saadvandi, J.,
cine, 384(2), 140–153.
Iturriaga, E., Ait-Daoud, N., Rawson, R. A., Hrymoc, M., Camp-
Volkow, N. D., Wang, G. J., Fischman, M. W., Foltin, R., Fowler, J. S.,
bell, J., Gorodetzky, C., Haning, W., Carlton, B., Mawhinney, J.,
Franceschi, D., Franceschi, M., Logan, J., Gatley, S. J., Wong, C.,
Weis, D., McCann, M., Pham, T., Stock, C., Dickinson, R., &
Ding, Y. S., Hitzemann, R., & Pappas, N. (2000). Effects of route
Li, M. D. (2013). Fine-grain analysis of the treatment effect of
of administration on cocaine induced dopamine transporter
topiramate on methamphetamine addiction with latent variable
blockade in the human brain. Life Sciences, 67(12), 1507–1515.
analysis. Drug and Alcohol Dependence, 130(1-3), 45–51.
REVIEW QUESTIONS
1. Which of the following medications, when com- improvements in sustained attention, metham-
bined with CM, has been shown to reduce co- phetamine-associated increases in diastolic blood
caine use more than either treatment alone or pressure, and self-reported feelings of anxiety in
placebo? people with methamphetamine use disorder?
A. D-Amphetamine A. Varenicline
B. Methylphenidate B. Modafinil
C. Modafinil C. Amphetamines
D. Bupropion D. Glutamate agonists
E. Disulfiram E. Rivastigmine
2. A 26-year-old White male law student in his 3. You determined that your patient meets criteria
second year of law school had a long history of for cocaine use disorder. Which of the following
experimental drug use, including alcohol and psy- treatment modalities proven to treat cocaine use
choactive drugs. Approximately 2 years ago, he disorder is based on the principles of operant
was introduced to cocaine in a social setting by conditioning?
friends. He has moved to a more daily pattern in A. CM
the last 2 months. He has found that the inha- B. Standard counseling
lation of methamphetamine stimulated his per- C. Twelve-step facilitation
formance and ability to study all night. Which D. CBT
of the following medications have demonstrated E. Family-based therapy
90 ADDICTION MEDICINE COMPREHENSIVE REVIEW
4. A 31-year-old single Hispanic woman with a 6. A 21-year-old woman was brought to urgent care
history of a SUD was taken to the emergency by a friend. The friend states that she has been
room by her friend with new-onset paranoid extremely anxious today. On examination, she
delusions and agitation. Per her girlfriend, she has dilated pupils, tachycardia, and diaphoresis.
has been smoking “something different” that a She is talking in complete sentences and denied
friend has been making using common house- the use of any substance. She denies nausea,
hold equipment, over-the-counter drugs, and vomiting, or changes in appetite. Use of which of
some other chemicals at his place. What are the following substances is most likely to explain
some of the neurologic effects of the substance the patient’s presentation?
she is using? A. Heroin
A. It induces intracellular dopamine-containing B. Naloxone
vesicles to release dopamine into the C. Amphetamine
synaptic cleft and blocks the reuptake of D. Alcohol
dopamine. E. Tobacco
B. It blocks the reuptake of dopamine from the
7. Specific variants in which of the following genes
synaptic cleft
would increase the risk of having a cocaine use
C. It breaks down L-DOPA into dopamine
disorder?
D. It competitively binds to adenosine receptors.
A. ADHD1
E. It induces extracellular dopamine-containing
B. DAT1
vesicles to release dopamine into the synap-
C. VNTR
tic cleft.
D. CAMK4
5. An antagonist of the noradrenergic alpha-2 E. ADH1
receptor and the 5-HT2A/C and 5-HT3 recep-
8. Which of the following is an adverse effect of
tors has demonstrated efficacy in reducing the
MDMA use?
rewarding effect of illicit stimulant use. Which
A. Lacrimation
compound has this pharmacological profile?
B. Miosis
A. Fluoxetine
C. Vomiting
B. Cannabis
D. Bruxism
C. Glutamate
E. Weight gain
D. Mirtazapine
E. Bupropion
DISSOCIATIVE DRUGS,
10
HALLUCINOGENS, CAFFEINE,
INHALANTS, AND OTHER
SUBSTANCES OF ABUSE
91
92 ADDICTION MEDICINE COMPREHENSIVE REVIEW
including many stimulants, hallucinogens, and GHB receptors. The effects of GHB include relaxation,
neurotransmitters. tranquility, and sensory and sexual enhancement; low
doses resemble the effects of mild alcohol intoxication.
■ Mescaline is a natural hallucinogen found in
cacti, including peyote and the San Pedro cactus. ■ GHB is also abused by bodybuilders. Although
■ Several so-called “research chemicals” have been there is limited evidence for this, GHB is believed
synthesized and are phenethylamine derivatives to increase fat metabolism and release of growth
with hallucinogenic effects. These include 2CB, hormone.
2CE, 2CI, and others. 2CI is one of the better- ■ It has a half-life of 30 to 60 minutes.
known research chemicals and was popularized ■ GHB has amnestic effects and is known for its
in a book by Dr. Alexander Shulgin and Anne use as a date-rape drug. Because of its short half-
Shulgin, who synthesized 2CI and many other life and lack of a rapid toxicology test, its pres-
phenethylamine-based compounds ence is difficult to detect.
■ Signs and symptoms of GHB intoxication
Atypical Hallucinogens include respiratory depression and bradycardia;
S. divinorum and hyoscine have hallucinogenic effects overdose is a medical emergency and can lead to
but differ from classical hallucinogens in that seroto- stupor, coma, and death.
nergic receptors do not mediate their effects. ■ GHB dependence and withdrawal syndromes
can develop with chronic use; withdrawal symp-
■ S. divinorum is the name of the plant from which tomatology can be treated with benzodiazepines
salvinorin A, a potent hallucinogen, is derived. or barbiturates.
Salvinorin A is a kappa-opioid agonist and does ■ Therapeutically, GHB is used for the treatment of
not share a structural similarity with any syn- narcolepsy with cataplexy and is available as the
thetic or naturally occurring opioid. Schedule III medication sodium oxybate.
■ Hyoscine, also known as scopolamine, is found
in several plants in the nightshade family and has
antimuscarinic anticholinergic activity. An over- FLUNITRAZEPAM
dose can present with delirium, hallucinations, Flunitrazepam, also referred to as “roofies,” is a benzodiaz-
and anticholinergic features. epine with rapid onset and a half-life of 20 hours. It causes
anterograde amnesia and is commonly used as a date-rape
Know This: drug. It is no longer available in the United States.
Some individuals continue to experience flashbacks
and reexperiencing of hallucinations (primarily visu- ANABOLIC ANDROGENIC STEROIDS
al) well after the intoxicant effects of the hallucino-
Anabolic androgenic steroids (AAS) are hormones
gen have ended. These flashbacks can occur weeks or
that include testosterone and its analogues. They bind
even years later from the last use. This phenomenon
to the androgen receptor producing their anabolic
is referred to as hallucinogen persisting perception
and androgenic effects, and have psychoactive effects
disorder and has been most frequently described fol-
through allosteric modulation and enhancement of
lowing use of LSD.
the GABA-A receptor. Steroid abuse is more common
among males, with an average age of onset of 20 years
GAMMA-HYDROXYBUTYRATE old, with up to one-third using steroids chronically
and developing dependence. The most common rea-
Gamma-hydroxybutyrate (GHB) is a neurotransmit- sons for abusing these compounds are for physical
ter and both a precursor and metabolite of GABA. or performance enhancement. Anabolic steroids are
Given its similarity to GABA, GHB is a central nervous available in different formulations, including as oral
system (CNS) inhibitor and considered a sedative- medications, topical gels, intramuscular injections,
hypnotic. GHB acts at both GABA-B and specific and transdermal patches.
94 ADDICTION MEDICINE COMPREHENSIVE REVIEW
Adverse Effects TA B L E 1 0 .1
■ Anabolic steroid use can lead to weight gain, Methods of Abusing Inhalants
kidney failure, alteration in serum lipids, liver Method of Use Action
damage, heart disease, and increased risk of Sniffing or snorting Directly inhaling fumes
coagulopathy and stroke. Bagging Inhaling fumes from substances
■ In males, AAS use can lead to baldness, gyneco- sprayed or deposited inside a bag
mastia, smaller testes and lower sperm count. In Huffing Spraying or soaking the substance
females, it can lead to increased facial and body on a rag or cloth to then holding it
hair, an enlarged clitoris, deepening of the voice, over the mouth or nose
and menstrual cycle irregularities. Glading Inhaling an air freshener aerosol
■ Long-term changes in the brain can lead to mood Dusting Spraying aerosol cleaners directly
changes and behavior, including paranoia, delu- into the mouth or nose
sions, and aggression.
age of inhalant abuse is between the ages of 14 and 16 found in many industrial solvents.
years. Inhalant abuse is more common in individuals ■ Intoxication presents similarly to an alcohol
There is usually a rapid pulmonary absorption, as these lemia, metabolic acidosis, distal renal tubular
are highly lipid-soluble compounds, reaching the brain acidosis, rhabdomyolysis, and muscle paralysis.
in seconds. The initial effects of these substances are ■ Long-term adverse effects on the nervous sys-
similar to those of anesthetic agents. Methods of abus- tem are severe and include dementia, ataxia,
ing inhalants are listed in Table 10.1. cerebral and cerebellar atrophy, and vision loss.
Routine urine drug screenings do not detect inhal-
ants, so detection relies on the clinical diagnosis. Nitrous Oxide
Some laboratory findings include an elevation of liver ■ Although classified as an inhalant, nitrous oxide
enzymes. There is no FDA-approved psychopharmaco- (NO) is an NMDA antagonist and dissociative.
logical treatment for inhalant abuse, and recommended NO is clinically used as a general anesthetic in
treatments include individual and family therapy. humans and animals.
10 Dissociative Drugs, Hallucinogens, Caffeine, Inhalants, and Other Substances of Abuse 95
do not differ much from those used to treat other Center for Behavioral Health Statistics and Quality. (2019). 2018 Na-
addictive behaviors. These treatments include indi- tional survey on drug use and health final analytic file codebook.
Substance Abuse and Mental Health Services Administration.
vidual and family therapy. Drug Enforcement Agency. (2017). Khat. In Drugs of Abuse: A
DEA resource guide: 2017 edition. https://www.dea.gov/sites/
REFERENCE default/files/sites/getsmartaboutdrugs.com/files/publications/
American Psychiatric Association. (2013). The diagnostic and statis- DoA_2017Ed_Updated_6.16.17.pdf#page=53.
tical manual of mental disorders (5th ed.). American Psychiatric Ferré, S. (2010). Role of the central ascending neurotransmitter sys-
Association. tems in the psychostimulant effects of caffeine. Journal of Al-
zheimer’s Disease, 20(Suppl1), S35–S49.
Sanacora, G., Heimer, H., Hartman, D., Mathew, S. J., Frye, M., Nem-
SUGGESTIONS FOR FURTHER READING eroff, C., & Robinson Beale, R. (2017). Balancing the promise
Boot, B. P., McGregor, I. S., & Hall, W. (2000). MDMA (ecstasy) neu- and risks of ketamine treatment for mood disorders. Neuropsy-
rotoxicity: Assessing and communicating the risks. Lancet, 355, chopharmacology, 42(6), 1179–1181.
1818–1821.
REVIEW QUESTIONS
following is the most likely explanation for the room with signs and symptoms of serotonin syn-
patient’s presentation? drome after ingesting a hallucinogen. Ingestion
A. Alcohol withdrawal of which of the following hallucinogens is most
B. PCP intoxication likely to lead to this presentation?
C. Cocaine intoxication A. Psilocybin
D. Dehydration B. LSD
E. Dementia with Lewy body C. S. divinorum
7. A 32-year-old man with a history of major de- D. Ayahuasca
pression on fluoxetine presents to the emergency E. DMT
11 ASPECTS OF PAIN
99
100 ADDICTION MEDICINE COMPREHENSIVE REVIEW
The Diagnostic and Statistical Manual of ■ At least one symptom is continuously present,
Mental Disorders, Fifth Edition, and Pain although different symptoms and current
The Diagnostic and Statistical Manual of Mental Dis- symptoms may come and go.
orders, Fourth Edition, pain disorder diagnosis was Pain Scales
eliminated and largely replaced by the Diagnostic
■ Visual analog scale: Respondents are asked to
and Statistical Manual of Mental Disorders, Fifth Edi-
report present pain intensity or pain intensity in
tion, of somatic symptom disorder. People suffering
the last 24 hours (Fig. 11.1).
from this disorder have a significant focus on physi-
■ Wong-Baker FACES Pain Rating Scale: The
cal symptoms that result in concern and problems
assessment is done by only looking at the patient’s
functioning.
facial expressions (Fig. 11.2).
Somatic Symptom Disorder:
Diagnostic Criteria PHARMACOTHERAPIES FOR PAIN
■ One or more physical symptoms that are distressing
SYNDROMES
or cause disruption in daily life. When choosing a medication to treat pain, it is essen-
■ Excessive thoughts, feelings, or behaviors related tial to consider the medication’s duration, the onset of
to the physical symptoms or health concerns action, and prior experiences. We review below some
with at least one of the following: commonly used medications effective in relieving pain.
■ Ongoing thoughts that are out of proportion
0 1 2 3 4 5 6 7 8 9 10
No Moderate Worst
pain pain possible
pain
Fig. 11.1 ■ The Numeric Pain Rating Scale. The patient is asked to make three pain ratings, corresponding to current, best, and
worst pain experienced over the past 24 hours. The average of the three ratings is used to represent the patient’s level of pain over
the previous 24 hours. (From Potter, P. A. (2016). Pain management. In A. G. Perry, P. A. Potter, & W. R. Ostendorf (Eds.), Nursing
interventions & clinical skills (6th ed., p. 319). Elsevier.)
Fig. 11.2 ■ Wong-Baker FACES Pain Rating Scale. (From Wong-Baker FACES Foundation (2020). Wong-Baker FACES pain rating
scale. Retrieved February 24, 2021 with permission from http://www.WongBakerFACES.org. Originally published in Whaley &
Wong’s Nursing Care of Infants and Children. Elsevier.)
11 Aspects of Pain 101
nonselective (e.g., ibuprofen, naproxen), or partially absorption, short duration of action, and toxic
selective (e.g., meloxicam, diclofenac). NSAIDs’ side metabolite (normeperidine), which can cause trem-
effects include ulcers and upper gastrointestinal tract ors and seizures.
bleeding, worsening congestive heart failure, aseptic
meningitis, psychosis, and tinnitus. NSAID use can Opioids
adversely affect the kidney by inducing sodium reten- Prescription opioids are among the many options for
tion and antagonizing diuretics. treating severe acute pain. Opioid analgesics are not rec-
ommended for the chronic treatment of pain syndromes.
Paracetamol (Acetaminophen) Opioids are discussed in more detail in Chapter 6.
Paracetamol works by blocking cyclooxygenase-2 and
inhibiting endocannabinoid reuptake in the central ner- Know This:
vous system. It carries minor antiinflammatory activity.
Meperidine does not cause pinpoint pupils.
Tricyclic Antidepressants
Tricyclic antidepressants block the reuptake of norepi-
SUGGESTIONS FOR FURTHER READING
nephrine and serotonin, increasing serotonin levels, and
Apfelbaum, J. L., Chen, C., Mehta, S. S., & Gan, T. J. (2003). Postop-
enhancing norepinephrine activity. They function as an
erative pain experience: results from a national survey suggest
effective adjunctive pain management medication. postoperative pain continues to be undermanaged. Anesthesia
and Analgesia, 97(2), 534–540.
Gabapentin Brennan, F., Carr, D. B., & Cousins, M. (2007). Pain management:
a fundamental human right. Anesthesia and Analgesia, 105(1),
Gabapentin carries direct and indirect effects on the 205–221.
GABA receptors and provides significant relief from Hall, J. E. (2010). Guyton and Hall textbook of medical physiology e-
neuropathic pain. book. Elsevier Health Sciences.
Kaufman, D. M., Geyer, H. L., & Milstein, M. J. (2016). Kaufman’s
Meperidine clinical neurology for psychiatrists (8th ed.). Elsevier.
Wong-Baker FACES Foundation. (2020). Wong-Baker FACES Pain
Meperidine is an opioid with serotonergic and anti- Rating Scale. Retrieved from http://www.WongBakerFACES.org.
cholinergic effects. It is not an ideal choice for the Originally published in Whaley & Wong’s nursing care of infants
management of chronic pain due to its poor oral and children. Elsevier.
REVIEW QUESTIONS
1. A 61-year-old patient presents with low back 2. A 68-year-old female patient with a history of lung
pain. He was recommended to start taking cancer and metastases to the lumbar spine is receiv-
ibuprofen as needed for his pain. He has been ing oxycodone 10 mg three times a day. The patient’s
eating well and denied any lifestyle changes. partner tells the medical resident that the patient
Six weeks later, he presents to the office with exhibits a lack of energy and motivation, feelings of
13 pounds of weight gain. Which of the following worthiness, low concentration, and reduced appetite
is the best explanation for the patient’s weight in the past 2 weeks. What should be the next step?
gain? A. Start an antidepressant
A. A decrease in renal function B. Physical therapy evaluation
B. A decrease in temperature C. Psychiatric evaluation
C. A decrease in creatinine levels D. The patient’s pain assessment
D. An increase in the glomerular filtration rate E. Medication assisted treatment for opioid use
E. An increase in plasma proteins disorder
102 ADDICTION MEDICINE COMPREHENSIVE REVIEW
DRUG AND ALCOHOL TESTING (e.g., for cannabis). The most basic UDS test is
known as SAMHSA-5 assays for amphetamines,
Drug and alcohol testing can and should be ordered in cocaine, marijuana, opiates, and phencyclidine
various settings. Understanding the context for which and is used by the Department of Transportation
such a thing is being ordered can help determine test- and other federal agencies. UDS testing is typi-
ing procedures. For example, for parents seeking to cally expanded in clinical settings to include vari-
monitor their children’s drug use, point-of-care testing ous other substances, including benzodiazepines,
at home may suffice. In contrast, when working with barbiturates, or synthetic opioids. Any positive
patients being evaluated in an emergency department test on a UDS should be sent to GC-MS confirma-
setting, a urine toxicology screening test provides tion testing. UDS uses cut-off values for screening
quick information and may guide clinical decisions. that are typically higher than the cut-off values
Substance use disorder (SUD) treatment providers fre- used for confirmation testing. UDS results are
quently use urine toxicology screening tests followed qualitative, showing whether the individual tested
by gas chromatography–mass spectrometry (GC-MS) positive (above the cut-off value) or negative for a
confirmation of any positive screens to guide treat- given drug. GC-MS confirmation results are both
ment regimen and monitor treatment outcomes. Per- qualitative and quantitative, showing the metabo-
sons sentenced of driving under the influence (DUI) of lite concentration tested in the urine. Quantitative
alcohol may be required to undergo monitoring with results should never be used to assess drug use fre-
transdermal alcohol testing bracelets. Liver transplant quency given the significant variability in urine-
evaluation frequently uses a battery of biomarkers to specific gravity and electrolyte content.
determine whether or not the individual abstains from ■ Hair: Hair drug testing can demonstrate drug use
alcohol. Finally, custody evaluations and other foren- with a detection period of up to 4 or 5 months.
sic assessments frequently rely on hair drug testing Testing results are both qualitative and quan-
to assess the evaluee’s drug use over the past several titative, showing the metabolite concentration
months. Medical Review Officers are physicians with tested in hair. Unlike urine testing, quantitative
added qualifications for interpreting toxicology testing results obtained from hair testing can distinguish
results. between light, moderate, and heavy drug use.
Drug Testing Compared to urine testing, hair testing is more
expensive, takes longer for results to become
Matrices available, and may miss recent drug use.
■ Urine: Urine drug screening (UDS) utilizes anti- ■ Saliva and nails: Drug testing using saliva or nails
body immunoassays that react to various drugs. as testing matrices are available. However, their
The detection period varies based on the drug and results are not as validated and clinically useful
its metabolism, ranging from 1 day to 1 month as urine or hair testing.
103
104 ADDICTION MEDICINE COMPREHENSIVE REVIEW
■ Liver function test: An indirect biomarker of ■ Loyalty principle: Requires that health care pro-
excessive and chronic alcohol use. It typically viders act in a manner that benefits their patients
shows elevated alanine aminotransferase and above any other interest.
aspartate aminotransferase levels.
In medical practice, it is not uncommon to face sit-
uations in which there is discordance in applying the
Know This:
ethical principles listed above. For example, in SUD
The Widmark equation is used to estimate blood treatment, needle exchange programs can be viewed
alcohol levels (BALs). BAL = weight of alcohol used as consistent with the principle of beneficence but not
(in oz)/r × weight of the subject (in oz); r, or the of nonmaleficence. Different ethical theories can help
Widmark factor, is approximately 0.7 for males and guide clinical decision making in such situations:
0.6 for females. ■ Dialectical principlism theory is based on the
major moral principles and requires balancing
competing principles to rich and ethically accep
MEDICAL ETHICS IN ADDICTION table solutions.
PSYCHIATRY AND ADDICTION ■ Deontological theory deals with the clinician’s
MEDICINE duty and moral obligation to “do the right
Ethics is a branch of philosophy that deals with per- thing.”
sonal, professional, and social values in human con- ■ Utilitarianism theory aims to achieve the greatest
duct. It provides a discussion of right and wrong, good for the greatest number of people.
good or bad, and the motives and ends of such con- ■ Liberal individualism theory places paramount
duct. In the context of medical practice, the following importance on the patient’s rights and auto
ethical principles are identified (note that autonomy, nomy.
beneficence, nonmaleficence, and justice are typically ■ Virtue theory examines whether the clinical
referred to as the four principles of medical ethics): provider intended to do good by the patient,
showing good character and moral values while
■ Autonomy principle: Requires that patients are placing less emphasis on strictly following the
free to make their own choices as long as they are rules and consequences.
competent and have autonomy in their thought ■ Consequentialism theory determines the moral
process, intentions, and actions when making worth of an action based on whether its outcomes
medical decisions. As such, the medical deci- or consequences are good or bad.
sion-making process must be free of coercion or
undue influence. FORENSIC ADDICTION PSYCHIATRY
■ Beneficence principle: Requires that any medi-
AND ADDICTION MEDICINE
cal decision offered, or any clinical intervention
performed serve to promote patients’ health and Forensic psychiatry is the psychiatric subspecialty in
welfare. which psychiatric principles are applied in legal con-
■ Nonmaleficence principle: Requires that any texts. In the following, we review important forensic
medical decision offered, or any clinical inter- psychiatric topics relevant to SUD treatment practice.
vention performed, does not harm the patient.
■ Justice principle: Requires that health care pro- Important SUD Legislation
viders make medical decisions that treat patients Harrison Narcotic Act of 1914: Congress passed this
equally and that the burden and benefits of new legislation to regulate the production, importation,
or experimental interventions are distributed and distribution of opioids. The act included protec-
fairly across all patients. tions for physicians prescribing opioids “at the cost of
■ Fidelity principle: Requires that health care provid- his professional practice only.” Around the time the act
ers follow through with promises made to patients. was passed, SUD was not considered a medical illness,
12 Drug Testing, Forensic Addictions, and Ethics 107
and, as a result, doctors were not allowed to prescribe may not be considered a state offense, it is constitu-
opioids to those with opioid use disorder. Many phy- tional, however, for substance-related misconduct to
sicians were arrested and imprisoned for prescribing be subject to both civil and criminal prosecutions.
opioids to SUD patients. People v. Saille (California Supreme Court; 1991):
Controlled Substances Act of 1970: In 1970, the In this case, the California Supreme Court ruled that
federal government enacted the Controlled Substances voluntary intoxication may not be used as a defense in
Act, classifying various addictive drugs based on their criminal procedures.
addictive potential, the potential for harm, and thera- Montana v. Egelhoff (United States Supreme Court;
peutic potential. The act has been amended multiple 1996): In this case, the Supreme Court found that
times since to include new and emerging drugs. Com- excluding voluntary intoxication related to a mental
monly used drugs are classified as follows: state in a criminal offense does not violate due process
protections of the Constitution. As such, it is consti-
■ Schedule I: marijuana, lysergic acid diethylamide
tutional for states to decline excusing a crime on the
(LSD), heroin
basis of voluntary intoxication.
■ Schedule II: morphine, fentanyl, methadone,
Board of Education v. Earls (United States Supreme
hydrocodone, oxycodone, amphetamine
Court; 2002): In this case, the Supreme Court ruled
■ Schedule III: buprenorphine, ketamine,
that it is legal to randomly drug test public school stu-
dronabinol
dents who participate in extracurricular activities.
■ Schedule IV: alprazolam, clonazepam, diazepam,
zolpidem
Know This:
■ Schedule V: pregabalin, codeine, diphenoxylate
■ Multiple forensic issues arise in treating ado-
Drug Addiction Treatment Act (DATA) of 2000: ##
SUGGESTIONS FOR FURTHER READING Powell v. Texas, 392 U.S. 514, 88 S. Ct. 2145 (1968).
Robinson v. California, 70 U.S. 660 (more) 82 S. Ct. 1417; 8 L. Ed. 2d
American Academy of Psychiatry and the Law. (2005). Ethics guide-
758; 1962 U.S. LEXIS 850.
lines for the practice of forensic psychiatry. http://www.aapl.org/
Swotinsky, R. B. (2014). The medical review officer’s manual:
ethics-guidelines.
MROCC’S guide to drug testing (5th ed.). OEM Press.
Aoun, E. G., & Kim, J. (2020). Forensic addiction psychiatry. In K.
T. Brady, F. R. Leven, M. Galanter, & H. D. Kleber (Eds.), The
American Psychiatric Press textbook of substance abuse (6th ed.).
American Psychiatric Press Publishing.
REVIEW QUESTIONS
1. Which of the following is true about SUDs C. A positive random drug test suggests that the
among physicians? individual uses the drug recreationally but
A. SUD prevalence rates among psychiatrists does not have an SUD.
and pathologists are higher than the general D. A positive drug test shows recent drug use.
population. E. Hair testing for biomarkers of excessive
B. SUD prevalence is lower for physicians than alcohol use is an effective strategy to ensure
the general population. abstinence for someone who is recently
C. SUD prevalence rates among emergency phy- charged with a DUI.
sicians and anesthesiologists are higher than
3. Which of the following clinical situations is associ-
the general population.
ated with the highest risk of violence and criminal
D. Physicians referred to physician health
behaviors?
programs have a worse prognosis than those
A. A 34-year-old man with major depressive
who do not.
disorder
E. The prognosis of physicians with SUD is
B. A 26-year-old woman with cannabis use
worse than the general population.
disorder
2. Which of the following statements about drug C. A 19-year-old man with schizophrenia and
and alcohol testing is correct? phencyclidine intoxication
A. Drug testing is the most effective means of D. A 24-year-old woman with cocaine use
detecting SUDs. disorder
B. A positive for-cause drug test performed E. A 69-year-old man with alcohol use disorder
indicates that the individual most likely has
an SUD.
13
PSYCHOSOCIAL APPROACHES
TO SUBSTANCE USE DISORDER
MANAGEMENT
109
110 ADDICTION PSYCHIATRIC MEDICINE: A COMPREHENSIVE BOARD REVIEW
even the support of group fellowship) or the “alcoholic” The 12 steps of AA:
label cannot meaningfully participate in AA.
1. I can’t (We are powerless over alcohol, and our
For such persons finding AA unsuitable to their
lives have become unmanageable.)
needs, organizations such as SMART Recovery, Sec-
2. God can (Power greater than ourselves can
ular Organizations for Sobriety, Moderation Man-
restore us to sanity.)
agement, and LifeRing serve as an alternative to the
3. Let God (Turn our will and our lives over to
spiritually oriented AA. We will use SMART Recovery
the care of God as we understood Him.)
as a model for this discussion alternative to 12-step-
4. Look within (Made a searching and fearless
based mutual-support groups.
moral inventory of ourselves.)
Self-Management and Recovery Training (SMART)
5. Admit wrongs (Admitted to God, to ourselves,
Recovery aims to teach persons with SUD coping strat-
and to another human being the exact nature
egies and a logical approach to thinking and acting.
of our wrongs.)
Groups utilize trained facilitators and seek to change
6. Ready self for change (Were entirely ready to
maladaptive behaviors leading to substance use. The
have God remove all these character defects.)
program lists four core tenets:
7. Seek God’s help (Humbly asked Him to
1. Building and maintaining motivation to abstain remove our shortcomings.)
2. Learning how to cope with urges 8. Become willing (Made a list of all persons
3. Managing feelings, thoughts, and maladaptive we had harmed, and became willing to make
behaviors amends to them all.)
4. Balancing immediate and long-term rewards 9. Make amends (Made direct amends to such
(monetary and enduring satisfactions people wherever possible, except when to do
so would injure them or others.)
SMART Recovery offers in-person and online
10. Daily inventory (Continued to take personal
programs, both of which are of equal effectiveness in
inventory, and when we were wrong, promptly
achieving abstinence or reducing drinking and sub-
admit it.)
stance-related problems. Similarly, Moderation Man-
11. Pray and meditate (Sought through prayer and
agement applies cognitive behavioral therapy (CBT)
meditation to improve our conscious contact
techniques in a peer-support context with a member-
with God as we understood Him, praying only
elected goal of either abstinence or moderation of
for knowledge of His will for us and power to
drinking. Moderation Management promotes self-
carry that out.)
control, responsibility, choice, rational thinking, and
12. Give it away (Having had a spiritual awaken-
insight into drinking behaviors.
ing due to these steps, we tried to carry this
message to alcoholics, and to practice these
Know This: principles in all our affairs.)
■ Mutual-support groups provide long-term social HALT (Hungry, Angry, Lonely, or Tired) is an AA
and spiritual support to recovery and augment mnemonic for triggers and emotional states that pro-
and cement the benefits of evidence-based treat- mote relapse.
ments for SUD (such as medications or psycho-
therapies) but are not and should not be con- ■ Relapse prevention strategies in 12-step–based
fused with or substituted with SUD treatment. programs use Marlatt Relapse Prevention model.
■ Twelve-step-based programs rely on active rather The model defines intrapersonal determinants of
than passive participation. The model views SUD relapse as:
■ Self-efficacy
as a chronic disease rather than a temporary state
■ Outcome expectancies (beliefs about the con-
of excessive substance use. The principles focus
on acceptance, surrendering, and actively partici- sequences one can anticipate as a result of
pating in meetings and recovery activities. substance use)
■ Craving
112 ADDICTION PSYCHIATRIC MEDICINE: A COMPREHENSIVE BOARD REVIEW
MI research finds that change in talk frequency of their substance use to consider and so-
and intensity promoted one’s commitment to lidify a commitment to change
change and increased the likelihood of suc- ■ DEARS (an overview of MI principles):
4. Planning: During the fourth stage of MI, thera- ■ Roll with resistance
lowed by asking the patient about their ideas and ■ DARN-C (an overview of change talk in
specific plans to operationalize change. The ther- order of intensity and confidence):
apist pays extra attention to “mobilizing change ■ Desire to change
help their patients set monitoring parameters to 2. Importance scaling can be used in the evoking
assess their progress continually. phase of MI to elicit change talk. The thera-
pist asked the patient to rate the importance
Motivational Enhancement Therapy (MET) refers to
of making the change on a scale from 0 to 10
a specific type of MI as a manualized structured four-
and asked them why they chose that number
session program offering patients personalized risk
instead of a lower number.
feedback. MET is frequently used in SUD research trials.
3. The four MI elements are partnership, accep-
tance (empathy), evocation, and compassion.
Know This:
4. The five Rs of MI are relevance, risks, road-
1. MI acronyms: blocks, rewards, and repetition. These refer to
■ OARS (a framework of important elements guiding principles to ensure that any assertion
guiding the therapist–patient communication): made to the patient is consistent with the de-
■ Open-ended questions encourage further sired outcomes.
elaboration and consideration 5. When initially introduced, MI utilized the
■ Affirmations that promote positive feelings stages of change as defined by the transtheo-
■ Reflections, likely the most important as- retical model of change (TTM) and viewed
pect of MI, includes simple reflections (for change as a linear, chronologic process. The
example: “So you are saying that alcohol is third edition of the MI textbook by Miller and
causing a drift in your relationship.”), dou- Rollnick (2013) presented a different behav-
ble reflections (for example: “so on the one ioral change perspective. There, it is more fluid
hand, alcohol helps you cope with stress than linear. As such, rather than following the
caused by work, and on the other hand, TTM stages of change, MI introduced the four
it is causing a drift in your relationship.”), stages of treatment: engaging, focusing, evok-
or amplified reflections (for example, after ing, and planning, highlighting how a patient
a patient tells you that they enjoy drinking can transition back and forth between stages.
alcohol, you overshoot and respond: “I However, the board examinations continue to
see, drinking alcohol has been a very posi- assess the TTM stages of change. We review
tive experience in your life and you have no the TTM stages of change below:
interest in cutting down.”) ■ Precontemplation: Patients in this stage are
■ Summary statements, extend reflections not considering change and present with ac-
and build momentum or interest in change, tive resistance. The main clinical risk during
redirect patients to more important aspects this stage is dropout from treatment.
114 ADDICTION PSYCHIATRIC MEDICINE: A COMPREHENSIVE BOARD REVIEW
■ Contemplation: Patients in this stage are to enhance the patient’s self-efficacy in maintaining
ambivalent about changing their behaviors. sobriety. The therapeutic process utilizes assertiveness
■ Preparation: Patients in this stage are taking training and role-playing. Commonly used coping
steps toward changing their behaviors and strategies include distraction, positive thought substi-
present with increasing confidence in their tution, and recall of negative consequences.
decision to change.
■ Action: Patients in this stage are overtly Know This:
modifying their behaviors and factors in the
environment that support continued sub- ■ CBT for SUD focuses on identifying high-risk
stance use. The main clinical risk during this antecedents and consequences of their use and
stage is relapse. promotes assertive communications, manag-
■ Maintenance: The goals for this stage in- ing urges and craving.
clude sustaining changes, consolidating ■ CBT is superior to other psychotherapeutic
gains, learning alternative coping strategies, modalities in patients with co-occurring anxiety
and recognizing triggers. and SUD.
■ Relapse and recycling: This stage is not in-
evitable. During this stage, patients’ main
clinical risk is to feel stuck in their old ways. Contingency Management
As such, the primary goal of treatment is For persons with SUD who are actively using, the valu-
avoiding becoming stuck and redefining ation of substance use can surpass prosocial behaviors
the relapse as an opportunity for further (such as going to work, maintaining a healthy lifestyle,
self-improvement. caring for one’s children). However, the introduction of
■ Termination: Patients in this stage have alternative reinforcers that are valued more than using
reached their ultimate goals and can exit the drugs or alcohol (such as monetary rewards, the fear
cycle of change without fear of relapse. of serious medical complications, or losing custody of
one’s children) tips the balance in favor of abstinence
and helps promote recovery. CM as therapy involves
Cognitive Behavioral Therapy positive reinforcement by rewarding patients with
The CBT model in SUD treatment relies on the same incentives for achieving their treatment goals such as
fundamental principle that thought processes, emo- abstinence. CM is not a stand-alone therapy and is typ-
tional responses, and behavioral reactions are inter- ically used in addition to other therapeutic modalities
connected. As adapted for SUD, CBT is based on the such as MI or CBT. Using the foundation of operant
Relapse Prevention model by Marlatt (and Gordon) conditioning, CM seeks to maximize the benefits of
based on social learning theory and operant condition- abstaining or reducing substance use. Incentives given
ing principles. The model posits that relapse follows to patients with nonpositive urine toxicology screens
immediate determinants of substance use (such as (drug testing, or ethyl-glucuronide for alcohol, or
triggering people, places, and things; coping strategies; saliva testing for nicotine), including monetary reward
or outcome expectancies) and covert antecedents to or other valuable prizes, serve as alternative reinforc-
substance use (factors that indirectly increase the like- ers whose value may supersede that of using drugs or
lihood of a patient using substances, including lifestyle alcohol.
factors and craving). During the course of treatment, In CM practice, patients with SUD would submit to
the therapist and patient develop a functional analy- a urine toxicology test. If they do not test positive for
sis framework identifying high-risk antecedents and any substance, they get to draw from a bucket of tick-
consequences of their use. Patients practice identify- ets associated with monetary (vouchers or equivalent
ing and avoiding high-risk situations and learn spe- valuables) prizes. In addition to the positive reinforce-
cific coping strategies (including stimulus control and ment aspect of the incentives, they serve the purpose
urge-management techniques and relapse road maps) of lessening the relative impact associated with the
13 Psychosocial Approaches to Substance Use Disorder Management 115
initial stages of recovery (including changing habits, prevention. The program focuses on recogniz-
losing substance-using peers, and experiencing pro- ing early relapse triggers and using mindfulness
tracted withdrawal symptoms). The randomness of skills to process situational cues and moderate the
the rewards further promotes abstinence. The imme- patient’s psychological responses to such cues.
diacy of the incentive delivery (directly following the ■ Community Reinforcement Approach: Multidi-
nonpositive urine toxicology findings) addresses the mensional intervention most effective for alcohol
immediate reward-seeking behavioral patterns seen in and stimulants commonly combined with CM.
persons with SUD. The focus is to create a sober living environment
Research studies have found CM to be most effec- valued more than the environment in which the
tive in treating cocaine, methamphetamine, opioid, patient actively uses drugs or alcohol. It utilizes
and cannabis use disorders and in traditionally diffi- positive reinforcement principles and teaches new
cult-to-treat patient groups, such as homeless patients and healthy coping strategies to promote sobriety.
or those with an antisocial personality disorder. How- ■ Aversion Therapy (or Aversive Conditioning Ther-
ever, discontinuation of CM often leads to reductions apy): This approach is based on classical condition-
in abstinence rates (it does not appear that the ben- ing theory by associating an unpleasant response
efits persist long when incentives are no longer pro- with substance use and ultimately creating a nega-
vided). As a result, the maximal lasting benefit is seen tive cognitive and emotional response to the unde-
in patients who participate in CM for at least 6 months. sirable stimulus (substance use). Aversive stimuli
include chemical aversion (such as with disulfi-
Other Psychotherapeutic Modalities ram and alcohol use), electrical shock, mechani-
■ Twelve-Step Facilitation: TSF is not a 12-step cal shocks, unpleasant smells or tastes, or negative
mutual-support group; it is a manualized and imagery visualization. Aversion therapy is most
structured 12-session approach that aims at effective for alcohol and cannabis use disorders.
promoting the patient’s engagement in 12-step ■ Dialectical Behavioral Therapy (DBT): When
mutual-support groups. In TSF, the therapist adapted to SUD, DBT uses the standard compo-
encourages the patient to participate in 12-step nents of DBT with a focus on emotional states
mutual-support groups and guides them through associated with abstinence and promoting moti-
the first four steps. Participation in TSF increases vation for change.
the likelihood that patients with SUD will affili- ■ Schema-Focused Therapy: This approach uses a
ate with a mutual-support group and reduce CBT-based intervention to identify maladaptive
their substance use (data supporting its effective- cognitive schemas from adverse developmental
ness in alcohol is strongest). events leading to cognitive distortions associated
■ Mindfulness-Based Interventions: The goal of with the maintenance of substance use.
treatment is to minimize stress reactivity and ■ Mentalization-Based Therapy: This cognitive-
develop affect regulation as healthy coping strat- based approach is founded on psychoanalytic
egies for cravings. This, in turn, supports the and social attachment theory. Mentalization
patient’s awareness and acceptance of their pres- refers to those mental activities that lead patients
ent situations and increases their ability to toler- with SUD to perceive and interpret behavior as
ate stressors and stress states that increase their intentional (rather than circumstantial) mental
vulnerability to relapse. One commonly used states. This allows patients to feel in control of
coping strategy is “urge surfing,” which seeks to their decision-making processes. Deficits and
ground patients in the present and address their impairments in mentalization contribute to mal-
tendency to shift their attentional focus away adaptive cognitive patterns associated with sub-
from the present. stance use maintenance.
■ Mindfulness-Based Relapse Prevention: A man- ■ Seeking Safety: This cognitive-based approach is
ualized and structured 8-week program using used for patients with comorbid SUD and post-
hybrid mindfulness and CBT approach to relapse traumatic stress disorder. The initial therapeutic
116 ADDICTION PSYCHIATRIC MEDICINE: A COMPREHENSIVE BOARD REVIEW
focus is to establish physical and psychological an undetected SUD at the primary care level, offering
safety as a foundation for recovery. those with less severe substance use simple interven-
■ Network Therapy: This approach uses the sup- tions to address simple substance use problems and
port of a group of family and peers to support refer those with more complex substance use treat-
patients with SUD in their recovery process. ment needs to specialized services. SBIRT components
include:
Know This: ■ Screening: Using screening questionnaires such
■ The Project MATCH study compared CBT, MI as the NIDA single-question screen, AUDIT-C,
(MET), and TSF for patients with alcohol use or DAST (see Chapter 1).
disorder. Its main finding is that all three psy- ■ Brief interventions (BIs): Discussed later.
chotherapeutic modalities produced roughly ■ Referral to treatment: Patients with more com-
equivalent clinical improvements. Subgroup plex substance use or SUD require specialized
analysis found that MI was superior for angry treatment services.
patients and those with lower initial motiva- BIs or early interventions have been developed to
tion. CBT was superior for patients with less address persons with various maladaptive substance
severe alcohol use disorder. TSF was superior use. BIs are meant to be easily integrated within patient
for patients with little social support for absti- consultations in primary care settings. Although there
nence and more severe alcohol use disorder. is no overarching definition of what BI is, it generally
■ In adolescents with SUD, family therapy ap-
involves:
pears to be superior to other psychotherapeu-
tic modalities for SUD treatment. ■ Assessing the quantity, frequency, and severity of
■ Stimulus control refers to modifying the envi- substance use, followed by providing the patient
ronment to minimize cues that promote relaps- with direct feedback on their health and the med-
es and maximize cues that are consistent with ico-social impact of their use. The goals of BI are
recovery. generally to reduce the amount of substance used.
■ Cognitive dissonance describes the mental ■ Establishing mutually agreeable goals to dis-
tension noted when patients hold mutually continue (typically for tobacco or illicit drugs)
exclusive beliefs. The tension resulting from or reduce (typically for at-risk alcohol use) sub-
such mutually exclusive beliefs affects patients’ stance use.
thoughts and behaviors to reduce that tension. ■ Using behavioral modification techniques to sup-
■ Mindfulness skills used in SUD include promot- port the patient in recognizing triggering cues or
ing increased awareness of bodily sensations, situations and developing coping techniques to
acceptance of the craving symptoms, and the deal with them.
uncoupling of cravings from perceived positive ■ Providing patients with reference material on self-
reinforcements relating to the reasons one uses help strategies, mutual-help groups, and the prob-
a substance (such as self-medicating). lems associated with maladaptive substance use.
■ Providing ongoing support.
The use of BI is also effective in addressing tobacco ■ Relevance: The intervention should be rele-
use, with data supporting asking smokers about their vant to a given smoking patient (such as co-
tobacco use at every visit. Studies examining BI for morbid health conditions, family, and social
illicit drug use found BI to be ineffective and costly, circumstances).
and as such, BI is not recommended to address the ■ Risks: Collaborate with patients to identify
needs of patients with illicit drug use. the risks associated with their tobacco use,
focusing on their personal circumstances.
Know This: ■ Rewards: Collaborate with patients to iden-
tify the rewards they would find in cessation.
1. The FRAMES acronym was developed to sum- ■ Roadblocks: Collaborate with patients to
marize the goals of BI: identify potential roadblocks to discontinu-
■ Feedback (on the personal risk associated
ing tobacco use.
with maladaptive substance use). ■ Repetition: Repeat the intervention on every
■ Responsibility (supporting the patient in
visit.
taking responsibility for the problem).
■ Advice (clear and explicit).
REFERENCE
■ Menu (menu of options to support the pa-
Miller, W. R., & Rollnick, S. (2013). Motivational interviewing: Help-
tient in changing their behaviors). ing people change (3rd ed.). Guilford Press.
■ Empathy (empathic approach to counsel-
REVIEW QUESTIONS
1. A rapid responder gave a 38-year-old patient D. It is obvious to both of us that you should
naloxone after an opioid overdose from using not be using cocaine, yet, you’ve been unable
cocaine laced with fentanyl. He was brought to stop. Why is that?
to the emergency room. After being monitored E. Do you think you might be using cocaine to
for a couple of hours, he requested to be dis- deal with depression or anxiety?
charged. The emergency physician expressed
3. Which of the following is an accurate application
concern about the patient’s substance use and
of CM principles for parents trying to ensure that
offered him SUD treatment referrals. The patient
their teenage daughter abstains from marijuana?
declined, stating “I am fine, nothing happened.”
A. Weekly drug screens with gas chromatogra-
Using the TTM, which stage of change best rep-
phy–mass spectrometry (GC-MS) confirma-
resents this patient’s state?
tion for positive screens.
A. Precontemplation
B. Restricting cell phone privileges every time
B. Contemplation
they catch her smoking marijuana.
C. Termination
C. Using a patient-centered approach to help her
D. Relapse and recycling
recognize how marijuana use is affecting her life.
E. Maintenance
D. Using point-of-care testing at home three times
2. You are seeing a 29-year-old female banker for a week and rewarding her with random gift cer-
cocaine use disorder. She tells you: “at first, I used tificates occasionally when she tests negative.
to take cocaine for fun; now I take it to function, E. Using point-of-care testing at home daily
to think at work, to concentrate.” She would like and rewarding her with $17 every time she
to address her cocaine use because her firm drug tests negative.
tests employees randomly but fears she will not
4. Which of the following principles of 12-step–
perform at work. Using the MI framework, what
based mutual-support groups is associated with
statement would be most helpful to respond?
increased rates of continued abstinence?
A. It sounds like you’re using cocaine for the
A. Surrendering to a higher power.
wrong reasons.
B. Commitment to participating in 12-step–based
B. Cocaine really makes you perform worse
mutual-support groups and to abstinence.
at work. If you really care for your job, you
C. Acknowledging that one is powerless over
should quit.
their substance of choice.
C. It sounds like you are interested in quitting
D. Taking inventory of oneself.
but are worried about how quitting may af-
E. Making a list of all persons one harmed and
fect your performance.
seeking to make amends.
14 PRACTICE TESTS
BLOCK 1
1. A 59-year-old man with a 10-year history of gam- she is interested in being prescribed medication
bling disorder and worsening anxiety presents to to treat OUD. The patient tells you that her use
his monthly appointment; you ask him about his began 10 years ago when she was prescribed
gambling behaviors. His reply is always the same: opioid pain medications for chronic and severe
“I am not giving up my gambling; my brother back pain. She continued using illicitly obtained
does it every day of his life and has no issues with opioid pain medication after her physician “cut
it.” Which one of the following stages of change her off” and ultimately transitioned to heroin
best describes this individual’s motivational level? use. The patient reports more than seven suicide
A. Action attempts by intentional opioid overdose but de-
B. Contemplation nies any current suicidal thoughts or depression.
C. Precontemplation The patient is hopeful that treatment will im-
D. Maintenance prove her condition. During your evaluation, the
E. Preparation patient appeared to be in significant distress as
a result of opioid withdrawal. What is the most
2. A 72-year-old man with leukemia is currently under-
appropriate next step?
going chemotherapy and is emaciated from weight
A. This patient should be sent to the psychiat-
loss as a result of chemotherapy-related nausea and
ric emergency room for an evaluation and
vomiting. You discuss the use of cannabis-related
consideration for inpatient hospitalization
compounds for his symptoms, but he expresses
given her suicide history.
that he tried smoking cannabis before and disliked
B. Prescribe clonidine, ibuprofen and loper-
the “high.” Which of the following compounds is
amide for her withdrawal symptoms and
both U.S. Food and Drug Administration (FDA)-
schedule her for a follow-up appointment in
approved to treat his chemotherapy-related nausea
one week.
and would minimize unwanted side effects?
C. Perform a naloxone challenge test. If no
A. Dronabinol
evidence of withdrawal is seen, prescribe
B. Nabilone
oral naltrexone in preparation for starting
C. Rimonabant
long-acting injectable naltrexone.
D. Medical marijuana
D. Send a patient to the emergency room to be
E. Cannabidiol (CBD)
admitted for a medically supervised opioid
3. A 52-year-old woman with severe opioid use detoxification.
disorder (OUD) and intravenous heroin use is E. Assess the severity of her opioid withdrawal
referred to your office. The patient tells you that and begin buprenorphine.
119
120 ADDICTION PSYCHIATRIC MEDICINE: A COMPREHENSIVE BOARD REVIEW
use disorder. Which of the following mechanisms 13. A 27-year-old college student is brought to the
is most relevant to disulfiram’s use in treating co- emergency room following an accidental opioid
caine use disorder? overdose. He is given naloxone and is held for a
A. Inhibition of aldehyde dehydrogenase few hours to stabilize in the emergency room.
B. Inhibition of alcohol dehydrogenase The treating physician counseled this patient
C. Disulfiram’s metabolite, ditiocarb, interferes about the risks associated with drug use, rec-
with the ubiquitin-proteasome system ommended SUD treatment, and offered refer-
D. Inhibition of monoamine oxidase ral options. The patient was appreciative of the
E. Inhibition of dopamine-beta-hydroxylase treatments he received in the emergency room
but declined the referrals, stating that his drug
11. What is the theoretical basis of CBT’s psychological
use is rare and that he will never overdose again.
theory as it applies to the treatment of SUD?
Based on the transtheoretical model of change,
A. CBT involves processing unconscious drives
what “stage of change” is this patient?
and understanding defense mechanisms as a
A. Precontemplation.
tool sobriety skill.
B. Contemplation.
B. CBT is a collaborative and client-centered
C. Preparation
approach that aims at exploring and resolv-
D. Action
ing patients’ ambivalence about the role of
E. Maintenance
substances in their life in a nonconfronta-
tional approach. 14. A 19-year-old man with OUD presents to your
C. CBT conceptualizes the interconnections office for follow-up. During your appointment,
between thought processes, emotional you help the patient plan for high-risk situations
responses, and behaviors reactions in drug and build avoidance skills to reduce the likeli-
and alcohol use. hood of relapse after 2 months of sobriety. As
D. CBT utilizes alternative reinforcers that are his physician, you are utilizing which of the fol-
valued more than using drugs or alcohol in lowing therapy modalities?
order to tip the balance in favor of absti- A. MET
nence and promote recovery. B. CBT
E. CBT aims to minimize stress reactivity and C. Dialectical behavioral therapy
develop affect regulation as healthy coping D. TSF
strategies for cravings. E. Supportive therapy
12. A 42-year-old woman presents to your office after 15. A 16-year-old female high school sophomore
her partner found her disoriented at home. On presents to your office with her mother. You have
examination, she is bradycardic, in respiratory noticed a drastic change in her appearance and
distress and disoriented. She has been treated for behavior, in addition to the development of sig-
pain after she injured her back in a car accident nificant tooth decay. The mother expressed that
over a year ago. According to her partner, she has she has not been herself, and you wonder about
been increasing her dose and buying extra pills methamphetamine use. Which of the following
from a friend. What should be administered to compounds holds promise as a treatment for
this patient so that her symptoms do not worsen? methamphetamine use disorder?
A. Clonidine A. Paroxetine
B. Guanfacine B. Aripiprazole
C. Naloxone C. Bupropion
D. Naltrexone D. Depakote
E. Methadone E. Carbamazepine
122 ADDICTION PSYCHIATRIC MEDICINE: A COMPREHENSIVE BOARD REVIEW
16. Flumazenil can reverse benzodiazepine and non- 19. According to a recent study out of Colorado, can-
benzodiazepine hypnotic (Z-drug) overdose, but nabis-related emergency room visits were found
not barbiturate or alcohol poisoning. Which of to have increased threefold following marijuana
the following best explains its selective therapeu- legalization. A disproportionately large number
tic effect? of these visits was because of the consumption of
A. Flumazenil binds directly onto benzodiaze- cannabis edibles, despite the fact that edibles make
pines and Z-drugs and renders them inactive. up only a tiny fraction of cannabis purchases in the
B. Flumazenil binds to the benzodiazepine- state. Which of the following best explains why ed-
binding site on the GABA-A receptor. ibles are responsible for a disproportionately large
C. Flumazenil binds to the alpha-4 and number of cannabis-related emergency room visits
-6 subunits of the GABA-A receptor. compared to the smoked route of administration?
D. Flumazenil binds to the GABA-B receptor. A. Edibles are frequently laced with other
E. Flumazenil noncompetitively inhibits the substances.
GABA-A receptor. B. Edibles are more likely to cause gastrointes-
tinal irritation and bleeding.
17. Which of the following structures is delta- C. Cannabis consumed via the oral route has
9-tetrahydrocannabinol? higher bioavailability.
D. Cannabis consumed via the oral route has
A. B.
lower bioavailability.
N OH
H E. The onset of action of edibles is longer and
H
harder to predict.
O 20. A 31-year-old college student with a history of
stimulant use disorder presents to the emergency
C. H D. department with recurrent diarrhea and fever. A
N NH2
O urine drug screening (UDS) test is positive for
CH3 cocaine, and complete blood count shows an
O
increased white blood cell count but negative
blood cultures. The patient’s fever resolves with-
E. H3C O out the use of antibiotics. Use of cocaine adul-
O terated with which of the following substances is
O H most likely to explain this patient’s presentation?
O H A. Lansoprazole
N CH
3
H 3C O B. Levamisole
C. Lamotrigine
18. Over the past several years, a 10-year-old boy D. Levamisole
with Lennox-Gastaut syndrome has tried multi- E. Loperamide
ple antiepileptic medications but has continued 21. The patient in Question 20 decided to start
to have daily poorly controlled seizures. Which treatment for cocaine use disorder. Which of the
of the following compounds was recently FDA- following medications has the most evidence
approved for the treatment of seizures in Len- of possible benefit for increasing rates of absti-
nox-Gastaut syndrome? nence in cocaine use disorder?
A. Dronabinol A. Duloxetine
B. Nabilone B. Naltrexone
C. Rimonabant C. Risperidone
D. Medical marijuana D. Topiramate
E. CBD E. Sertraline
14 Practice Tests 123
22. A 41-year-old White man presents into your of- he underwent a routine physical and UDS test.
fice complaining of financial difficulties. He has His UDS test returns positive for THC and is
been preoccupied with getting more gambling confirmed by gas chromatography–mass spec-
money and needing to gamble with increasing trometry (GC-MS). He contests these results
amounts of money to get the same “reaction.” and states that his brother was smoking canna-
He is asking you for “something” to decrease bis in his apartment 2 days before he submitted
his urges. Which of the following medications a sample, although he himself does not smoke
is most appropriate for addressing the patient’s cannabis. He also questions whether the result
concerns? may be a false-positive result, as he takes several
A. Propranolol medications. Which of the following is most ac-
B. Pregabalin curate regarding his UDS test result?
C. Benzoylmethylecgonine A. Passive inhalation of cannabis smoke likely
D. Buprenorphine resulted in a positive test.
E. Naltrexone B. It is likely a false positive from taking
pantoprazole.
23. Which of the following best explains why canna-
C. It is likely a false positive from taking
bis is legally available in many U.S. states while
ibuprofen.
being illegal under U.S. federal law?
D. It is likely a false positive from taking
A. Cannabis is categorized as schedule I, but
bupropion.
the judicial branch of the federal govern-
E. Passive inhalation of cannabis smoke is
ment issued a memorandum indicating the
unlikely to result in a positive test.
states would not be prosecuted for having a
state-legal cannabis enterprise. 25. A 15-year-old girl is brought to the emergency
B. Cannabis is categorized as schedule I but is room by her parents. At her friend’s birthday
permissible on a state-by-state basis if the party, she reportedly ate an entire bag of gum-
state law conflicts with federal law in favor my bears, not knowing that they were cannabis
of legalizing marijuana. edibles. On presentation, she is agitated, hallu-
C. Cannabis was rescheduled from schedule I cinating, and keeps repeating that she is going
to schedule II, putting it in the same catego- to die. She is tachycardic and hypertensive but
ry as substances considered to have medical not in respiratory distress, and her oxygen satu-
use but with high abuse potential, such as ration is normal. Which of the following best ac-
oxycodone and morphine. counts for why cannabis overdose does not lead
D. Cannabis was rescheduled from schedule I to respiratory depression?
to schedule III, putting it in the same catego- A. CB1 receptors are expressed in very low
ry as substances considered to have medical density in the brainstem.
use but moderate or low abuse potential, B. CB1 receptors are expressed in very high
such as buprenorphine. density in the hippocampus.
E. Cannabis was rescheduled from schedule I to C. CB2 receptors are expressed in very high
schedule IV, putting it in the same category density in the hippocampus.
as substances considered to have medical D. CB2 receptors are expressed in very low
use but lower abuse potential compared with density in the brainstem.
schedule III, such as the benzodiazepines. E. Endocannabinoids effectively regulate
respiratory drive even in the presence of
24. A 41-year-old man was recently hired as a fed-
exogenous cannabinoids.
eral employee. He has a history of depression,
gastric reflux disease, and low back pain, for 26. A 34-year-old man with severe OUD and comorbid
which he takes bupropion, pantoprazole, and major depressive disorder comes into the clinic for
ibuprofen. As part of his onboarding process, evaluation and consideration for treatment with
124 ADDICTION PSYCHIATRIC MEDICINE: A COMPREHENSIVE BOARD REVIEW
buprenorphine. The patient tells you that he has has increased his lorazepam dose on several occa-
been injecting heroin at least four times daily on sions due to an increase in his anxiety. During your
and off for the past 15 years. He acknowledges us- last visit, he was frustrated that you refused to in-
ing illicitly obtained buprenorphine on multiple oc- crease his lorazepam dose. Which of the following
casions to treat his OUD. During your evaluation, is most accurate regarding his management?
the patient admits that his father died of an opioid A. The patient should be tapered off lorazepam
overdose 1 month ago, and as a result, he has been due to evidence that he has developed a
feeling very depressed. He adds that 1 month ago, substance use disorder.
he attempted suicide by intentionally overdosing B. The patient should be tapered off lorazepam
on heroin. Which of the following is the most ap- due to evidence that it is ineffective for his
propriate next step of this evaluation? anxiety disorder.
A. Order stat laboratory tests for HIV and C. The patient’s current dose of lorazepam may
HCV. be too low to adequately treat his anxiety
B. Assess the severity of the patient’s disorder.
depression and current suicidal thoughts. D. The patient’s dose of lorazepam should be
C. Inform the patient that he is not eligible left as is, and he should be encouraged to
for treatment with buprenorphine because address these issues in therapy.
he has a history of obtaining it from illicit E. The patient should be referred to the hospital
sources. for an inpatient psychiatric hospitalization.
D. Inform the patient that he is not eligible for 29. A 57-year-old White man is engaged in psycho-
treatment with buprenorphine because of therapy to address the high cost of his gambling
his recent suicide attempt. behavior. During therapy, he began understanding
E. Prescribe fluoxetine. how each visit to the casino and time spent with
27. A 48-year-old man is currently admitted to people on the “tables” briefly improves his mood.
inpatient rehabilitation for sedative/hypnotic This behavioral pattern can be best explained by
(benzodiazepine) and alcohol use disorder. He which of the following learning theories?
completed detoxification 1 month ago and is com- A. Phobias of objects theories
plaining of persistent depressed mood, poor appe- B. Operant conditioning
tite, and insomnia. Which of the following would C. Classical conditioning
be the next best step in his treatment? D. Albert experiment
A. Initiate naltrexone for treatment of E. Reductionism
prolonged alcohol withdrawal symptoms. 30. Which of the following is true regarding sex dif-
B. Initiate acamprosate for treatment of ferences in alcohol use disorder?
prolonged alcohol withdrawal symptoms. A. Women have a higher prevalence of alcohol
C. Readmit to detoxification owning to use disorder.
persistent withdrawal symptoms. B. Women with alcohol use disorder have a
D. Perform a psychiatric evaluation for co- higher risk of negative medical sequelae.
occurring psychiatric disorders. C. Women have a higher prevalence of binge
E. Reassure the patient that these symptoms will drinking.
likely self-resolve without any intervention. D. Women are more likely to begin drinking at
an earlier age.
28. A 28-year-old man with a generalized anxiety dis-
E. Women are less likely to have a co-occurring
order on venlafaxine 225 mg daily, lorazepam
psychiatric disorder.
0.5 mg twice a day, and enrolled in CBT presents
for a follow-up appointment. He has no other 31. A 37-year-old transgender woman presents to
medical problems. His mother was recently diag- treatment for OUD. During your session, she
nosed with cancer 2 months ago. Since then, he shares that what makes it hard to quit is that
14 Practice Tests 125
using heroin “is the best feeling in the world.” 34. A 54-year-old man with a long history of depression,
She states that nothing she has experienced can anxiety, and excessive alcohol use presents to your
match the feeling of using heroin. Which two office reporting poor mood and anxiety symptoms
brain regions are most fundamental in mediat- in the context of ongoing drinking. The patient tells
ing the experience she is describing? you that he is not interested in cutting down on the
A. Ventral tegmental area (VTA), nucleus ac- drinking because: “Honestly, I know it’s bad, but it’s
cumbens (NAc) the only thing that helps when I’m feeling anxious.”
B. NAc, prefrontal cortex Which of the following statements is consistent with
C. VTA, prefrontal cortex a motivational interviewing approach?
D. Amygdala, prefrontal cortex A. A lot of people do not understand the
E. Amygdala, NAc relationship between alcohol and mood or
anxiety disorder. Let me give you the facts.
32. A 59-year-old man with alcohol use disorder
B. Let’s talk about the emotional states you are
finds that when he stops using alcohol, he feels
in right before you start drinking.
anxious, irritable, and markedly dysphoric.
C. I will make you a deal. If you successfully
Which of the following compounds is dimin-
quit drinking, I will consider prescribing you
ished in the amygdala during withdrawal states,
lorazepam to manage your drinking.
leading to his symptoms?
D. It is obvious to me that you were drinking
A. Dynorphin
is worsening your mood and anxiety symp-
B. Norepinephrine
toms. Let’s talk about your past traumatic
C. Neuropeptide Y
experiences that cause you to act in such
D. Corticotrophin-releasing factor (CRF)
self-destructive ways.
E. cAMP response element-binding protein
E. It sounds like you are worried that your
(CREB)
anxiety will worsen if you stop drinking while
33. A 55-year-old man with a history of alcohol at the same time recognizing the need to ad-
use disorder has been in remission for the past dress your alcohol use.
20 years. He attends Alcoholics Anonymous
35. A family would like to institute a recovery plan
(AA) meetings weekly and maintains contact
to help their teenage daughter discontinue mari-
with his sponsor. After his wife unexpectedly
juana use. Which of the following approaches is
passes away, he becomes distraught and grief-
most consistent with contingency management
stricken, and relapses to heavy alcohol use.
(CM) principles?
Which of the following brain regions and neu-
A. Providing her with ample psychoeducation
rocircuitry is most important in mediating his
about the negative long-term consequences
reinstatement to alcohol use?
of marijuana use
A. Activation of corticotropin-releasing factor
B. Helping her recognize distorted thoughts
and norepinephrine in the amygdala and
and negative emotions that precede her
VTA
marijuana use
B. Activation of glutamatergic circuits involving
C. Weekly drug testing at the nearest labora-
the medial prefrontal cortex and ventral
tory and grounding her any time she tests
striatum
positive for marijuana
C. Basolateral amygdala with feed-forward
D. Weekly drug testing at the nearest labora-
inhibition of the prelimbic prefrontal cortex
tory and rewarding her with money any time
D. Activation of dopaminergic cells in the VTA
she tests negative for marijuana
that project to the NAc
E. Weekly drug testing at home and rewarding
E. Inhibition of GABA-ergic interneurons in the
her with money any time she tests negative
VTA, which indirectly increases dopaminer-
for marijuana
gic neurotransmission in the NAc
126 ADDICTION PSYCHIATRIC MEDICINE: A COMPREHENSIVE BOARD REVIEW
36. Administration of which neuropeptide has been nitive and behavioral changes conducive to
found to modulate the drug-seeking behavior change and recovery.
for methamphetamines? C. The TC staff are neutral experts in SUD.
A. Testosterone D. The daily social activities and chores are an
B. Estrogen integral part of the TC and play an impor-
C. Tyrosine tant treatment role. TCs that incorporate
D. Oxytocin gardening and dishwashing have better
E. Phentermine outcomes than those that do not.
E. TCs are only effective for persons with SUD
37. A 67-year-old patient with a long history of
who are not homeless because the residen-
gastrointestinal adhesions and chronic renal
tial aspect of the program would be ineffec-
disease presents to the emergency room with
tive for those who are homeless.
stomach pain. The patient was given medica-
tion for pain. You noticed that he started having 40. The police bring a male–female couple in their
a seizure minutes later. Which of the following 20s with methamphetamine intoxication to the
opioid analgesic metabolites could have caused emergency room. The woman’s mother pro-
this patient presentation? vides a history that the couple has been smoking
A. 6-ß-naltrexone metabolite methamphetamine for close to 2 years. Which
B. Norpethidine of the following is most accurate regarding the
C. Codeine-6-glucuronide gender differences in methamphetamine use
D. Morphine-3-glucuronide and response to treatment?
E. 6-Acetylmorphine A. Women show higher dropout rates and
more use during treatment than men.
38. Substance use can alter gene expression through
B. The average age of initiating methamphet-
cellular signaling pathways that change the ex-
amine use among men is 25 years old.
pression of transcription factors. Which of the
C. Women initiate methamphetamine use at
following transcription factors increases rapidly
later ages compared to men.
in the NAc with cocaine use and leads to dynor-
D. Men tend to have more severe methamphet-
phin upregulation?
amine use than females.
A. Delta-FosB
E. Men show higher dropout rates and more
B. Neuropeptide Y
use during treatment than women.
C. CREB
D. Myocyte-enhancing factor-2 41. A 23-year-old woman with methamphetamine
E. Nuclear Factor κB use disorder presents to the emergency room and
is agitated, aggressive, and reporting visual and
39. Therapeutic communities (TC) are a type of
auditory hallucinations. She was admitted to the
residential treatment facility that is traditionally
medical floor for observation and treatment. Af-
longer term (typically 6–12 months) and utilizes
ter 70 hours of admission, the patient becomes
every aspect of the residential community as a
more restless, with vital signs showing heart rate
treatment method and means to provide ser-
of 110 and a blood pressure of 158/90 mm Hg.
vices. What does “residential community as a
Which of the following diagnoses is most likely to
treatment method” mean?
account for her change in clinical status?
A. Having all residents of the TC living in the
A. Alcohol withdrawal
same building is the only way to get them to
B. Amphetamine-induced psychosis
participate in group therapy sessions.
C. Benzodiazepine withdrawal
B. Persons with SUD learn to utilize the social
D. Lithium intoxication
activities and chores to model change and
E. Manic episode
rely on their new community to practice cog-
14 Practice Tests 127
42. A 33-year-old man was brought to urgent care into a parked car while driving to work in the
by his girlfriend. On examination, his tempera- morning. Which of the following is true regard-
ture is 104°F, heart rate is 110 beats per minute, ing this incident?
blood pressure is 154/90 mm Hg, and his physi- A. The PCP should have prescribed a short-
cal examination is notable for tachypnea and acting benzodiazepine instead of zolpidem.
muscle rigidity. He has no significant medical or B. The patient should have been started on
surgical history. He is started on supplemental long-acting zolpidem instead of regular
oxygen via nasal cannula, and he dies 1 hour lat- zolpidem.
er. In this case, methamphetamine could have C. The patient have been counseled that driving
caused his death by which complication? following zolpidem use in the night prior is
A. Weight gain not recommended.
B. Hypotension D. The patient most likely co-ingested zolpidem
C. Hyperthermia with another substance such as alcohol, as
D. Hypervolemia appropriate use of zolpidem rarely results in
E. Anorexia adverse events.
E. The patient was started on higher than the
43. A 34-year-old woman with a history of border-
recommended dose of zolpidem.
line personality disorder, generalized anxiety dis-
order, and tobacco use disorder presents to your 45. Which of the following is true regarding
office for psychiatric evaluation owning to recent benzodiazepine-related overdose deaths?
events causing overwhelming anxiety. She is re- A. Benzodiazepine overdose deaths have been
questing lorazepam. She has made two-lifetime on the decline over the past two decades in
suicide attempts and has multiple prior psychi- the United States.
atric hospitalizations. She is currently taking la- B. Alcohol is involved in the majority of benzo-
motrigine, fluoxetine, and trazodone. Which of diazepine overdose deaths.
the following is essential in your consideration C. Opioids are involved in the majority of
to prescribe or not prescribe a benzodiazepine? benzodiazepine overdose deaths.
A. Benzodiazepines increase the odds of a D. Benzodiazepines are involved in the majority
future suicide attempt. of opioid overdose deaths.
B. Prescription of a benzodiazepine is contrain- E. Most benzodiazepine overdose deaths occur
dicated because she already has an SUD. with benzodiazepines as the sole substance.
C. Benzodiazepines are the most effective
46. A 55-year-old man with a generalized anxiety dis-
treatments for generalized anxiety disorder.
order has been on clonazepam 1 mg three times
D. Lorazepam has an adverse drug interaction
daily for the past 15 years. He wishes to taper off
with one of the medications she is currently
owning to concerns about his long-term cognitive
prescribed.
performance. When he tried to stop on his own
E. Patients who come to your office requesting
before, he experienced intolerable anxiety and in-
any specific medication are generally drug-
somnia. Which of the following is the best strat-
seeking.
egy for helping him taper off this medication?
44. A 35-year-old woman with a history of insomnia A. Initiate an antiepileptic such as valproic acid
and no other medical problems presents to her or carbamazepine, then taper clonazepam
primary care doctor. After multiple medication over the course of days.
trials with noncontrolled medications, her pri- B. Perform a taper with clonazepam over the
mary care doctor prescribes zolpidem 10 mg to course of months.
take at night. She begins taking it as directed, C. Transition the patient to diazepam and
and several days later, she crashes her vehicle taper over the course of days.
128 ADDICTION PSYCHIATRIC MEDICINE: A COMPREHENSIVE BOARD REVIEW
BLOCK 2
1. Multiple psychotherapeutic approaches have his cannabis use and has been vaping high-po-
been found to be effective in treating SUDs. tency cannabis and smoking several joints per
Which of the following statements is accurate? day. He presents to the emergency room because
A. As applied to SUD, CBT aims to establish of severe nausea and repeated vomiting for the
a functional analysis framework identifying past 2 weeks. He undergoes a full workup, which
high-risk antecedents and consequences of shows acute renal failure from dehydration but
their use. is otherwise negative. Which of the following is
B. Mindfulness-based approaches involve es- most accurate regarding his likely condition?
tablishing discrepancies between behaviors, A. It is uncommon in individuals who have
values, and consequences. been long-term daily users of cannabis.
C. Motivational interviewing is not a stand- B. The majority of people presenting to the
alone therapy and is typically used in addi- emergency room with this condition are
tion to other therapeutic modalities. women.
D. TSF describes the psychotherapeutic C. Ondansetron can provide effective symp-
benefits patients describe after attending tomatic relief.
AA meetings. D. Capsaicin can provide effective symptomatic
E. Mentalization-based therapy uses the relief.
support of a group of family and peers to E. Cold baths can provide effective symptom-
support patients with SUD in their recovery atic relief.
process.
5. Which of the following compounds is an endog-
2. A 25-year-old woman of East Asian descent enous ligand for the CB1 receptor?
reports experiencing facial redness, nausea, A. Dynorphin
and discomfort when she consumes even small B. Enkephalin
amounts of alcohol. Her reaction is likely due to C. CBD
a genetic variant in the enzyme ________ that D. Anandamide
metabolizes __________ more slowly. E. Neuropeptide Y
A. ALDH; acetate
6. A 21-year-old man with a history of depression
B. ADH; acetate
and daily cannabis use for the past year has re-
C. ALDH; acetaldehyde
cently escalated his cannabis use over the past
D. ADH; acetaldehyde
month and is now smoking several blunts a day.
E. ADH; ethylene glycol
During his last appointment with his psychia-
3. Approximately which of the following percent- trist, he reported that he often feels uncomfort-
ages of people can be expected to meet crite- able and paranoid on the subway and hears peo-
ria for a methamphetamine use disorder in the ple talking about him and calling him derogatory
United States? names. In the context of cannabis use, specific
A. 3% polymorphisms in which of the following genes
B. 1% increases the likelihood of experiencing symp-
C. Less than 0.5% toms such as those described by the patient?
D. Between 0.5% and 1% A. AKT1
E. 5% B. ADH2
C. CNR1
4. A 26-year-old man with a history of low back pain
D. FAAH
and obesity has been a daily cannabis user since
E. OPRM1
high school. In the past month, he has increased
130 ADDICTION PSYCHIATRIC MEDICINE: A COMPREHENSIVE BOARD REVIEW
7. Which of the following best describes the differ- 10. A 34-year-old woman is brought in by ambulance
ences between Cannabis indica and Cannabis sativa? to the emergency department with a suspected
A. The male plants of C. indica have greater overdose. She is unresponsive, cyanotic, and
THC content than the female plants. bradypneic. On examination, her skin is cold and
B. C. indica is typically grown by farmers for her pupils are constricted. Paramedics tried using
hemp fiber and has negligible THC content. 4 mg of intranasal naloxone with no response. An
C. C. indica has a higher ratio of CBD to THC. overdose of which of the following substances is
D. C. indica is federally legal in the United the most likely cause of her presentation?
States. A. Acetaminophen
E. C. indica has the highest cannabis potency in B. Buprenorphine
the leaves and stems and not the flowering C. Doxepin
tops. D. Fentanyl
E. Clonazepam
8. A 35-year-old woman with no significant medi-
cal problems has been vaping both nicotine and 11. A methadone program has tabulated the meth-
THC products for the past 3 years. She began adone doses of all of the patients in the clinic
experiencing progressive dyspnea, cough, and with OUD. What is the right measure to estimate
pleuritic chest pain over the course of a month. how much scores vary from the mean value?
Upon presentation to the emergency room, A. Median
her oxygen saturation was 85% and chest CT B. Mode
showed ground glass opacities in her lungs bi- C. Standard deviation
laterally. Workup for infectious etiologies was D. Standard error
negative. She is admitted to the intensive care E. Range
unit and treated with prednisone with signifi-
12. An 18-year-old pregnant woman presents to the
cant improvement. Which of the following was
clinic for a prenatal checkup. She is enrolled in
most likely responsible for her condition?
a methadone maintenance program for OUD.
A. Carbon monoxide in THC aerosol
Which of the following statements best explains
B. Carbon monoxide in nicotine aerosol
the prevalence of psychiatric disorders among
C. Vitamin E acetate (VEA) in THC cartridges
pregnant women with OUD?
D. VEA in nicotine cartridges
A. Anxiety disorders are the most common
E. Propylene glycol in THC cartridges
psychiatric disorders seen in this group.
9. A 27-year-old woman with alcohol use disorder B. Mood disorders are the most common psy-
and a history of cocaine and heroin use by injec- chiatric disorders seen in this group.
tion is referred to your methadone clinic. On meth- C. Pregnant women with an OUD do not suffer
adone induction, she discontinues all illicit opioid from mood disorders.
use but continues to drink alcohol excessively and D. Pregnant women with an OUD do not suffer
inject cocaine. Laboratory testing reveals that she from anxiety disorders.
is HIV positive. When is the most appropriate time E. Schizophrenia is the most common psychi-
for her to begin treatment for HIV using highly ac- atric disorder seen in this group.
tive antiretroviral treatment (HAART)?
13. A methadone clinic wants to estimate how many
A. Only if she develops Kaposi sarcoma
patients are receiving false-positive results on
B. Only if she just continues all injection drug
urine toxicology screens for illicit opioids. What
use
statistical measure should you use?
C. Only if she discontinues all alcohol use
A. Sensitivity
D. When her CD4 count is less than 200
B. Positive predictive value
E. Immediately
C. Specificity
14 Practice Tests 131
21. Based on animal models involving opioid self- 24. A 31-year-old man is brought to urgent care by
administration, which one of the following his partner owning to increased paranoia and
statements is true regarding the reinforcing ef- decreased appetite. On physical examination,
fects of opioids? his pupils are dilated, and he appears suspi-
A. The stress responsive hypothalamic pituitary cious. His partner reports that he was recently
adrenal axis has been implicated. diagnosed with adult attention-deficit disorder
B. Patients experience hypocortisolemia or and was prescribed amphetamine for it. Which
other disturbances of the hypothalamic– one of the following is the most appropriate
pituitary–adrenal system. treatment for an amphetamine overdose?
C. Opioids have a significant impact on rein- A. Bicarbonate
forcement mechanisms in the VTA. B. Ammonium chloride
D. Contributions to reward focused on the C. Atropine
serotonergic projection to the VTA. D. Naltrexone
E. No disturbances of the hypothalamic– E. Flumazenil
pituitary–adrenal system have been
25. A 43-year-old man with a history of diabetes and
reported.
tobacco use comes to the clinic with the desire
22. A 19-year-old woman with a 16-month history to quit smoking. He has tried to stop on his own
of OUD and hepatitis C delivered her second but continues to smoke and experiences strong
child prematurely and presents to your office. At nicotine cravings. What intervention has demon-
the time of delivery, she received 30 mg of meth- strated a superior efficacy in smoking cessation?
adone. She was negative for HIV. There were no A. Recommend bupropion only
complications during delivery. She is interested B. Recommend varenicline and behavioral
in breastfeeding her newborn and is seeking your modification
advice. What will you advise this patient to do? C. Recommend NRT only
A. She should not breastfeed immediately. D. Behavioral modification only
B. She should not be recommended to use E. Recommend mirtazapine
methadone.
26. A 31-year-old man and his wife have become
C. She should start breastfeeding immediately;
increasingly distant because of her disappoint-
the benefits outweigh the risks.
ment and hurt over his online pornography use.
D. She should be started on naltrexone.
They have now experienced numerous financial
E. She should wait at least 6 months to
consequences because he has reached the credit
breastfeed.
limit of two of their credit cards paying for por-
23. A 21-year-old woman is brought to urgent care nography. He is not sleeping well, and it has
with heart palpitations, dilated pupils, and chest been hard for him to go to work in the morning.
pain, which started approximately 30 minutes ago Which medications have demonstrated efficacy
while watching the Super Bowl game at a friend’s in the treatment of sexual addiction?
house. She appears guarded and detached from A. Naltrexone
her surroundings. Her urine drug screen is nega- B. Tramadol
tive, except for cannabinoids. Which one of the C. Amphetamine
following diagnoses would be the most appropri- D. Sertraline
ate for this patient at this time? E. Buspirone
A. Cocaine-induced anxiety
27. Which of the following is accurate about the ef-
B. Alcohol withdrawal
fects of chronic excessive alcohol use?
C. PCP syndrome
A. Brain imaging studies reveal frontal lobe
D. Cannabis-induced anxiety
hypertrophy and neuropsychological testing
E. Alcohol intoxication
reveal cognitive deficits.
14 Practice Tests 133
B. Neuropsychological testing of cognitive 30. Several medications are approved by the FDA for
function does not reveal any impairments. the treatment of OUD. Which of the following is
C. No changes are noted on brain imaging and an FDA-approved medication for the treatment
neuropsychological testing reveal cognitive of co-occurring PTSD and OUD?
deficits. A. Buprenorphine
D. Brain imaging studies revealed a loss of B. Naltrexone
brain tissue and cognitive impairments are C. Methadone
noted on neuropsychological testing. D. Baclofen
E. Elevated concentration of serotonin is found E. None of the above
in the cerebrospinal fluid.
31. A 19-year-old man is brought to the emergency
28. Which of the following personality disorders has department by a friend after being found acting
the highest rates of comorbid SUDs? bizarrely and with a flat facial expression. His vital
A. Borderline personality disorder signs appear to be within normal limits, but he is
B. Obsessive compulsive personality disorder not moving while sitting on the stretcher. You start
C. Dependent personality disorder asking a question, and he immediately becomes
D. Schizoid personality disorder hostile and attempts to assault you. This patient
E. Antisocial personality disorder most likely ingested which of the following drugs?
A. Heroin
29. A 38-year-old man with a history of major de- B. Mescaline
pressive disorder, cannabis and alcohol use dis- C. Methamphetamine
order, obesity, and type 2 diabetes presents to D. Cocaine
your clinic. He takes sertraline, and his alcohol E. PCP
use disorder is currently in remission. He contin-
32. A 24-year-old man is brought to the emergen-
ues to smoke cannabis daily, and he reports that
cy room by friends after ingesting an unknown
whenever he stops smoking, he feels depressed
quantity of gamma-hydroxybutyrate (GHB) dur-
and anxious and is unable to sleep or eat. He
ing a party. His friends tell you that they saw him
asks you if any treatments can help him stop
vomiting multiple times and having convulsions,
smoking cannabis. Which of the following state-
and then observed his breathing become shal-
ments is most accurate regarding his treatment
options? low and labored. On arrival, he is immediately
intubated and restrained. Which of the follow-
A. Rimonabant is a CB1 antagonist used for
ing clinical signs or symptoms is commonly ob-
treatment of cannabis use disorder and can
served in GHB overdose?
help him with weight loss.
A. Psychosis
B. He is experiencing a worsening of symptoms
B. Bradycardia
related to major depressive disorder, and his
C. Mydriasis
dose of sertraline should be increased.
D. Tachypnea
C. There are no FDA-approved medications or
E. Hypertension
evidence-based psychosocial interventions
for cannabis use disorder. 33. A 54-year-old woman with a history of schizoaf-
D. There are off-label medications that can be fective disorder and cocaine, alcohol, and to-
prescribed to alleviate cannabis withdrawal bacco use disorders presents to the emergency
symptoms. room with chest pain and is admitted for myo-
E. Genetic testing can be helpful to determine cardial infarction. Her alcohol level on admis-
the best medication approach for treating sion is 102, and her urine drug screen is positive
his cannabis use disorder. for cocaine and negative for all other substances.
134 ADDICTION PSYCHIATRIC MEDICINE: A COMPREHENSIVE BOARD REVIEW
She is placed on a Clinical Institute Withdrawal A. She has been investing more time looking for
Assessment for Alcohol (CIWA) protocol, which doctors to prescribe the medication.
is discontinued on day 3 after she did not exhibit B. Larger doses of opioids are needed for
any signs or symptoms of alcohol withdrawal. analgesia.
On day 5, her heart rate and blood pressure in- C. The patient becomes anxious before the next
crease, becoming more agitated and confused. dose.
Which of the following would best explain her D. The patient’s constipation has improved.
change in clinical status? E. The patient was prescribed naloxone.
A. Alcohol withdrawal
37. A 17-year-old girl is brought to the physician
B. Sedative withdrawal
after her mother learned that she was having
C. Cocaine withdrawal
sexual intercourse with various partners and us-
D. Opioid withdrawal
ing cocaine. She does not use condoms or other
E. Onset of a psychotic episode
contraception. Of the following therapies listed,
34. The GABA-A and GABA-B receptors are the pri- which type of treatment would use incentives to
mary receptor targets for many medications and encourage sobriety in this patient?
drugs of abuse. Which of the following com- A. Behavioral couples therapy
pounds does not exert its primary effect through B. Dialectical behavioral therapy
the GABA-A or GABA-B receptor? C. CBT
A. Baclofen D. Contingency management
B. GHB E. MET
C. Propofol
38. A 21-year-old man presents to your office with
D. Gabapentin
lower extremity pain for which you prescribe ac-
E. Phenobarbital
etaminophen. He then returns to your office 2 days
35. A study is assessing the activity of a new opioid later complaining of gastrointestinal symp
inhibitor in patients with chronic OUD. All pa- toms, lacrimation, and yawning. His blood pres
tients enrolled in the study are informed that sure is high, and his pupils are dilated. Which
they would be treated with the inhibitor. They of the following substances is responsible for his
are assigned to successive dose cohorts of 20 to presentation?
50 mg/day of the medication. Approximately 10 A. Heroin
patients are assigned to each dose. Treatment B. Cocaine
efficacy is determined based on urine toxicology C. Alcohol
screens and physical examinations conducted D. Phenobarbital
regularly throughout the study. This study is best E. Cannabis
described as which of the following?
39. A grandmother brings her 15-year-old grand-
A. Open-label clinical trial
daughter to your office, as she is concerned that
B. Randomized clinical trial
her granddaughter is spending excessive time
C. Case-controlled study
online playing video games. Her school perfor-
D. Single-blind, randomized, controlled trial
mance has decreased, and she has been more
E. Crossover study
irritable, isolating herself, and not sleeping. She
36. A pain management doctor assesses a 55-year-old has spent a significant amount of her grand-
female patient with lumbar spine injury, the patient mother’s savings downloading information and
rates her pain as a 9 out of 10 on a numeric rating upgrading her games. Which of the following
scale of 0 to 10. Which statement indicates the de- areas of her brain is being activated when she is
velopment of opioid tolerance in this patient? engaged in playing video games?
14 Practice Tests 135
D. Lorazepam D. Caffeine
E. Clonazepam E. Tobacco
45. An adolescent who has been using illicit opioids 48. Which of the following is not a medical comor-
4 to 5 days a week presents to your office for bidity of chronic and excessive alcohol use?
an evaluation. He has been more isolative and A. Glaucoma
missing multiple days of school. He appears to B. Dilated cardiomyopathy
be aware of the problems that drugs are caus- C. Myopathy
ing in his life. His use has been increasing slowly D. Hyperuricemia
over the last 12 months. Which of the following E. Osteonecrosis of the hip
is the most appropriate diagnosis?
49. A 24-year-old woman is brought in by a friend to
A. OUD
urgent care with altered mental status and visual
B. Opioid intoxication
hallucinations. Her friend said that their musi-
C. Personality disorder
cal band was practicing when her friend started
D. Withdrawal syndrome
playing the wrong songs and acting bizarrely.
E. Tolerance
On examination, the patient’s blood pressure
46. A 48-year-old man with OUD presents to your is high, and she has a low-grade fever. Which
clinic seeking treatment with buprenorphine/ of the following is another complication of the
naloxone. Point-of-care urine drug testing was substance likely being used by the patient?
positive for opiates. Using the following sub- A. Shortness of breath
stances would most likely cause a false-positive B. Hypotension
test? C. Lacrimation and yawning
A. Rifampicin D. Pinpoint pupils
B. Omeprazole E. Distortion of senses
C. Sertraline
50. Which of the following medications for alcohol
D. Phenytoin
use disorder predominantly works by attenuat-
E. Venlafaxine
ing the positively reinforcing effects of alcohol?
47. Which of the following substances is the most A. Disulfiram
widely used illicit drug in the United States? B. Naltrexone
A. Opioids C. Acamprosate
B. Alcohol D. Sertraline
C. Cannabis E. Gabapentin
BLOCK 3
1. What did the Supreme Court of California rule the crime should be diverted to drug courts
in 1991 in People v. Saille as it relates to alcohol rather than the regular adversarial judicial
intoxication as a criminal defense? system.
A. Alcohol intoxication is an affirmative D. Alcohol intoxication at the time of the
defense in criminal proceedings if the commission of the crime does not negate
defendant was unable to appreciate the specific intent or mens rea.
wrongfulness of his acts at the time of the E. Capital punishment for defendants who
commission of the crime. were intoxicated with alcohol at the time
B. Alcohol intoxication at the time of the com- of the commission of the crime is uncon-
mission of the crime indicates guilt. stitutional, as it violates the ban on cruel
C. Defendants who were intoxicated with and unusual punishment under the Eighth
alcohol at the time of the commission of Amendment.
14 Practice Tests 137
9. Regarding the use of different matrices for drug functioning. Which of the following is the most
testing, which of the following is true? accurate diagnosis?
A. Drug testing utilizing a blood matrix has the A. Fetal alcohol syndrome (FAS)
shortest window of detection. B. Partial FAS
B. Hair drug testing is often accurate and C. Alcohol-related neurodevelopmental disor-
should not be used in clinical situations. der
C. Urine drug testing utilizes polymerase chain D. Alcohol-related birth defect
reaction (PCR) technology to detect the E. Inherited FAS
presence of various drugs.
12. Which of the following are absolute contrain-
D. Hair drug testing is the least expensive and
dications for using disulfiram for treatment of
most widely used drug test.
alcohol use disorder?
E. The presence of PCP can be demonstrated
A. Migraine headaches
using urine or saliva matrices but not hair.
B. Hepatic impairment
10. A 27-year-old banker is required to submit to a C. Coronary occlusion
preemployment urine drug screening test. No D. Physiological dependence on opioids
drugs were detected on immunoassay. Which of E. Severe depression
the following scenario with the medical review offi-
13. There are several personality factors that are
cer determines that the negative test result is valid?
associated with a higher risk of alcohol use
A. pH: 4; temperature: 98°F; urine creatinine:
disorder. These include:
18 mg/dL; specific gravity: 1.035; nitrites:
A. High conscientiousness
9 mg/dL
B. High neuroticism
B. pH: 6; temperature: 98°F; urine creatinine:
C. High extraversion
18 mg/dL; specific gravity: 1.007; nitrites:
D. Low extraversion
4 mg/dL
E. High agreeableness
C. pH: 6; temperature: 103°F; urine creatinine:
1 mg/dL; specific gravity: 1.007; nitrites: 14. A 47-year-old man with OUD who is enrolled in
9 mg/dL a methadone maintenance treatment program
D. pH: 6; temperature: 103°F; urine creatinine: (MMTP) is required to attend the MMTP 6 days
18 mg/dL; specific gravity: 1.035; nitrites: a week, but he was only required to come twice
4 mg/dL weekly after 1 year of abstinence. He then has a
E. pH: 6; temperature: 98°F; urine creatinine: relapse with multiple positive urine toxicologies,
24 mg/dL; specific gravity: 1.035; nitrites: and subsequently, he is required to go to the clin-
1 mg/dL ic 6 days a week again. This treatment approach
11. A 10-year-old boy in foster care is brought to your is best described by which behavioral principle?
clinic due to poor attention, hyperactivity, learn- A. Classical conditioning
ing problems, and frequent fighting in school. His B. Incentive salience
foster parents confirm that his biological mother C. Operant conditioning
used alcohol heavily throughout pregnancy. On D. Negative reinforcement
examination, he has no discernible facial dysmor- E. Positive reinforcement
phisms characteristic of prenatal alcohol expo-
15. A 26-year-old man with alcohol use disorder who
sure, and he is in the 50th percentile for height,
committed arson while intoxicated is now court-
weight, and head circumference. Neurologi-
mandated to your alcohol treatment program.
cal examination and brain imaging are normal.
On presentation to your program, he explains
On neuropsychological testing, he has an IQ of
that he plans to attend the program because, af-
85 and clear deficits in attention and executive
ter 6 months of “good behavior” on his part, the
14 Practice Tests 139
court will reduce his requirement for treatment 18. In which of the following five scenarios might
participation. His explanation of his motivation the person be adjudicated not guilty?
to attend treatment is an example of which be- A. A 27-year-old man with alcohol use disorder
havioral principle? who drove while intoxicated because alcohol
A. Classical conditioning use caused disinhibition
B. Incentive salience B. A 27-year-old man who assaulted another
C. Operant conditioning patron at a bar after someone “roofied”
D. Negative reinforcement him by surreptitiously putting flunitrazepam
E. Positive reinforcement (Rohypnol) in his drink
C. A 27-year-old woman who committed a
16. A 16-year-old girl with depression and cannabis nonviolent crime under the influence of PCP
use disorder who lives with her parents presents D. A 27-year-old banker who committed the
to your clinic. She has been failing several classes crime under the influence of cocaine but
in high school and was recently arrested for can- does not meet criteria for SUD
nabis possession. She presents to your clinic, E. A 27-year-old physician who was sexually
and she has low motivation to stop using can- inappropriate with a patient because they
nabis. You recommend a combination of MET were under the influence of recreational
and CBT. Which additional treatment modality drugs but had no intent to harm had they
has the strongest evidence based data for treat- been sober
ing this patient’s cannabis use disorder?
A. Psychodynamic psychotherapy 19. In SBIRT, which of the following tools carries the
B. Supportive psychotherapy strongest evidence to screen for maladaptive al-
C. Family-based treatment cohol use?
D. Relapse prevention A. CAGE questionnaire
E. Twelve-step group attendance B. Michigan Alcoholism Screening Test (MAST)
C. Addiction Severity Index (ASI)
17. Which of the following is true regarding SUDs D. TWEAK questionnaire
among physicians? E. Alcohol Use Disorders Identification Test
A. SUDs are extremely rare in physicians (AUDIT)
because of higher education. However, the
20. A 47-year-old man with alcohol and cocaine
prognosis for a physician with an SUD is
use disorder, bipolar disorder, and multiple sui-
extremely poor.
cide attempts is well known in your emergency
B. The prevalence of SUDs among physicians
room. He self-presents to the emergency room,
is similar to that in the general population.
reporting suicidal ideation, and has an alco-
Physicians with SUDs have a good prognosis
hol level of 380 mg/dL. Although he is agitat-
as the potential to lose one’s license serves
ed, yelling, and pacing around the emergency
as an alternative reinforcer for sobriety.
room, he shows no signs of body sway or atax-
C. SUDs among physicians do not manifest
ia, speaks without slurring, and is oriented to
with the traditional symptoms of SUDs seen
person, place, and time. Which of the following
in the general population. Physicians are less
best explains why he does not exhibit symptoms
likely to develop withdrawal and tolerance
that are frequently exhibited at his presenting
but more likely to be charged with driving
alcohol level?
under the influence.
A. He is in a manic episode, which accounts for
D. Changes in sleep, weight, or mood are not
his psychomotor agitation.
associated with SUDs among physicians.
B. He coingested cocaine, which counteracts
E. Physicians with SUDs never present with
the effects of his alcohol level.
needle marks or bruises.
140 ADDICTION PSYCHIATRIC MEDICINE: A COMPREHENSIVE BOARD REVIEW
lorazepam. On completing the program, she to the intensive care unit. On evaluation, her
was started on disulfiram to help maintain her mental status examination is consistent with
sobriety. Two weeks later, she was arrested for delirium. Additionally, asterixis and ascites are
disorderly conduct and found to be paranoid noted. Laboratory testing revealed hepatitis C
and combative. Which of the following is the antibodies and antigens, creatinine of 5.4 mg/dL,
most likely explanation for this presentation? BUN of 87 mg/dL, and a very dilute urine. Which
A. She relapsed to alcohol. of the following is most likely to effectively treat
B. She stopped taking the risperidone. her condition?
C. Her medication regimen caused a recurrence A. Prescribing an ACE inhibitor such as lisino-
of psychotic symptoms. pril
D. She is using other substances such as PCP. B. Prescribing an antithrombotic agent such as
E. This presentation is consistent with benzodi- clopidogrel
azepine withdrawal. C. Lung transplant
D. Liver transplant
28. Which of the following best represents the rela-
E. Portal vascular shunting
tionship between alcohol use disorder and co-
morbid psychiatric disorders? 32. With regard to TSF, which of the following most
A. Chronic and excessive alcohol use causes accurately describes the goals of the psychother-
psychiatric symptoms. apeutic intervention?
B. Chronic and excessive alcohol use causes A. TSF posits meeting patients where they are
other psychiatric disorders. at, helping them resolve their ambivalence
C. Manifestations of chronic and excessive al- toward attending AA meetings.
cohol use may resemble symptoms of other B. TSF aims to encourage attendance and
psychiatric illnesses. meaningful utilization of 12-step meetings.
D. Alcohol use disorder may coincidently C. TSF aims to encourage patients to discuss
co-occur in patients with other psychiatric the results of their urine drug tests at 12-
illnesses. step meetings.
E. All of the above are possible. D. TSF aims to provide an evidenced based,
nonreligious alternative to AA and NA.
29. Which of the following is not a risk factor for
E. TSF is the collective name for psychothera-
completed suicide?
pies utilized in AA, NA, LifeRing, or SMART
A. Alcohol intoxication
recovery.
B. Alcohol use disorder
C. Previous suicide attempt 33. Which of the following is true concerning HCV
D. Intellectual disability infections in the United States?
E. Access to means for suicide A. Although HCV is mostly transmitted by
blood, at least 30% of those infected with
30. Which of the following medications would not be
the virus contracted it sexually.
safe to be used for alcohol withdrawal management
B. The HCV vaccine is only recommended for
in patients with severely impaired liver function?
injection drug users because of its potential
A. Chlordiazepoxide
to cause cardiac arrhythmia.
B. Lorazepam
C. HCV is most prevalent among those born
C. Oxazepam
between 1945 and 1965.
D. Gabapentin
D. Persons with co-occurring HIV and HCV
E. Temazepam
carry a significantly elevated risk of liver cir-
31. A 42-year-old woman presents to the emergency rhosis or liver-related mortality.
room with rapidly progressive renal decompensa- E. Malarial infections are protective against
tion. She requires urgent dialysis and is admitted contracting HCV.
142 ADDICTION PSYCHIATRIC MEDICINE: A COMPREHENSIVE BOARD REVIEW
34. Psychiatric side effects are most common in pa- E. How many times did you have 15 or more
tients using which of the following antiretroviral drinks in a single week for men or 8 or more
medication for HIV treatment? drinks in a single week for women over the
A. Nevirapine past year?
B. Zidovudine
37. A 31-year-old man presents to the emergency
C. Lamivudine
room with decreased respiratory rate, slurred
D. Efavirenz
speech, pinpoint pupils, and disorientation.
E. Tenofovir
Which of the following medications should be
35. A 27-year-old woman patient is brought into used to counteract the patient’s symptoms?
the psychiatric emergency room by emergency A. Naltrexone
medical services with symptoms of psychosis. B. Naloxone
She acknowledged using phencyclidine (PCP) C. Buprenorphine
and was admitted to the inpatient psychiatric D. Fluphenazine
unit for further management and stabilization E. Valproic acid
of the psychiatric symptoms. Which of the fol- 38. During an annual physical examination, a
lowing would be most helpful for the treating 46-year-old woman informs her primary care
psychiatrist to determine whether the patient’s physician that she has been drinking five alco-
psychosis results from a primary psychotic ill- holic beverages three to four times every week.
ness instead of a psychosis induced by her use Laboratory testing revealed elevated transami-
of PCP? nases three times higher than the cutoff value.
A. A urine drug test showing the presence of Her physician discussed the relationship between
PCP her abnormal findings and her excessive alco-
B. Using collateral information obtained from hol use and recommended that she cut down
emergency medical services, as well as the on drinking to prevent further hepatic function
patient’s roommate to complement the deterioration. She appeared to understand and
structured diagnostic evaluation agreed to drink less. Which of the following best
C. Having a family history of schizoaffective demonstrates this treatment approach?
disorder A. Motivational interviewing
D. Having a long history of PCP and metham- B. Brief treatment
phetamine use C. Screening and brief intervention
E. Performing multiple longitudinal psychiatric D. CBT for SUD
assessments E. Referral to treatment
36. Which of the following is a validated single 39. The use of Screening and Brief Intervention (SBI)
screening question for the detection of alcohol would be expected to be most effective for which
use disorder? of the following patients?
A. How many times have you consumed alco- A. A 34-year-old man with cocaine use disorder
hol to the point of intoxication over the past who is recovering from a cocaine-induced
year? acute coronary event.
B. Have you consumed alcohol before 10 a.m. B. A 28-year-old woman with moderate alco-
in the past year? hol use disorder in treatment with an addic-
C. Have you unsuccessfully attempted to cut tion psychiatrist.
down or discontinue your alcohol use in the C. A 28-year-old woman who is participating in
past? a drug court program for heroin use.
D. How many times did you have 5 or more D. A 34-year-old man who drinks 5 alcoholic
drinks in a single day for men or 4 or more beverages 6 days a week seen at his primary
drinks for women over the past year? care clinic for his annual physical exam.
14 Practice Tests 143
C. Interaction with ibuprofen for chronic pain interventions, which of the following pharmaco-
D. Need to always try the oral formulation logical agents is the treatment of choice for this
first patient’s condition?
E. Need for at least mild withdrawal with the A. Disulfiram
first use B. Gabapentin
C. Ondansetron
48. Which one of the following pharmacotherapies
D. Acamprosate
should be considered a first-line treatment for
E. Naltrexone
pathologic gambling?
A. Topiramate 50. The majority of individuals with alcohol use dis-
B. Lithium order exhibit cognitive deficits with neuropsy-
C. Naltrexone chological testing. Which of the following cog-
D. Bupropion nitive domains show the earliest reversal upon
E. Amphetamine establishment of abstinence?
A. Short-term memory
49. A 46-year-old woman with a long history of an
B. Abstraction
alcohol use disorder, hepatitis C, and cirrho-
C. Problem solving
sis is referred to your clinic for treatment. She
D. Verbal learning
expresses a desire to discontinue her alcohol
E. Visuospatial abilities
consumption. In addition to psychotherapeutic
PRACTICE TESTS ANSWER KEY
(CHAPTER 14)
methamphetamine use disorder. Aripiprazole did acamprosate, and the third was a combined behav-
not have an effect on cue-induced methamphet- ioral intervention (CBI). One of the striking find-
amine craving but was associated with an increase ings of this study was that acamprosate did not
in some of the rewarding and stimulatory effects outperform placebo in reducing drinking when
produced by methamphetamine use (Newton et al., given alone or in combination with CBI. At the
2008). end of the 16-week treatment period, among the
best treatment outcomes were observed in patients
6. C. Nitrous oxide
receiving naltrexone with or without CBI (Anton
Nitrous oxide is an odorless, colorless gas and effec- et al., 2006). Only naltrexone and acamprosate
tive sedative with a rapid onset and recovery. How- were studied in this trial, ruling out answer choices
ever, if it is misused and abused it can lead to the C, D, and E.
irreversible inactivation of vitamin B12. A neuro-
logical examination can reveal sensory ataxia and 10. E. Inhibition of dopamine-beta-hydroxylase
pseudoathetosis in the upper limbs with reduced Disulfiram inhibits dopamine-beta-hydroxylase
vibration sensation bilaterally. Laboratory studies (DBH), which metabolizes dopamine to norepi-
can show low serum vitamin B12 concentration, nephrine. Inhibiting DBH increases dopamine
mild macrocytosis, and raised serum homocyste- levels, which is believed to reduce the positive,
ine concentration. rewarding effect of cocaine. Disulfiram inhibits
aldehyde dehydrogenase, its relevant mechanism
7. D. Serotonin
of action for treating alcohol use disorder (answer
3,4-Methylenedioxymethamphetamine (MDMA) choice A); it does not inhibit alcohol dehydroge-
use is associated with deficits in sleep, sexual nase (answer choice B). Ditiocarb is a metabolite
dysfunction, reduced immune-competence, and of disulfiram, which has been found to have anti-
increased oxidative stress. Serotonin plays an neoplastic effects through its interference with the
essential role in mood, sleep, and appetite regula- ubiquitin-proteasome system (answer choice C).
tion. The excess release of serotonin by MDMA can Disulfiram is not a monoamine oxidase inhibitor
cause a depletion of serotonin causing a dysregula- (answer choice D) (Kosten et al., 2013).
tion of emotions, impulsiveness, and aggression.
11. C. CBT conceptualizes the interconnections
8. C. Twelve-step facilitation (TSF) between thought processes, emotional responses,
Project MATCH compared three psychosocial and behaviors reactions in drug and alcohol use.
interventions for treatment of alcohol use disorder: As adapted for SUD, CBT is based on the Relapse
CBT, MET, and TSF. Its main finding was that all Prevention (RP) model by Marlatt (and Gordon),
three interventions produced roughly equivalent which is based on social learning theory and oper-
clinical improvement. Subgroup analysis found ant conditioning principles. The model posits that
that MET was better for angrier patients with lower relapse follows immediate determinants of sub-
initial motivation. CBT was superior for patients stance use (such as triggering people, places, and
with less severe alcohol use disorder, and TSF was things; coping strategies; or outcome expectancies)
superior for patients with little social support for and covert antecedents to substance use, including
abstinence (Matching alcoholism treatments to lifestyle factors and craving. Answer A describes
client heterogeneity, 1998). psychodynamic therapies. Answer B describes
9. B. Naltrexone motivational interviewing. Answer D describes
contingency management. Answer E describes
The COMBINE study was a multicenter, random-
mindfulness-based interventions.
ized placebo-controlled trial that compared dif-
ferent combinations of three interventions for 12. C. Naloxone
alcohol use disorder; two of the interventions were Naloxone is a mu-opioid receptor antagonist
medication management with naltrexone and/or that is used in opioid overdoses to counteract
PRACTICE TESTS ANSWER KEY 163
life-threatening effects on the central nervous ideas influence mood, thoughts, physical reac-
system and respiratory system. Naloxone has no tions, and behaviors. Dialectical behavior therapy
potential for abuse. Naloxone wears off in 20 to 90 is based on CBT, focusing on emotional and social
minutes, and a person may require administration aspects, and was developed to help people cope
of more than one dose. with extreme or unstable emotions, sense of emp-
tiness, and harmful behaviors. MET is a counsel-
13. A. Precontemplation
ing approach that helps resolve ambivalence about
The transtheoretical model of change (TTM) is a engaging in treatment, evoking rapid and inter-
useful model to determine a given patient’s readi- nally motivated change, reducing substance use,
ness to change their behaviors as it applies to sub- and stopping drug use.
stance use. It includes the following stages:
15. C. Bupropion
■ Precontemplation: Patients in this stage are not There are no FDA-approved medications for
considering change and present with active treating cocaine or methamphetamine use disor-
resistance. The patient in this vignette appears ders. Bupropion is a norepinephrine–dopamine
to be in the precontemplation stage. The main reuptake inhibitor with a structure similar to
clinical risk during this stage is dropout from amphetamine. Some studies have found it helpful
treatment. during the initial treatment of cocaine and meth-
■ Contemplation: Patients in this stage are ambiv- amphetamine use disorder. It has been found to
alent about changing their behaviors. significantly reduce cravings and treat depressive
■ Preparation: Patients in this stage are taking symptoms, which are common during withdrawal
steps toward changing their behaviors and pres- of methamphetamines and other stimulants.
ent with increasing confidence in their decision
to change. 16. B. Flumazenil binds to the benzodiazepine-bind-
■ Action: Patients in this stage are overtly modify- ing site on the GABA-A receptor.
ing their behaviors and factors in the environ- Flumazenil competitively antagonizes the GABA-
ment that support continued substance use. The A receptor at the benzodiazepine binding site,
main clinical risk during this stage is relapse. where both benzodiazepines and Z-drugs bind.
■ Maintenance: The goals for this stage include Given its mechanism of action, flumazenil can
sustaining changes, consolidating gains, learn- reverse benzodiazepines and Z-drug overdoses,
ing alternative coping strategies, and recogniz- but not barbiturate or alcohol poisoning.
ing triggers. 17. B.
■ Relapse and recycling: This stage is not inevi-
Delta-9-tetrahydrocannabinol can be easily iden-
table. The main clinical risk during the stage is
tified by its long aliphatic chain which renders
for patients to feel stuck in their old ways and as
it highly lipophilic and hydrophobic. Answer
such, the primary goal of treatment is avoiding
choice A is the chemical structure for phencycli-
becoming stuck and redefining the relapse as an
dine. Answer choice C is the chemical structure of
opportunity for further self-improvement.
3,4,-methylenedioxymethamphetamine (MDMA).
■ Termination: Patients in this stage have reached
Answer choice D is the chemical structure for
their ultimate goals and are able to exit the cycle
amphetamine. For both MDMA and amphet-
of change without fear of relapse.
amine, you can observe the phenethylamine
14. B. CBT structure, which is common to many psychoac-
tive compounds. Answer choice E is the chemical
CBT is a combination of two therapeutic
structure for heroin.
approaches, cognitive therapy and behavioral ther-
apy. CBT helps a person remove self-sabotaging 18. E. CBD
thoughts and high-risk situations while working Prescription CBD (marketed as Epidiolex) was
on avoidance skills by focusing on how a person’s recently FDA-approved for the treatment of
164 PRACTICE TESTS ANSWER KEY
seizures in Dravet and Lennox-Gastaut syndrome results do not definitively provide topiramate’s effi-
for patients aged 2 years and older. Dronabinol is cacy in managing cocaine use disorder (Shinn &
synthetic delta-9-THC that is FDA-approved for Greenfield, 2010).
the treatment of chemotherapy-related nausea and
vomiting, as well as weight loss in AIDS (answer 22. E. Naltrexone
choice A). Nabilone is a THC analogue with mini- Naltrexone is approved by the FDA to treat alco-
mal euphoric effects that is FDA-approved for hol and opioid use disorders. Naltrexone inhib-
cancer chemotherapy-related nausea and vomit- its dopamine release in the nucleus accumbens
ing (answer choice B). Rimonabant is a CB1 recep- and ventral pallidum, reducing gambling urges.
tor antagonist that was studied for the treatment Other medications, such as the selective sero-
of cannabis use disorder and was pulled from the tonin reuptake inhibitors (SSRIs), have been
global market due to serious psychiatric side effects tested based on the hypothesis that serotonin
(answer choice C). Medical marijuana is not FDA- dysfunction could be a potential mediator of
approved for any condition (answer choice D). gambling addiction.
19. E. The onset of action of edibles is longer and 23. A. Cannabis is categorized as Schedule I, but the
harder to predict. judicial branch of the federal government issued
Cannabis edibles have exploded in popularity over a memorandum indicating the states would not
the past decade, as have cannabis-related visits to be prosecuted for having a state-legal cannabis
the emergency room from their consumption. The enterprise.
primary reason is that patients, frequently younger Under U.S. federal law, cannabis is a Schedule I
and more inexperienced users, will consume more controlled substance with no currently accepted
edibles to pursue a psychoactive effect not realiz- medical use; however, individual states can have
ing that the onset of action of edibles is longer and medical and recreational cannabis programs
more variable, ranging from 1 to 3 hours. Com- because of a memorandum released by the U.S.
pared with patients who consume cannabis via Department of Justice in 2013 called the Cole
the smoked route, patients presenting with can- Memorandum, indicating that state-legal cannabis
nabis intoxication from edibles present with more enterprises would not be prosecuted.
severe intoxication and psychiatric symptoms.
Although edible cannabis does in fact have lower 24. E. Passive inhalation of cannabis smoke is unlikely
bioavailability, this does not explain the phenom- to result in a positive test.
enon described in the vignette (answer choice D) In the vast majority of cases, passive inhalation of
(Monte et al., 2019). cannabis smoke will not produce positive urine tests
at commonly utilized cutoff concentrations used for
20. D. Levamisole UDS testing of cannabis (answer choice A). Extreme
Levamisole is an anthelmintic agent used illicitly cannabis smoke exposure (e.g., sitting in a nonventi-
as a cocaine adulterant. It looks like cocaine and lated room for an hour or longer next to individuals
acts as a bulking agent, increasing the total weight smoking high-potency cannabis) may rarely result
of the sample, so the selling street price increases. in a positive test, but this is likely limited to the
Levamisole can cause neutropenia, agranulocyto- hours following cannabis exposure. Both ibuprofen
sis, and skin necrosis. and pantoprazole can rarely test false positives for
THC, but confirmation with GC-MS rules out these
21. D. Topiramate possibilities (answer choices B and C). Bupropion
Topiramate is believed to decrease the dopamine can test false positive for amphetamines, not for
release in the corticomesolimbic system involved THC (answer choice D). In this vignette, it is not
in reward and reinforcement mechanisms of plausible that his reported exposure would result in
using cocaine; however, clinical trials’ conflicting a positive UDS test (Cone et al., 2015).
PRACTICE TESTS ANSWER KEY 165
25. A. CB1 receptors are expressed in very low density evaluation for co-occurring psychiatric disorders
in the brainstem. and be longitudinally reassessed. Naltrexone would
Many drug overdose deaths are primarily caused not be an appropriate treatment for prolonged
by respiratory failure. However, cannabis does not alcohol withdrawal symptoms (answer choice A).
have significant respiratory effects because of the Acamprosate may be the right choice but still does
low density of CB1 receptors in the brainstem, not preclude a full psychiatric evaluation (answer
regulating respiratory drive. The highest concen- choice B). There is no evidence that this patient
tration of CB1 receptors is found in the hippo- requires readmission for detoxification after 1
campus and basal ganglia, but this is not directly month of inpatient rehabilitation (answer choice
relevant to the question (answer choice B). CB2 C). Symptoms secondary to substance intoxica-
receptors are mostly expressed in immune cells, tion or withdrawal will generally resolve without
although they can be found in the central nervous psychotropic medication, but this is unlikely given
system (answer choice D). Endocannabinoids reg- the time course of his symptomatology (answer
ulate several critical physiological functions, but choice E).
the respiratory drive is not one of them (answer
choice E). 28. C. The patient’s current dose of lorazepam may be
too low to adequately treat his anxiety disorder.
26. B. Assess the severity of the patient’s depression This question is testing your assessment of “pseu-
and current suicidal thoughts. doaddiction,” which is the notion that certain
The most critical risk factor for suicide is having a patients may be receiving inadequate doses of
history of past suicide attempts. SUD is a signifi- medication for their condition. They may repeat-
cant risk factor for attempted and completed sui- edly request increases in dosages and may be
cide. Additionally, acute and severe psychosocial incorrectly seen as being “drug-seeking” owning
stressors further increase the risk. As a result, this to ostensibly having an SUD. In the context of
patient presents with a very high risk for suicide benzodiazepine use and the treatment of anxiety,
and should be evaluated for current suicidal think- approaches to these patients include increasing
ing. Ensuring patient safety if he is suicidal might the dose or frequency of dosing, or considering a
require inpatient hospitalization. This patient is longer-acting agent.
undoubtedly at risk of contracting HIV and HCV.
However, laboratory testing for these infections 29. B. Operant conditioning
should be offered after acute stabilization and The primary influence on human behavior is
assessment of imminent threats (answer choice A). learning from the environment. Positive reinforce-
Having a history of suicide attempts or obtaining ment strengthens an action by providing a reward.
buprenorphine from illicit sources is not a contra- In this case, he has known the rewarding feeling
indication for buprenorphine treatment (answer of sitting at the table and gambling. Sitting at the
choices C and D). This patient might benefit from table ensures that he would repeat the action again
treatment with an SSRI. However, it is preferable to and again. The removal of an unpleasant reinforcer
wait until he completes medically assisted opioid can also strengthen behavior. This is known as
detoxification and reassess his depressive symp- negative reinforcement. Classical conditioning is
toms before prescribing an antidepressant medica- a condition discovered by Pavlov, a physiologist
tion (answer choice E). (answer choice C). A stimulus is paired with and
precedes the unconditioned stimulus (until the
27. D. Perform a psychiatric evaluation for co-occur- conditioned stimulus alone is sufficient to elicit a
ring psychiatric disorders. response). Classical conditioning is a reductionist
The prevalence of co-occurring psychiatric disor- explanation of behavior by breaking it down into
ders among patients with an SUD is approximately smaller stimulus-response units of action (answer
50%. This patient should receive a full psychiatric choice E).
166 PRACTICE TESTS ANSWER KEY
30. B. Women with alcohol use disorder have a higher is a transcription factor that is upregulated by
risk of negative medical sequelae. substance use and leads to increased dynorphin
Women have a lower prevalence of alcohol use (answer choice E).
disorder, including binge drinking, begin drink- 33. A. Activation of CRF and norepinephrine in the
ing at a later age, and are more likely to have a amygdala and VTA
co-occurring psychiatric disorder than men. In this vignette, the patient undergoes stress-
However, women have a shorter time from initia- induced reinstatement following the death of his
tion of alcohol use to the onset of alcohol-related wife. Stress-induced reinstatement depends on
problems, commonly referred to as “telescoping.” the activation of CRF and norepinephrine in the
Women appear to be more vulnerable to the neu- amygdala and the VTA (Koob & Volkow, 2016).
rotoxic effects of alcohol use and the development Answer choice B describes the neurocircuitry
of medical sequelae (e.g., cirrhosis and alcoholic relevant for drug-induced reinstatement. Answer
cardiomyopathy) than men. Variations may be due choice C describes the neurocircuitry relevant
to differences in alcohol metabolism, as women for cue-induced reinstatement. Answer choice D
have a slower rate of alcohol metabolism, a lower describes the basic neurocircuitry of the reward
percentage of total body water, and lower first-pass pathway. Answer choice E describes the biological
metabolism (Ceylan-Isik et al., 2010). effects of opioids on reward circuitry.
31. A. Ventral tegmental area (VTA), nucleus accum- 34. E. It sounds like you are worried that your anxiety
bens (NAc) will worsen if you stop drinking while at the same
This question is testing your knowledge of basic time recognizing the need to address your alcohol
brain reward circuitry. The reward pathway is the use.
mesolimbic dopamine pathway. It includes dopa- In motivational interviewing, patients are encour-
minergic cell bodies in the VTA in the midbrain aged to explore both the positive (such as disin-
that project to the NAc in the ventral striatum. All hibition, socialization, and pleasure, or in this
drugs directly or indirectly exert their main rein- case, self-medicating) and the negative and harm-
forcing effects through the VTA–NAc pathway. ful aspects of using drugs or alcohol. Reflections
Opioids inhibit GABAergic interneurons in the are likely the most critical aspect of motivational
VTA, disinhibiting VTA dopaminergic neurons interviewing, includes simple considerations (e.g.,
and increasing dopaminergic neurotransmission “So you are saying that alcohol is causing a drift in
to the NAc. Opioids also act directly on opioid your relationship.”), double reflections (e.g., “So on
receptors present in the NAc (Nestler, 2005). the one hand, alcohol helps you cope with stress
caused by work, but on the other hand, it is causing
32. C. Neuropeptide Y
a drift in your relationship.”), or amplified reflec-
Neuropeptide Y is a neuromodulator with anxio- tions (.e.g, after a patient tells you that they enjoy
lytic effects. It is part of the brain’s “antistress” sys- drinking alcohol, you overshoot and respond:
tem; it functionally opposes corticotropin-releasing “I see, drinking alcohol has been a very positive
factor. Alcohol withdrawal decreases neuropeptide experience in your life, and you have no interest
Y expression in the amygdala, increasing vulner- in cutting down.”). The statement in option E is an
ability to negative emotional states. Dynorphin is example of a double reflection.
an opioid peptide that is upregulated in withdrawal
states and leads to increased dysphoria (answer 35. E. Weekly drug testing at home and rewarding
choice A). Noradrenergic systems are upregulated her with money any time she tests negative for
during withdrawal, leading to anxiety and agitation marijuana.
(answer choice B). CRF mediates the physiological CM is based on the introduction of alternative
stress response and regulates the hypothalamic– reinforcers that are valued more than using drugs
pituitary–adrenal axis (answer choice D). CREB or alcohol (such as monetary rewards) that tip the
PRACTICE TESTS ANSWER KEY 167
balance in favor of abstinence and help promote upregulates dynorphin (answer choices D and E)
recovery. CM as a therapy involves positive rein- (Robison & Nestler, 2011).
forcement by rewarding patients with incentives
39. B. Persons with SUD learn to utilize the social activ-
for achieving their treatment goals such as absti-
ities and chores to model change and rely on their
nence (hence answer C is incorrect). The immedi-
new community to practice cognitive and behav-
ate reward of negative tests serves an important
ioral changes conducive to change and recovery.
reinforcement purpose. As a result, utilizing point-
of-care testing at home would be more effective The effectiveness of TC is thought to result from
than drug testing at a laboratory in which results the community serving as a model context for
can take several days to be reported (hence answer prosocial relationships and activities. TC staff are
D is incorrect). described as “rational authorities” and are often
peers in recovery rather than SUD experts. Pro-
36. D. Oxytocin grams focus on accountability, responsibility,
Oxytocin is a hormone secreted by the posterior and structuring a socially productive life. The TC
lobe of the pituitary gland. It has been shown to model has been adapted to correctional settings.
modulate substance use-related reward and seek- When combined with community-based TC on
ing behaviors in the subthalamic nucleus of people release from incarceration, such models are asso-
abusing methamphetamine. ciated with improved substance use and criminal
outcomes. The therapeutic focus on “resocializa-
37. B. Norpethidine tion” is that isolation and dysfunctional social and
Meperidine is known to induce seizures from the interpersonal coping mechanisms precede the
accumulation of meperidine’s metabolite, nor- long-term facility presentation for many patients.
pethidine. It would be best to avoid meperidine in
the elderly and patients with underlying hepatic or 40. A. Women show higher dropout rates and more
renal disease. The medication increases the risk of use during treatment than men.
normeperidine accumulation and increased risk There are significant gender differences in the use
for seizures. The use of meperidine in combina- of methamphetamines. Women initiate metham-
tion with other agents known to increase sero- phetamine use at an earlier age, have more severe
tonin has been linked with serotonin syndrome use, higher attrition rates from treatment, yet they
development. respond better to treatment than men. Women
have a lower incidence of emergency depart-
38. C. CREB ment-related deaths involving methamphetamine
Opioid, cocaine, and alcohol use can lead to the (Dluzen & Liu, 2008).
upregulation of CREB in the NAc. This creates an
41. A. Alcohol withdrawal
upregulation of dynorphin, binds to kappa-opioid
receptors, and produces a dysphoria-like response. Symptoms of alcohol withdrawal can start as early
These adaptations are believed to lead to drug tol- as 5 to 6 hours after the last drink. After the first 48
erance and a dysphoric state in the absence of the hours, alcohol withdrawal symptoms can include
drug. Delta-FosB is another key transcription fac- seizures; high blood pressure; and tactile, auditory,
tor relevant in addiction; in contrast to CREB, its and visual hallucinations. Particularly in patients
levels rise more slowly with repeated substance use who are unable to provide a reliable clinical his-
and remain high for months following abstinence. tory, it is important to always keep other intoxica-
Higher levels of delta-FosB lead to increases in the tion or withdrawal syndromes on the differential.
positive, rewarding effect of the substance (answer
choice A). Neuropeptide Y is not a transcription 42. C. Hyperthermia
factor (answer choice B). Myocyte-enhancing fac- Methamphetamine use can lead to cardiovascular,
tor-2 and nuclear factor κB are both transcrip- gastrointestinal, and renal complications. Tachy-
tion factors under study for addiction, but neither cardia and hypertension are frequently observed
168 PRACTICE TESTS ANSWER KEY
tachycardia. They help to decrease the central stim- than complete abstinence. BI for alcohol is associ-
ulatory effects of cocaine. ated with an overall decrease in alcohol use, drink-
Other causes of cocaine-induced coronary artery ing days, binge drinking, and alcohol-related deaths.
disease involve vasculopathy and vasoconstriction.
Cocaine use can also cause left ventricular hyper-
trophy, hyperthrombotic states, aortic dissection BONUS ANSWER
and rupture, and arrhythmias. Cocaine’s vascular
and cardiotoxic effects are not dose related. 51. D. Clozapine
48. D. Start nicotine replacement therapy Cigarette smoking and daily dose have positive
Hospitalized smokers benefit from NRT because it correlations with the plasma concentration of
has a rapid onset of action. Varenicline and bupro- medications metabolized by the CYP450 enzyme
pion are slower to reduce symptoms of irritability CYP1A2. Because clozapine is a substrate of the
and craving. enzyme being induced by smoking, the medica-
tion concentration of the medication and its effi-
49. B. AA has no dogma, theology, or creed. cacy in most cases might be affected. Remember
Even though AA utilizes spiritual concepts, it is not that smoking and not nicotine induces the isoen-
a religion and has no dogmas or creeds. It is best zyme CYP1A2. CYP1A2 activity is higher in heavy
described as a spiritually driven program and a way smokers than in nonsmokers, and quitting can
of life. Techniques used to promote change rely on normalize the CYP1A2 activity.
experiential learning, building structure, learning
coping strategies, and strengthening social net-
works. The goals of such a group include promoting
BLOCK 2
self-efficacy and a new identity. Although AA is an
abstinence-based and community support group
1. A. As applied to SUD, CBT aims to establish a
program, it does not posit that persons with alcohol
functional analysis framework identifying high
use disorder can learn to drink in moderation once
risk antecedents and consequences of their use.
in recovery. In large-scale studies, commitment to
abstinence was demonstrated to be the most impor- The CBT model in SUD treatment relies on the
tant predictor of prognosis and sustained recovery. same fundamental principle that thought pro-
cesses, emotional responses, and behavioral reac-
50. A. A 43-year-old banker who has been drinking a tions are interconnected. As adapted for SUD, CBT
bottle of wine daily for the past 6 months in the is based on the Relapse Prevention (RP) model by
context of increased stress at work. Marlatt based on social learning theory and oper-
SBIRT refers to an evidence-based practice to iden- ant conditioning principles. The model posits that
tify, reduce, and prevent maladaptive substance relapse follows immediate determinants of sub-
use in primary care or other non-SUD-focused stance use (such as triggering people, places and
settings. SBIRT serves as a public health initiative things, coping strategies, or outcome expectancies)
to identify persons at risk for developing an SUD and covert antecedents to substance use, includ-
or with an undetected SUD at the primary care ing lifestyle factors and craving. During the treat-
level. It offers those with less severe substance use ment course, the therapist and patient develop a
simple interventions to address simple substance functional analysis framework identifying high-
use problems and refer those with more complex risk antecedents and consequences of their use.
substance use treatment needs to specialized ser- Patients practice identifying and avoiding high-
vices. Evidence supporting brief interventions (BIs) risk situations and learn specific coping strategies
efficacy is strongest for maladaptive alcohol use. BI (including stimulus control and urge-management
is particularly effective for patients with less severe techniques and relapse road maps) to enhance the
alcohol use problems where the intervention goal is patient’s self-efficacy in maintaining sobriety. Net-
typically a reduction of alcohol consumption rather work therapy, not mentalization-based therapy,
170 PRACTICE TESTS ANSWER KEY
have a negative result on UDS tests. Sensitivity is response in alcohol use disorder, though this does
defined as the proportion of individuals with illicit not include naltrexone (answer choice B). CYP2A6
opioid use who have a positive result on a diagnos- encodes the cytochrome P450 2A6 enzyme, which
tic test. A test that has higher specificity will have metabolizes nicotine; specific polymorphisms are
fewer false positives, whereas a test with higher associated with heavier smoking (answer choice
sensitivity will have fewer false negatives. Posi- D). Specific polymorphisms of the AKT1 gene
tive predictive value is the probability that patients increase the risk for psychosis in cannabis users
who have a positive screening test result have the (answer choice E) (Anton et al., 2008).
disease. In contrast, the negative predictive value
18. B. Naltrexone
is the probability that a negative test result means
that the patient does not have a disease. The timeframe and description of symptoms are
most consistent with precipitated opioid with-
14. A. Formation of cocaethylene drawal from methadone because of oral naltrexone
Concomitant alcohol and cocaine use can form administration. This vignette demonstrates why it
a third active compound, ethylbenzoylecgonine, is essential to educate patients on the importance
commonly known as cocaethylene. The plasma of letting all their health care providers know that
half-life of cocaethylene is longer and can cause when they are taking methadone, especially given
behavioral and physiological effects similar to that methadone is not listed on state prescription
cocaine. monitoring programs when prescribed from an
opioid treatment program. In this case, the patient
15. D. Mirtazapine
likely did not disclose that he was on methadone
Mirtazapine is currently used to treat depression. and was prescribed naltrexone for his alcohol
It antagonizes noradrenergic alpha-2 receptor and use disorder, leading to precipitated withdrawal.
the serotonin 5-HT2A/C and 5-HT3 receptors. It Disulfiram reaction is another possibility, but it is
has demonstrated efficacy in reducing the reward- less likely given that he was not drinking or intoxi-
ing effect of cocaine and methamphetamine. cated at the time this occurred; disulfiram reac-
16. B. Tobacco use disorder tions occur minutes after alcohol consumption and
include cutaneous flushing, head throbbing, tachy-
Tobacco use disorder is the most common SUD
cardia, nausea, vomiting, and respiratory difficul-
in the United States. Approximately 60% to 80%
ties. Gabapentin, baclofen, and acamprosate are
of current smokers meet criteria for tobacco use
not known to cause this reaction (answer choices
disorder. Nicotine is the primary addictive sub-
C, D, and E).
stance in tobacco. Among those who experiment
with cigarettes, 20% to 30% will meet tobacco use 19. C. CBT
disorder criteria in their lifetime. CBT is a short-term, goal-oriented psychother-
17. C. OPRM1 apy treatment that focuses on developing skills to
manage factors that influence relapse. Its goal is to
The OPRM1 gene encodes the mu-opioid receptor.
change patterns of thinking or behavior.
Patients treated with naltrexone who have the G
allele of the A118G SNP of the OPRM1 gene have 20. B. Crack
been found to have a lower rate of relapse com- Crack cocaine is the free-base form of cocaine that
pared to patients homozygous for the A allele; this can be smoked. Benzoylecgonine is the active com-
genetic polymorphism may help identify patients pound tested in UDS tests for cocaine.
who are more likely to respond to naltrexone.
AUTS2 is associated with alcohol use disorder in 21. C. Opioids have a significant impact on reinforce-
genome-wide association studies (answer choice ment mechanisms in the VTA.
A). SLC6A4 encodes the serotonin transporter pro- The VTA is the most sensitive site for the brain’s
tein studied as a potential moderator of treatment self-administration of opioids. When animals
PRACTICE TESTS ANSWER KEY 173
self-administer heroin, they respond to a decrease The patient is unable to quit despite his best inten-
in dose by increasing the frequency of injec- tions. Varenicline has demonstrated superior effi-
tions. The administration of opioids into the VTA cacy in clinical trials when compared with placebo
increases opioid self-administration, whereas and bupropion. Smoking cessation is best man-
injection of opioid receptor antagonists in this aged with a combination of pharmacotherapy and
region does not affect heroin self-administration. behavioral modification.
22. C. She should start breastfeeding immediately; the 26. D. Sertraline
benefits outweigh the risks.
In the case in Question 26, pornography use is
Methadone is the treatment of choice for pregnant adversely affecting the patient’s life. It has resulted
women with OUD. Its long half-life prevents the
in a damaged relationship with his wife, finan-
fetus from undergoing withdrawal. There is no
cial instability, and difficulties at work. The use of
contraindication to breastfeeding for patients with
SSRIs in treating sexual addiction is based on the
hepatitis C or those using methadone. However,
proposed mechanism of serotonergic dysfunction
women who are HIV positive should not breast-
of sexual behavior. Naltrexone was not found to be
feed, as they can pass HIV on to the infant through
more effective than a placebo in one study but is
maternal milk.
effective in treating gambling disorder. Stimulants,
23. D. Cannabis-induced anxiety dopamine agonists, and tricyclic antidepressants
Cocaine use should be suspected, but a negative have not been found to have efficacy in treating
urine toxicology screen ruled that out. Because of sexual addiction.
the symptoms’ chronology, it is unlikely that the
27. D. Brain imaging studies revealed a loss of brain
patient is experiencing alcohol withdrawal syn-
tissue and cognitive impairments are noted on
drome. The symptoms described here are consis-
neuropsychological testing.
tent with a panic attack induced by cannabis use.
Anxiety caused by cannabis often occurs in inex- Persons with alcohol use disorder or chronic his-
perienced users. The higher the dose, the more tory of excessive alcohol use may present with
likely anxiety is to appear. Paranoid thoughts are neurocognitive impairment. These impairments
also frequent in marijuana intoxication, precipitat- can be detected on neuropsychological testing
ing fear, and anxiety. Synthetic marijuana usually and brain imaging. Magnetic resonance imag-
does not show up in ordinary urine drug assays, ing studies and diffusion tensor imaging reveal a
and dilated pupils will not accompany the with- loss of brain tissue with accelerating gray matter
drawal syndrome. loss. In more than half of persons with alcohol
use disorder, abnormalities can be seen on CT
24. B. Ammonium chloride
imaging even in the absence of cognitive deficits.
Amphetamines are weak bases that get trapped in Some of these changes may be reversible with
acidic environments, making them neutral and abstinence, suggesting that the abnormalities
lipid soluble. If an acid like ammonium chloride is might be at least in part evidence of brain tissue
added, the amphetamine is ionized and is excreted dehydration.
in the urine. Bicarbonate is used to treat over-
doses with weak acid drugs, such as barbiturates 28. E. Antisocial personality disorder
and aspirin. Flumazenil is used to treat overdoses SUDs are often comorbid with personality dis-
of benzodiazepines, and naloxone is used to treat orders, most commonly antisocial personal-
opioid overdoses. Naltrexone is used to prevent ity disorder, borderline personality disorder,
relapse in patients with OUD. and schizotypal personality disorder. Of these,
25. B. Recommend varenicline and behavioral an antisocial personality disorder is the most
modification. common.
174 PRACTICE TESTS ANSWER KEY
29. D. There are off-label medications that can be pre- commonly occurs because of respiratory arrest.
scribed to alleviate cannabis withdrawal symptoms. According to a case series of patients presenting to
Currently, there are no FDA-approved treatments the emergency room with GHB intoxication, 38%
for cannabis use disorder, although a number of presented with bradycardia (Liechti et al., 2006).
commonly used psychiatric medications have been 33. B. Sedative withdrawal
studied, are currently under study, or are used off-
One of the key differences between benzodiaz-
label. Some medications that are used off-label to
epine and alcohol withdrawal is that the timing,
alleviate cannabis withdrawal include quetiapine,
severity, and range of symptoms is more vari-
mirtazapine, and gabapentin. Rimonabant is a CB1
able in benzodiazepine withdrawal; the with-
receptor antagonist and an antiobesity medication
drawal syndrome depends on the dose, duration,
that showed efficacy for treatment of cannabis use
and half-life of the ingested benzodiazepine and
disorder in preliminary clinical trials, but its seri-
their metabolites. Recall that standard urine drug
ous psychiatric side effects led to discontinuation of
screens do not detect some of the most commonly
its study (answer choice A). Given the time course
used and abused benzodiazepines (e.g. clonaze-
of the patient’s symptoms, cannabis withdrawal is
pam, alprazolam, and lorazepam). In this vignette,
more likely than the development of a depressive
the patient may have ingested a longer-acting ben-
episode (answer choice B). There is no evidence
zodiazepine (e.g., clonazepam) that is missed by
that genetic testing can help determine the correct
standard UDS tests. The timing and nature of the
medication approach for cannabis use disorder
patient’s symptoms are not consistent with alco-
(answer choice E) (Brezing & Levin, 2018).
hol, cocaine, or opioid withdrawal, nor with a
30. E. None of the above psychotic episode.
The FDA has approved three medications to treat 34. D. Gabapentin
OUD: buprenorphine, methadone, and naltrex- Despite its name and the fact that it is structurally
one. But no medication is approved specifically for similar to GABA, gabapentin does not exert its
comorbid OUD and PTSD. No medications are effects through the GABA-A or GABA-B receptor.
explicitly approved for any SUD comorbid with Gabapentin binds to a voltage-gated calcium chan-
any other psychiatric disorder, for that matter. nel containing the alpha-2-delta-1 subunit that is
31. E. PCP distributed throughout the central nervous sys-
tem. Baclofen and GHB are both GABA-B recep-
PCP is an N-methyl-D-aspartate (NMDA) recep- tor agonists. Propofol is a GABA-A agonist and has
tor antagonist and D2 partial agonist causing NMDA antagonist activity as well. Phenobarbital is
paranoia and depersonalization. Psychosis, agita- a barbiturate and is a positive allosteric modulator
tion, and dysphoria can occur as well as occasional of the GABA-A receptor.
unprovoked aggressive behavior such as in this
case. The management of PCP intoxication mainly 35. A. Open-label clinical trial
consists of supportive care. An open-labeled clinical trial is a clinical trial type
in which information on the study is not withheld
32. B. Bradycardia
from the researchers and participants, which is in
GHB overdose is among the most dangerous contrast with a blinded experiment, in which data
because of its steep dose-response curve; its LD50 are withheld to reduce bias.
is only five times greater than the typical recre-
ational dose. Common signs and symptoms in 36. B. Larger doses of opioids are needed for analgesia.
GHB intoxication include vomiting (more com- Opioids are a mainstay of acute pain management.
mon with ethanol co-ingestion), seizures, respi- However, long-term use of opioids can create tol-
ratory distress, hypothermia, hypotension and erance and induced hyperalgesia, a paradoxical
bradycardia. Death from GHB overdose most increase in pain with opioid administration. The
PRACTICE TESTS ANSWER KEY 175
mechanism of opioid tolerance is complex. The improve after being abstinent for a few weeks. Oth-
mechanisms can include those mediated by opioid ers might suffer from independent or secondary
receptor desensitization and internalization, glu- psychiatric disorders.
tamate receptors, and the cAMP-protein kinase A
pathway. Glutamate receptors have long proved a a 41. C. Alcohol withdrawal
key component in the synaptic networks respon- This patient presents with a history of anxiety
sible for behavioral tolerance. symptoms and panic attacks that occurs mostly
in the mornings after abstaining from any alco-
37. D. Contingency management
hol use overnight. Consuming alcohol leads to
Contingency management provides rewards for a resolution of his symptoms. This is consistent
the desired behaviors, such as negative urine toxi- with alcohol withdrawal–related panic attacks and
cology screen tests. It is effective for various issues anxiety. Panic attacks related to substance use are
and conditions, including impulsive behaviors, seen most commonly with marijuana and lysergic
defiance, and substance use disorders. acid diethylamide (LSD) intoxication and alcohol
38. A. Heroin withdrawal. The vignette does not provide enough
information to support the diagnosis of general-
Some signs and symptoms of heroin intoxication
ized anxiety disorder or panic disorder. Further,
include flushing of the skin, dry mouth, pupillary
alcohol use would not be expected to prevent panic
constriction, sedation, and reduced respiratory
attacks or treat anxiety in patients with a general-
rate. Heroin withdrawal symptoms include pupil- ized anxiety disorder or panic disorder.
lary dilation, muscle spasms, lacrimation, yawn-
ing, restlessness, pain, gastrointestinal upset, and 42. C. Diagnosis of depression made at least 1 week
increased heart rate and blood pressure. The Clini- after the patient discontinues drinking
cal Opiate Withdrawal Scale (COWS) is a great A meta-analysis of placebo-controlled trials of
tool to quantify the severity of opiate withdrawal antidepressant medications among patients with
symptoms. SUD and depressive disorders by Nunes and
39. D. Nucleus accumbens Levin (2008) found that diagnosing depression
The nucleus accumbens is a crucial site in mediat- after at least 1 week of abstinence was associated
with a greater effect of antidepressant medication
ing reward behavior, and it is directly involved in
treatment. A depressive disorder that persists after
reinforcing addictive behavior for both SUD and
1 week or more of abstinence is likely indepen-
behavioral addictions.
dent of substance use and should be treated. For
40. C. He should be screened for depression upon patients who cannot achieve any amount of absti-
admission and reassessed frequently thereafter. nence, the meta-analysis recommended using
It is not uncommon for patients with substance clinical judgment in making decisions to treat
use disorders to present with various co-occurring depressive disorder based on the patient’s history,
psychiatric symptoms, particularly mood and neu- past episodes of independent depression, and
severity of the depressive disorder. Further, most
rovegetative symptoms. It is important to assess
positive studies included tricyclic antidepressants
any such symptoms seriously on an ongoing basis
or antidepressants with noradrenergic or mixed
to determine whether the patient requires active
mechanisms, such as serotonin and norepineph-
treatments, including with medications and spe-
rine reuptake inhibitors. Most studies using SSRI
cialized psychotherapies, and to prevent serious
were negative.
complications of untreated psychiatric illnesses,
including profound functional decline and/or 43. E. Start an antidepressant medication after this
suicidal behaviors. Many patients with substance- patient completes the medically supervised alcohol
induced mood disorders would be expected to detoxification if he remains symptomatic.
176 PRACTICE TESTS ANSWER KEY
In this clinical situation, it is unclear whether onset of an SUD, with the SUD playing a caus-
this patient’s depressive symptoms are substance ative role in the onset of the co-occurring psy-
induced and likely to resolve spontaneously or rep- chiatric disorder. Unlike substance-induced
resent an independent depressive disorder. Given disorders, with secondary psychiatric disorders,
that there are no acute safety concerns reported the symptoms do not improve after a period of
in the vignette, it is not unreasonable to hold off abstinence.
on starting antidepressant medications until he
completes alcohol detoxification and withdrawal 44. C. Oxazepam
symptoms subside. When thinking about the psy- Traditional immunoassay tests for benzodiaz-
chiatric manifestations of SUD, one must differen- epines detect both oxazepam and nordiazepam.
tiate between: Diazepam is metabolized to nordiazepam and
■ The expected effects of the substance: Drugs temazepam, and those compounds are further
and alcohol intoxication or withdrawal cause metabolized to oxazepam. Chlordiazepoxide and
psychiatric signs and symptoms that are part of clorazepate are both metabolized to nordiazepam.
the normal toxidrome of a given substance. For Clonazepam, alprazolam, and lorazepam are not
example, feeling euphoric after using cocaine or metabolized to oxazepam. Their metabolites have
anxious when going through alcohol withdrawal limited cross-reactivity with traditional immuno-
are expected effects of cocaine intoxication or assay, resulting in false negatives.
alcohol withdrawal and should not be labeled as
45. A. OUD
a manic or anxiety syndrome.
■ Substance induced disorders: This category OUD is defined as a problematic pattern of opi-
refers to the presence of psychiatric symptoms oid use leading to clinically significant impair-
that: ment or distress. The patient in this case has been
■ Vastly exceed the expected effects of being isolating more, and his drug use has been affect-
intoxicated with or withdrawing from a sub- ing his social functioning and school responsibil-
stance ities. He is using opioids in increasing amounts
■ Meet diagnostic criteria for a psychiatric dis- and for longer than 12 months, and has contin-
order (psychotic, bipolar, depressive, anxiety, ued using opioids despite the negative conse-
obsessive compulsive, sleep, sexual, or neuro- quences of their use. The patient meets criteria
cognitive disorder) for an OUD.
■ Develop during or soon after substance intox-
did not reduce criminal responsibility. The court ■ Deontological theory deals with the clinician’s
reasoned that evidence of mental illness should duty and moral obligation to “do the right
not be admitted showing or negating the capacity thing.”
to form any mental state. ■ Utilitarianism theory aims to achieve the great-
est good for the greatest number of people.
2. E. (1): Beneficence and (2): autonomy ■ Liberal individualism theory places paramount
Autonomy, beneficence, nonmaleficence, and jus- importance on the patient’s rights and auton-
tice are the four principles of medical ethics. omy.
Autonomy principle requires that patients are ■ Virtue theory examines whether the clinical
free to make their own choices as long as they are provider intended to do good by the patient,
competent and have autonomy in their thought showing good character and moral values while
process, intentions and actions when making placing less emphasis on strictly following the
medical decisions. As such, the medical decision- rules and consequences.
making process must be free of coercion or undue ■ Consequentialism theory determines the moral
influence. worth of an action based on whether its out-
■ Beneficence principle requires that any medi- comes or consequences are good or bad.
cal decision offered, or any clinical intervention In this case, the psychiatrist determined that pro-
performed, serve to promote the health and wel- tecting the beneficence principle would violate the
fare of patients. It requires that clinical providers patient’s autonomy and vice versa. The psychia-
acquire the necessary knowledge and skills. trist used a paternalistic approach, prioritizing the
■ Nonmaleficence principles require that any health and welfare of the patient (beneficence) over
medical decision offered, or any clinical inter- the patient’s autonomy. This is consistent with the
vention performed, does not harm the patient. deontological theory.
■ Justice principle requires that health care pro-
viders make medical decisions that treat patients 4. C. A 27-year-old athlete prescribed codeine after a
equally and that the burden and benefits of new meniscal injury surgical repair.
or experimental interventions are distributed In a standard UDS test commonly referred to as
fairly across all patients. an SAMHSA-5 assay, the “opiates” immunoassay
In this case, the physician’s actions suggest a pater- uses antibodies specific for morphine metabo-
nalistic approach. The physician valued promot- lites. It detects the presence of natural and some
ing the health and welfare of the patient (seeking semisynthetic opioids such as heroin, morphine,
to improve the burden of the patient’s depression) and codeine. Detecting synthetic opioids such
over the patient’s autonomy and professed choice as meperidine, methadone, hydrocodone, oxy-
to continue smoking marijuana. codone, buprenorphine, or fentanyl requires an
extended opioid panel.
3. B. Deontological theory
In medical practice, it is not uncommon to face sit- 5. C. Acamprosate
uations where there is discordance in applying the Acamprosate should only be started when the
autonomy, beneficence, nonmaleficence, and justice patient has achieved abstinence; because of its
ethical principles. Different ethical theories can help GABAergic and antiglutamatergic activity, it is
guide clinical decision making in such situations: believed to attenuate the symptoms of protracted
alcohol withdrawal that place patients at risk for
■ Dialectical principlism theory is based on the relapse during early abstinence.
major moral principles and requires the balanc-
ing of competing principles in order to find a 6. A. It is a positive allosteric modulator of GABA-A
rich and ethically acceptable solution. receptors.
PRACTICE TESTS ANSWER KEY 179
thin vermillion border, or smooth philtrum), (2) and the use of these reinforcements to promote
growth retardation, (3) evidence of brain involve- behavior change is an example of operant condi-
ment, and (4) neurobehavioral impairments. Doc- tioning (Stitzer & Petry, 2006).
umented alcohol exposure is not required for the
FAS diagnosis. 15. D. Negative reinforcement
Drug courts have a strong behavioral orienta-
12. C. Coronary occlusion tion and generally utilize negative reinforcements
Severe myocardial disease or coronary occlusion more often than positive support to maintain the
is an absolute contraindication to disulfiram use. desired behavior. In negative reinforcement, a
The disulfiram–alcohol reaction can cause coro- negative stimulus (in this case, required program
nary vasospasm, chest pain, and, in rare cases, attendance) is reduced with a behavior (continued
cardiorespiratory death. Hepatic impairment is a abstinence). In positive reinforcement, a reward is
relative but not absolute contraindication to its use. provided for the desired behavior (e.g., praise or a
Disulfiram can cause fatal drug-induced hepatitis small monetary reward).
in 1/30,000 patients and increase cholestatic and
fulminant hepatitis. Physiological dependence on 16. C. Family-based treatment
opioids is a contraindication for using naltrexone, Studies have consistently demonstrated the effi-
but not disulfiram. Depression (severe or other- cacy of family-based therapy for adolescents with
wise) and migraines are not contraindications for cannabis use disorder, with some studies show-
using disulfiram. ing superiority compared with other individual
and group-based treatments. Family-based treat-
13. B. High neuroticism ment addresses a wider scope of issues, including
Alcohol use disorder is significantly associated family communication, peer networks, work, and
with higher neuroticism, lower conscientiousness, school attendance. Answer choices A, B, and E do
and lower agreeableness. It is not associated with not have a strong evidence based for treatment of
differences in extraversion or openness to experi- adolescent cannabis use disorder. Relapse preven-
ence (Malouff et al., 2007). tion (answer choice D) would not be appropriate
for an individual who is actively using substances
14. C. Operant conditioning and ambivalent about stopping (Hogue & Liddle,
Methadone maintenance programs provide incen- 2009).
tives for abstinence by reducing required days of
attendance based on the duration of abstinence; 17. B. The prevalence of SUDs among physicians is
conversely, active substance use is punished by similar to that in the general population. Physi-
the removal of take-home medication privileges. cians with SUDs have a good prognosis, as the
Contingency management is an evidence-based potential to lose one’s license serves as an alterna-
treatment approach that systematically reinforces tive reinforcer for sobriety.
desired behaviors by using rewarding and punish- Physicians, in general, are as likely as the general
ing reinforcements and is an example of operant population to be diagnosed with an SUD. Among
conditioning. Classical conditioning describes physicians, anesthesiologists and emergency phy-
how a neutral (unconditioned) stimulus elicits a sicians have the highest rates of SUDs. Once the
response after being paired with a biological potent SUD is identified and the physician submits to
(conditioned) stimulus (answer choice A). Incen- treatment and monitoring for SUD, prognosis for
tive salience refers to the desire or motivation to treatment is good as nearly four in five physicians
use a substance after being exposed to substance- achieve and sustain recovery. The fear of losing
related cues (answer choice B). Contingency one’s license combined with the highly structured
management in MMTP utilizes both positive and treatment programs offered through the physician’s
negative reinforcements (answer choices D and E), health programs contribute to these high recovery
PRACTICE TESTS ANSWER KEY 181
rates. SUD in physicians should be suspected when severity in seven potential problem areas (medi-
they exhibit signs or symptoms (albeit often non- cal status, employment and support, drug use,
specific) such as the deterioration in their physical alcohol use, legal status, family and social status,
appearance, irritability, behavioral changes, signifi- and psychiatric status) but is not widely utilized
cant weight changes, changes in sleeping patterns, in SBIRT.
or smelling of alcohol.
20. C. He has developed tolerance to alcohol and can
18. B. A 27-year-old man who assaulted another withstand higher alcohol levels
patron at a bar after someone “roofied” him by sur- Patients without alcohol tolerance who present
reptitiously putting flunitrazepam (Rohypnol) in with a blood alcohol level between 300 and 400
his drink mg/dL are at risk for coma, respiratory depres-
Engaging in a criminal act does not necessarily sion, and death. In this case, the patient very likely
mean that the person is criminally responsible or is a heavy chronic alcohol user and has developed
that they would be adjudicated guilty of the crime. a high tolerance to its intoxicant effects. Neither
Legal incompetence, insanity, and involuntary cocaine nor a concurrent manic episode would
intoxication represent three exceptions whereby explain this patient’s functional status at this serum
a person may commit unlawful behavior without alcohol level (answer choices A and B). Even if the
being adjudicated guilty. Involuntary intoxication patient had a genetic polymorphism that allowed
refers to situations in which a person is tricked, him to rapidly metabolize alcohol, it would not
forced under duress or undue influence to con- explain his functional status at this alcohol level
sume an intoxicating substance, or experience an (answer choice E).
atypical reaction to a substance or an intoxicating
21. C. GABAergic underactivity and glutamatergic
medication side effect.
overactivity
19. E. Alcohol Use Disorders Identification Test Alcohol enhances inhibitory GABA-A receptor
(AUDIT) function and inhibits excitatory glutamatergic
The Alcohol Use Disorders Identification Test NMDA receptors. Cessation of alcohol use in an
(AUDIT) is a validated questionnaire identifying individual who is alcohol dependent results in the
excessive or otherwise maladaptive patterns of pathophysiology of alcohol withdrawal: loss of
alcohol use. Its short version, the AUDIT-C con- GABAergic inhibition, glutamatergic overactivity,
sists of three questions: and a severe hyperadrenergic state.
■ How often do you have alcohol? 22. E. Ability to abstain from alcohol temporarily
■ How many standard drink units (SDUs) do you
Cloninger’s typology is frequently referred to in
drink on a typical day?
clinical practice and is a validated approach to
■ How often do you have six or more drinks on
categorizing alcohol use disorder. Type I patients
one occasion?
typically develop alcohol use disorder later in
A score of four or more for men or three or more life, are more likely to suffer from other psychi-
for women is considered positive. Of the options atric disorders, affect men and women equally,
listed, the AUDIT is the only tool that was vali- and typically respond better to treatment. In con-
dated in multiple controlled studies. trast, type II patients are defined as having earlier
The MAST screens for alcohol use in the elderly; onset of alcohol use disorder (<25 years of age),
the TWEAK (tolerance, worry, eye-opener, a prominent family history of alcohol use disor-
amnesia, K-cutting down) questionnaire screens der, higher prevalence of antisocial behaviors,
for alcohol use in pregnant women; the CAGE are predominantly men, and are often unable to
questionnaire is not a validated instrument with abstain from alcohol even temporarily (Leggio
wide applicability in SBIRT; ASI assesses SUD et al., 2009).
182 PRACTICE TESTS ANSWER KEY
patients with comorbid personality disorders ■ Develop during or soon after substance intox-
and SUD, an emphasis on structure and limit- ication or withdrawal
setting is required. Effective treatment often ■ Improve after a period of abstinence
which is associated with a lower odds of suicide the likelihood that SUD patients will affiliate with
(Harris & Barraclough, 1997). a mutual support group and reduce their substance
use (data supporting its effectiveness in alcohol is
30. A. Chlordiazepoxide strongest).
Chlordiazepoxide is metabolized hepatically. It has
a longer half-life of active metabolites compared 33. C. HCV is most prevalent among those born
to the other options. The other benzodiazepines between 1945 and 1965.
listed above are metabolized by glucuronide con- HCV is the most common chronic bloodborne
jugation. This enzyme is more rapid and produces infectious agent, four times more prevalent than
inactive metabolites, making it safer for liver dis- HIV. Its prevalence is highest among those born
ease patients. Gabapentin metabolism is not pri- between 1945 and 1965 (between 3% and 4%), the
marily hepatic. group known as the Baby Boomer generation. Up to
60% to 80% of injection drug users are HCV posi-
31. D. Liver transplant tive. Untreated, it can lead to progressive hepatic
This patient presentation is consistent with liver disease, cirrhosis, and hepatocellular carcinoma.
cirrhosis caused by chronic untreated HCV. Up to Risk factors for worse prognosis include alcohol
60% to 80% of injection drug users are HCV posi- use, HCV genotype 1, older age at the time of infec-
tive. Untreated, it can lead to progressive hepatic tion, male gender, co-occurring HBV, and higher
disease, cirrhosis, and hepatocellular carcinoma. viral load at the time of treatment initiation. The
It can cause hepatorenal syndrome, a renal fail- medical management of HCV includes ensuring
ure in patients with acute and severe liver failure that the person is vaccinated against HAV/HBV and
from decompensated cirrhosis or acute fulminant limiting alcohol consumption. Historically, HCV
hepatitis, causing jaundice, coagulopathy, and treatment consisted of interferon and ribavirin.
hepatic death and renal failure or variceal bleed- Newer interferon-free HCV antivirals have been
ing. Hepatic cirrhosis manifests with signs and developed and are becoming more easily accessible.
symptoms of hepatitis in addition to portal venous These include sofosbuvir, simeprevir, and ledipasvir.
disease, portal hypertension, ascites, bacterial
peritonitis, hepatocellular carcinoma, varices, and 34. D. Efavirenz
hepatic failure. This condition is irreversible even Antiretroviral medications used to treat patients
with discontinuation of alcohol use. Liver trans- with HIV include nucleoside reverse transcriptase
plants are the only effective therapeutic interven- inhibitors (NRTI), nonnucleoside reverse tran-
tion for cirrhosis. Hepatic cirrhosis is associated scriptase inhibitors (NNRTI), protease inhibitors
with a 50% 2-year mortality rate and is the sec- (PI), or integrase inhibitors (II). Highly active anti-
ond most common cause of liver transplants. To retroviral therapy (HAART) refers to a customiz-
be eligible for a transplant, one must abstain from able combination of antiretroviral medications to
alcohol use for at least 6 months. Transplants are help patients with HIV achieve an undetectable
indicated with ascites, encephalopathy, jaundice, viral load. Standard HAART protocols typically
or portal hypertension. include two NRTIs combined with one NNRTI, or
one PI or one II. Efavirenz, an NNRTI, is the anti-
32. B. TSF aims to encourage attendance and mean- retroviral medication most associated with psychi-
ingful utilization of 12-step meetings. atric adverse effects, including anxiety, depression,
TSF is not a 12-step mutual support group. It is a suicidal ideation, confusion, and hallucinations.
manualized and structured 12-session approach
that promotes the patient’s engagement in 12-step 35. E. Performing multiple longitudinal psychiatric
mutual support groups. In TSF, the therapist assessments
encourages the patient to participate in 12-step When thinking about the psychiatric manifesta-
mutual support groups and guides them through tions of substance use, one has to differentiate
the first four steps. Participation in TSF increases between:
PRACTICE TESTS ANSWER KEY 185
■ The expected effects of the substance: Drugs The NIAAA single-question alcohol screen has
and alcohol intoxication or withdrawal cause been validated as a single-item screen for detecting
psychiatric signs and symptoms that are part of alcohol use disorder with a sensitivity and specific-
the normal toxidrome of a given substance. For ity equivalent to longer screening instruments like
example, feeling euphoric after using cocaine or the AUDIT. Persons who screen positive should
anxious when going through alcohol withdrawal then receive an in-depth assessment to determine
are expected effects of cocaine intoxication or the risk level. Brief intervention and advice follow
alcohol withdrawal and should not be labeled as (per the SBIRT model).
a manic or anxiety syndrome.
37. B. Naloxone
■ Substance-induced disorders: This category
refers to the presence of psychiatric symptoms Naloxone is used in opioid overdoses to counteract
that: life-threatening effects on the central nervous sys-
■ Vastly exceed the expected effects of being tem and respiratory system.
intoxicated with or withdrawing from a sub- 38. C. Screening and brief intervention
stance
SBIRT serves as a public health initiative to iden-
■ Meet diagnostic criteria for a psychiatric dis-
tify persons at risk for developing a SUD or with
order (psychotic, bipolar, depressive, anxiety,
an undetected SUD at the primary care level. It
obsessive compulsive, sleep, sexual or neuro-
offers less severe substance use simple interven-
cognitive disorder)
tions to address simple substance use problems
■ Develop during or soon after substance intox-
and refer those with more complex substance use
ication or withdrawal
treatment needs to specialized services. There’s no
■ The symptoms improve after a period of absti-
information in the vignette suggesting that this
nence
patient requires a referral to specialized SUD treat-
■ Co-occurring primary psychiatric disorders:
ment services, and as a result, screening and a brief
This category refers to psychiatric disorders that
intervention would suffice. Motivational inter-
either preceded or followed the onset of a SUD,
viewing (MI) is an evidence-based psychothera-
but neither condition played a causative role
peutic intervention aiming to resolve ambivalence
in the onset of the other. For example, tobacco
and increase motivation to change maladaptive
use disorder and schizophrenia often co-occur,
behaviors. Brief interventions are often modeled
but schizophrenia does not cause the onset of
on the MI approach.
tobacco use disorder and tobacco use disorder
does not cause the onset of schizophrenia. 39. D. A 34-year-old man who drinks 5 alcoholic bev-
■ Secondary psychiatric disorders: This category erages 6 days a week seen at his primary care clinic
refers to psychiatric disorders that follow the for his annual physical exam.
onset of an SUD, with the SUD playing a causative SBI refers to an evidence-based practice to identify,
role in the onset of the co-occurring psychiatric reduce, and prevent maladaptive substance use
disorder. Unlike substance-induced disorders, used in primary care or other non-SUD-focused
with secondary psychiatric disorders, the symp- settings. Evidence supporting BI efficacy is stron-
toms do not improve after a period of abstinence gest for maladaptive alcohol use. BI is particu-
A comprehensive clinical evaluation that is per- larly useful with patients with less severe alcohol
formed multiple times longitudinally will assist the use problems. BI for alcohol is associated with an
prudent psychiatrist in making the correct diag- overall reduction in alcohol use, drinking days,
nostic determination. binge drinking, and alcohol-related deaths. Stud-
ies examining BI for illicit drug use found BI to be
36. D. How many times did you have five or more ineffective and costly, and as such, BI is not recom-
drinks in a single day for men or four or more mended to address the needs of patients with illicit
drinks for women over the past year? drug use.
186 PRACTICE TESTS ANSWER KEY
40. A. The perceived risk of marijuana use has declined and gastrointestinal disturbances. Low birth
among this age group. weight, oligohydramnios, and a decreased head
For over the past decade, the perceived risk of circumference are common findings.
cannabis use has been declining, accompanied by 45. D. State-dependent learning
an increase in cannabis use among teenagers and
State-dependent learning occurs when behavior
young adults. The inverse relationship between
acquired in the presence of a particular drug is
perceived risk and use also applies to cigarette
performed better on subsequent occasions when
smoking, which has declined among this age group,
that drug is present than when it is absent. Toler-
as its perceived risk has been increasing. The THC
ance can be created by the constant and chronic
content in cannabis has steadily been increasing
use of a substance, and a higher amount is needed
due to selective breeding for higher THC: CBD
to have the same desire wanted, while the sud-
plants (answer choice C and D). Marijuana is clas-
denly reduced dose might cause withdrawal. These
sified as an illegal Schedule I drug by the federal
symptoms can be mild (e.g., nausea or vomiting)
government (answer choice E).
or more serious (e.g., psychosis or seizures).
41. D. Liver disease 46. D. Discontinue zolpidem
In the United States, alcohol-related mortality has Zolpidem, zaleplon, and eszopiclone are all
alarmingly doubled between 1999 and 2017. This known to rarely induce complex behaviors while
has been the subject of extensive national discus- asleep, including sleepwalking, driving, eating,
sion and was highlighted by the economists Anne preparing food, and having sex. The patient gen-
Case and Angus Deaton. These deaths, also coined erally has no recollection of these actions. Because
“deaths of despair,” are primarily due to liver dis- of case reports of serious injuries and fatalities,
ease (31%), followed by overdoses (18%), either the FDA added a contraindication to prescribing
with alcohol alone or in combination with other these medications for any patient with a history of
drugs, followed by cardiovascular disease (11%) complex sleep behaviors while on these medica-
based on mortality data from 2017. However, glob- tions. It is also contraindicated to continue pre-
ally, alcohol-related injuries (traffic accidents, self- scribing these medications if patients experience
harm, or violence) are the most common cause of complex sleep behavior after taking any medica-
alcohol-related mortality. tions (Dyer, 2019).
42. B. Placental abruption 47. B. The requirement for at least 7 days of abstinence
before beginning treatment
The increased level of catecholamines caused by
cocaine use during pregnancy can cause vasospasm It is recommended not to start naltrexone until 7 to
in the uterine blood vessels, increasing the risk of 14 days of opioid abstinence to decrease the likeli-
experiencing placental separation and abruption. hood of precipitating withdrawal symptoms. Dur-
ing this period, patients may drop out of care or
43. B. Phencyclidine lose interest, making this a potential disadvantage
Both ketamine and phencyclidine are NMDA of naltrexone.
receptor antagonists. They mainly affect the cen-
48. C. Naltrexone
tral nervous system causing hallucinations, light-
headedness, headache, nightmares, and sensory Naltrexone effectively reduces the frequency and
changes. intensity of gambling urges and should be consid-
ered a first-line treatment for pathologic gambling.
44. A. Low birth weight No significant difference has been shown between
Exposing the fetus to heroin and other illicit sub- bupropion, mood stabilizers, and placebo in reduc-
stances can cause central nervous system hyperir- ing the frequency and intensity of gambling urges
ritability, autonomic nervous system dysfunction, in recent studies (Yau & Potenza, 2015).
PRACTICE TESTS ANSWER KEY 187
49. D. Acamprosate Brezing, C. A., & Levin, F. R. (2018). The current state of pharma-
cological treatments for cannabis use disorder and withdrawal.
The FDA approved three medications for the treat- Neuropsychopharmacology, 43(1), 173–194.
ment of alcohol use disorder: disulfiram, naltrex- Ceylan-Isik, A. F., McBride, S. M., & Ren, J. (2010). Sex difference in
one, and acamprosate. Of these, acamprosate would alcoholism: who is at a greater risk for development of alcoholic
be the preferred medication to use in patients with complication? Life Sciences, 87(5-6), 133–138.
Cone, E. J., Bigelow, G. E., Herrmann, E. S., Mitchell, J. M., LoDico,
hepatic insufficiency. It is excreted in urine as an
C., Flegel, R., et al. (2015). Non-smoker exposure to second-
unmetabolized drug. As a result, it should not be hand cannabis smoke. I. Urine screening and confirmation re-
used as a first-line agent for patients with renal sults. Journal of Analytical Toxicology, 39(1), 1–12.
impairment. Naltrexone is metabolized hepatically DiClemente, C. C., Schlundt, D., & Gemmell, L. (2004). Readiness,
via non-cytochrome–mediated dehydrogenase. It and stages of change in addiction treatment. American Journal
of Addiction, 13(2), 103–119.
is subject to an extensive first-pass effect and can
Di Forti, M., Iyegbe, C., Sallis, H., Kolliakou, A., Falcone, M.
be associated with an increase in serum transami- A., Paparelli, A., et al. (2012). Confirmation that the AKT1
nases. It is contraindicated in patients with acute (rs2494732) genotype influences the risk of psychosis in can-
hepatitis or hepatic failure. Disulfiram is metabo- nabis users. Biological Psychiatry, 72(10), 811–816.
lized hepatically via glucuronidation and possible Dluzen, D. E., & Liu, B. (2008). Gender differences in methamphet-
amine use and responses: A review. Gender Medicine, 5(1), 24–35.
side effects include cholestatic hepatitis, fulminant
Dodds, T. J. (2017). Prescribed benzodiazepines and suicide risk: A
hepatitis, and hepatic failure. It is relatively contra- review of the literature. The Primary Care Companion for CNS
indicated in patients with hepatic cirrhosis. Gaba- Disorders, 19(2), 16r02037.
pentin and ondansetron are used as second-line Dyer, O. (2019). FDA issues black box warnings on common insom-
treatment options for alcohol use disorder but are nia drugs. British Medical Journal, 365, l2165.
Farkas, R. H., Unger, E. F., & Temple, R. (2013). Zolpidem and driv-
not FDA approved for this condition.
ing impairment-identifying persons at risk. The New England
50. D. Verbal learning Journal of Medicine, 369(8), 689–691.
Harris, E. C., & Barraclough, B. (1997). Suicide as an outcome for
Some or all of the above cognitive domains have mental disorders. A meta-analysis. British Journal of Psychiatry:
been found to become impaired in 50% to 70% The Journal of Mental Science, 170, 205–228.
of individuals with alcohol use disorder; how- Heinzerling, A., Armatas, C., Karmarkar, E., Attfield, K., Guo, W.,
ever, verbal learning impairments are most quick Wang, Y., et al. (2020). Severe lung injury associated with use of
e-cigarette, or vaping, products—California, 2019. JAMA Inter-
to reverse following cessation of use with some nal Medicine, 180(6), 861–869.
studies demonstrating recovery within the first 2 Hogue, A., & Liddle, H. A. (2009). Family-based treatment for ado-
weeks of abstinence. As longer-term abstinence is lescent substance abuse: Controlled trials and new horizons in
reached, learning and memory issues are still pres- services research. Journal of Family Therapy, 31(2), 126–154.
ent in some individuals. Koob, G. F., & Volkow, N. D. (2016). Neurobiology of addiction: A
neurocircuitry analysis. The Lancet Psychiatry, 3(8), 760–773.
Kosten, T. R., Wu, G., Huang, W., Harding, M. J., Hamon, S. C., Lap-
REFERENCES palainen, J., et al. (2013). Pharmacogenetic randomized trial for
Anton, R. F., O’Malley, S. S., Ciraulo, D. A., Cisler, R. A., Couper, cocaine abuse: Disulfiram and dopamine β-hydroxylase. Bio-
D., Donovan, D. M., et al. COMBINE Study Research Group. logical Psychiatry, 73(3), 219–224.
(2006). Combined pharmacotherapies and behavioral inter- Leggio, L., Kenna, G. A., Fenton, M., Bonenfant, E., & Swift, R. M.
ventions for alcohol dependence: The COMBINE study: A (2009). Typologies of alcohol dependence. From Jellinek to ge-
randomized controlled trial. Journal of the American Medical netics and beyond. Neuropsychology Review, 19(1), 115–129.
Association, 295(17), 2003–2017. Liechti, M. E., Kunz, I., Greminger, P., Speich, R., & Kupferschmidt,
Anton, R. F., Oroszi, G., O’Malley, S., Couper, D., Swift, R., Pettinati, H. (2006). Clinical features of gamma-hydroxybutyrate and
H., et al. (2008). An evaluation of mu-opioid receptor (OPRM1) gamma-butyrolactone toxicity and concomitant drug and alco-
as a predictor of naltrexone response in the treatment of alcohol hol use. Drug and Alcohol Dependence, 81(3), 323–326.
dependence: Results from the Combined Pharmacotherapies Malouff, J. M., Thorsteinsson, E. B., Rooke, S. E., & Schutte, N. S.
and Behavioral Interventions for Alcohol Dependence (COM- (2007). Alcohol involvement and the Five-Factor model of
BINE) study. Archives of General Psychiatry, 65(2), 135–144. personality: A meta-analysis. Journal of Drug Education, 37(3),
Bachhuber, M. A., Hennessy, S., Cunningham, C. O., & Starrels, J. L. 277–294.
(2016). Increasing benzodiazepine prescriptions and overdose Matching alcoholism treatments to client heterogeneity: Project
mortality in the United States, 1996-2013. American Journal of MATCH three-year drinking outcomes. (1998). Alcoholism:
Public Health, 106(4), 686–688. Clinical and Experimental Research, 22(6), 1300–1311.
188 PRACTICE TESTS ANSWER KEY
Monte, A. A., Shelton, S. K., Mills, E., Saben, J., Hopkinson, A., Sonn, Shinn, A. K., & Greenfield, S. F. (2010). Topiramate in the treatment
B., et al. (2019). Acute illness associated with cannabis use, by of substance-related disorders: A critical review of the litera-
route of exposure: An observational study. Annals of Internal ture. Journal of Clinical Psychiatry, 71(5), 634–648.
Medicine, 170(8), 531–537. Sorensen, C. J., DeSanto, K., Borgelt, L., Phillips, K. T., & Monte,
Nestler, E. J. (2005). Is there a common molecular pathway for ad- A. A. (2017). Cannabinoid hyperemesis syndrome: Diagnosis,
diction? Nature Neuroscience, 8(11), 1445–1449. pathophysiology, and treatment—A systematic review. Journal
Newton, T. F., Reid, M. S., De La Garza, R., Mahoney, J. J., Abad, of Medical Toxicology, 13(1), 71–87.
A., Condos, R., et al. (2008). Evaluation of subjective effects of Stitzer, M., & Petry, N. (2006). Contingency management for treat-
aripiprazole and methamphetamine in methamphetamine-de- ment of substance abuse. Annual Review of Clinical Psychology,
pendent volunteers. International Journal of Neuropsychophar- 2, 411–434.
macology, 11(8), 1037–1045. Yau, Y. H., & Potenza, M. N. (2015). Gambling disorder and other
Nunes, E. V., & Levin, F. R. (2008). Treatment of co-occurring depres- behavioral addictions: Recognition and treatment. Harvard Re-
sion and substance dependence: Using meta-analysis to guide clin- view of Psychiatry, 23(2), 134–146.
ical recommendations. Psychiatric Annals, 38(11), nihpa128505.
Robison, A. J., & Nestler, E. J. (2011). Transcriptional and epigen-
etic mechanisms of addiction. Nature Reviews. Neuroscience,
12(11), 623–637.
REVIEW QUESTIONS ANSWER KEY
(CHAPTERS 1–13)
145
146 REVIEW QUESTIONS ANSWER KEY
have suggested the involvement of the μ-opioid It is not uncommon for patients with SUDs to
system in the reward processes of behavioral present with various co-occurring psychiatric
addictions leading to the hypothesis that opioid symptoms, particularly mood and neurovegetative
antagonists can reduce the repetitive behaviors and symptoms. It is important to assess any such symp-
the urges of addictive behaviors such as gambling. toms seriously on an ongoing basis to determine
Controlled studies on naltrexone, an opioid antag- whether the patient requires active treatments,
onist, have shown positive results in reducing the including medications and specialized psycho-
desire to gamble. There has not been a significant therapies, and to prevent serious complications of
difference between bupropion or mood stabilizers untreated psychiatric illnesses, including profound
such as lithium, topiramate, and placebo to treat functional decline and/or suicidal behaviors. Many
gambling disorder. patients with substance-induced mood disorders
would be expected to improve after being abstinent
6. C. Telescoping Phenomenon
for a few weeks. Others might suffer from indepen-
Sex differences have been noticed in gambling dent or secondary psychiatric disorders.
behaviors. It has been found that females have a
progression from nonproblematic to excessive 3. C. Alcohol withdrawal
engagement in gambling behaviors later in life than This patient presents with a history of anxiety
males. It also progresses faster from recreational to symptoms and panic attacks that occurs mostly
problematic. This phenomenon is described as the in the mornings, after abstaining from any alco-
telescoping effect or phenomenon. hol use overnight. Consuming alcohol leads to
a resolution of his symptoms. This is consistent
with alcohol withdrawal–related panic attacks and
CHAPTER 2 anxiety. Panic attacks related to substance use are
1. D. Acamprosate seen most commonly with marijuana and LSD
The U.S. Food and Drug Administration (FDA) intoxication and alcohol withdrawal. The vignette
approved three medications to treat AUD: disulfi- does not provide enough information to support
ram, naltrexone and acamprosate. Of these, acam- the diagnosis of generalized anxiety disorder or
prosate would be the preferred medication to use panic disorder. Further, alcohol use would not be
in patients with hepatic insufficiency. It is excreted expected to prevent panic attacks or treat anxiety
in urine as an unmetabolized drug. As a result, it in patients with a generalized anxiety disorder or
should not be used as a first-line agent for patients panic disorder.
with renal impairment. Naltrexone is metabolized 4. C. Diagnosis of depression made at least 1 week
hepatically via noncytochrome-mediated dehy- after patient discontinues drinking
drogenase and is subject to the extensive first-pass
A 2008 meta-analysis of placebo-controlled tri-
effect. Its use can be associated with increased
als of antidepressant medications among patients
serum transaminases, and it is contraindicated
with SUD and depressive disorders by Nunes and
in patients with acute hepatitis or hepatic failure.
Levin (2008) found that diagnosing depression
Disulfiram is metabolized hepatically via gluc-
after at least 1 week of abstinence was associated
uronidation. Its side effects include cholestatic
with a greater effect of antidepressant medication
hepatitis, fulminant hepatitis, and hepatic failure.
treatment. A depressive disorder that persists after
It is relatively contraindicated in patients with
1 week or more of abstinence is likely independent
hepatic cirrhosis. Gabapentin and ondansetron are
of substance use and should be treated. For patients
used as second-line treatment options for AUD but
who cannot achieve any amount of abstinence,
are not FDA approved for this condition.
the meta-analysis recommended using clinical
2. C. He should be screened for depression on admis- judgment in making decisions to treat the depres-
sion and reassessed frequently thereafter sive disorder based on the patient’s history, past
REVIEW QUESTIONS ANSWER KEY 147
episodes of independent depression, and severity myocardial ischemia. Cocaine causes hyperten-
of the depressive disorder. Further, most positive sion, increased systemic vascular resistance, and
studies included tricyclic antidepressants or antide- vasoconstriction. In addition to the standard pro-
pressants with noradrenergic or mixed mechanisms tocols used for non cocaine-related cardiac events,
such as serotonin and norepinephrine reuptake the medical treatment of cocaine-related angina
inhibitors. Most studies using SSRIs were negative. and myocardial infarction should include benzo-
diazepines. Benzodiazepines are necessary to man-
5. E. Start an antidepressant medication after this
age the hypertension and tachycardia and decrease
patient completes the medically supervised alcohol
the central stimulatory effects of cocaine.
detoxification if he remains symptomatic Other causes of cocaine-induced coronary
In this clinical situation, it is unclear whether artery disease involve vasculopathy and vasocon-
this patient’s depressive symptoms are substance- striction. Cocaine use can also cause left ventricu-
induced and likely to resolve spontaneously or rep- lar hypertrophy, hyperthrombotic states, aortic
resent an independent depressive disorder. Given dissection and rupture, and arrhythmias.
that there are no acute safety concerns reported, it
is not unreasonable to hold off on starting antide- 7. B. Assess the severity of the patient’s depression
and current suicidal thoughts
pressant medications until they complete alcohol
detoxification and withdrawal symptoms subside. The most important risk factor for suicide is hav-
When thinking about the psychiatric manifesta- ing a history of past suicide attempts. SUD is a
tions of SUD, one has to differentiate between: major risk factor for attempted and completed
suicide. Additionally, acute and severe psychoso-
■ The substance’s expected: Drug and alcohol cial stressors further increase the risk. As a result,
intoxication or withdrawal cause psychiatric this patient presents with a very high risk for sui-
signsss and symptoms that are part of the nor- cide and should be evaluated for current suicidal
mal toxidrome of a given substance. thinking. Ensuring the patient’s safety if he is sui-
■ Substance induced disorders: This category cidal might require inpatient hospitalization. This
refers to the presence of psychiatric symptoms patient is certainly at risk of having contracted
that vastly exceed the expected effects of being HIV and HCV. However, laboratory testing for
intoxicated with or withdrawing from a sub- these infections should be offered after acute sta-
stance and demonstrate symptoms of another bilization and assessment of imminent risks. Hav-
psychiatric disorder (psychotic, bipolar, depres- ing a history of suicide attempts or of obtaining
sive, anxiety, obsessive compulsive, sleep, sexual, buprenorphine from illicit sources is not a contra-
or neurocognitive disorder). indication for treatment with buprenorphine. This
■ Co-occurring primary psychiatric disorders: patient might benefit from treatment with an SSRI.
This category refers to psychiatric disorders that However, it is preferable to wait until he completes
either preceded or followed the onset of an SUD, a medically assisted opioid detoxification and reas-
but neither condition played a causative role in sess his depressive symptoms before prescribing
the other’s onset. the antidepressant medication.
■ Secondary psychiatric disorders: This category
8. E. Assess the severity of his opioid withdrawal and
refers to psychiatric disorders that follow the
begin induction with buprenorphine
onset of an SUD, with the SUD playing a caus-
ative role in the onset of the co-occurring psy- This patient is coming to your clinic seeking treat-
chiatric disorder. ment using medications for opioid use disorder.
Given his history of chronic pain and his ongoing
6. A. Increased cardiac workload and systemic vascu- pain despite the opioid use, buprenorphine might
lar resistance be a better therapeutic option than naltrexone
Cocaine use is associated with significant cardio- because of its analgesic effect. Given that he is cur-
vascular pathologies that could cause angina and rently in withdrawal, you might be able to begin
148 REVIEW QUESTIONS ANSWER KEY
the office-based buprenorphine induction on the certainly preferable that patients with HIV abstain
same day, maximizing the chances for his recovery. from drug and alcohol use, HAART is not contra-
In contrast, prescribing him clonidine and other indicated in patients who are actively using it.
medication support for opioid withdrawal and
11. D. Brain imaging studies revealed a loss of brain
discharging him with a follow-up appointment in
tissue, and cognitive impairments are noted on
1 week will increase the likelihood that he will con-
neuropsychological testing
tinue using illicit opioids after leaving your office
to relieve the withdrawal symptoms. Given the Persons of AUD or a chronic history of exces-
information presented in the vignette, he does not sive alcohol use may present with neurocognitive
appear to indicate a need for an inpatient admis- impairment that is detected on neuropsychologi-
sion for detoxification. Further, although having a cal testing and brain imaging. Magnetic resonance
history of suicide and SUD are major risk factors imaging studies and diffusion tensor imaging reveal
for suicide, the patient is not currently suicidal or a loss of brain tissue with accelerating grey matter
depressed. There is no indication to send him to loss. In more than half of persons with AUD, abnor-
the psychiatric emergency room. malities can be seen on computed tomography
imaging even in the absence of cognitive deficits.
9. B. Send him to the emergency room Some of these changes may be reversible with absti-
This patient should be evaluated urgently in an nence suggesting that the abnormalities might at
emergency room setting. Given his history of least be in part evidence of brain tissue dehydration.
injection drug use, sharing needles, chest pain,
12. E. Antisocial personality disorder
fatigue, fever, and tachycardia, he is at elevated risk
for infective myocarditis (endocarditis). Inject- SUDs are often comorbid with personality dis-
ing drugs increases the likelihood of bacterial orders, most commonly, antisocial personality
injections from nonsterile injection procedures. disorder, borderline personality disorder, and
Additionally, drug and alcohol use are associated schizotypal personality disorder. Of these, antiso-
with behavioral disinhibition, which could fur- cial personality disorder is the most common.
ther increase the risk of bacterial infections. Bac- 13. E. None of the above
teria, most commonly Staphylococcus, can travel
The FDA approves three medications to treat opi-
with blood and deposit as vegetations on cardiac
oid use disorder: buprenorphine, methadone, and
valves (most commonly the tricuspid valve). Bac-
naltrexone (both as a daily oral formulation and a
terial endocarditis is lethal if untreated. Treatment
long-term injectable formulation). No medication
involves a long course of intravenous antibiotics
is approved specifically for comorbid opioid use
and at times valve replacement.
disorder and PTSD. In fact, no medications are
10. E. Immediately approved specifically for any SUD comorbid with
Guidelines recommend starting HAART as soon as any other psychiatric disorder.
possible. There is no recommended CD4 count cut- REFERENCE
off at which to initiate therapy. In fact, evidence sug- Nunes, E. V., & Levin, F. R. (2008). Treatment of co-occurring de-
gests that starting HAART when CD4 count is high pression and substance dependence: using meta-analysis to
is associated with improved long-term outcomes. guide clinical recommendations. Psychiatric Annals, 38(11)
nihpa128505.
HAART is associated with improved CD4 count,
virologic suppression, and preserved immune CHAPTER 3
function. Earlier HAART initiation may result in
better immunologic responses and clinical out- 1. B. Conditioned place aversion
comes, reductions in AIDS and non–AIDS-asso- Conditioned place preference and self-administra-
ciated morbidity and mortality, and a significant tion measure the reinforcing effect of a substance
reduction in HIV transmission risk. Although it is and are animal models relevant for the binge/
REVIEW QUESTIONS ANSWER KEY 149
intoxication phase of the addiction cycle. Drug- probe protruding into the box. After touching the
and cue-induced reinstatement are relevant for the probe and receiving a shock, the animal generally
preoccupation/anticipation stage of the addiction buries the probe with the woodchips; observers
cycle. will measure variables related to burying including
the time to burying, the height of the woodchip
2. B. NPY
mound, and the total time spent burying. These
NPY is a neuromodulator shown to have anxio- measures are sensitive for increased anxiety-like
lytic effects and is part of the brain’s “anti-stress” states. Choices B, C, D, and E do not measure anx-
system that functionally opposes CRF. Dynorphin iety-like responses.
is upregulated in withdrawal states and leads to
increased dysphoria. Central noradrenergic sys- 6. B. Negative reinforcement
tems become activated in withdrawal and increase In negative reinforcement, a negative stimulus is
anxiety and agitation. The HPA axis becomes acti- removed with a behavior, making it more likely
vated in withdrawal and leads to elevated levels of that the behavior will be repeated. In this example,
CRF, ACTH, and glucocorticoids; ACTH does not resumption of heroin use reduces opioid with-
counteract or modulate the effects of CRF. drawal symptoms. Removal of these highly aver-
sive symptoms is a powerful negative reinforcer.
3. A. Norepinephrine
Dopamine is ubiquitously important in directly 7. D. Operant conditioning
or indirectly mediating the rewarding effect of all Operant conditioning refers to behavior modi-
drugs of abuse. Alcohol directly affects GABA, fication through use of punishing and rewarding
serotonin, and opioid receptors in the binge/intox- consequences. Methadone maintenance programs
ication stage. Norepinephrine is the least relevant use a contingency management approach through
neurotransmitter system in mediating the reward- their allotment of take-home doses for patients
ing effects of any substance of use. It is most rel- who have demonstrated abstinence.
evant in the withdrawal/negative affect stage in
8. A. Glutamatergic projections from the PFC to
which noradrenergic overactivity in the locus coe-
dopamine neurons in the VTA
ruleus is implicated in opioid withdrawal and can
be effectively treated with central alpha-agonists Incentive salience refers to cue-learning in which
such as clonidine or lofexidine. a stimulus becomes conditioned with a drug’s
rewarding effect, to the point in which the stimu-
4. C. Reinforcing drugs will lower the threshold for lus can induce motivation to drug-seek. Incentive
ICSS salience is regulated through glutamatergic projec-
The Olds and Milner classic experiment was an tions from the PFC to dopamine neurons in the
ICSS animal study showing that animals would VTA.
perform a response (lever pressing) to self-admin-
ister a stimulus delivered via implanted brain elec- 9. B. Hallucinogens
trodes to brain reward circuits. ICSS provides a According to data from large surveys of adult
way to study substances’ effects on brain reward twins, hallucinogen use disorder has the low-
thresholds. The addictiveness of substance is cor- est heritability among the common substances of
related with its ability to lower ICSS threshold. abuse, whereas cocaine and opioids have among
Answer choice A describes conditioned place aver- the highest (Ducci & Goldman, 2012).
sion, and answer choice B describes conditioned
10. A. GWAS
place preference.
Answer choice B is an older method used to iden-
5. A. Defensive burying tify chromosomal regions coinherited with the
In defensive burying, the animal is placed in a box phenotype of interest (e.g., substance abuse)
filled with woodchip bedding and an electrified among related individuals. Linkage studies have
150 REVIEW QUESTIONS ANSWER KEY
largely been replaced by GWAS, which are per- This question is testing your knowledge of the
formed by querying the genome with microarrays Project COMBINE research study. Disulfiram was
of thousands to millions of genetic markers to not included as one of the study’s three interven-
examine whether particular alleles are more com- tions, immediately ruling out answer choices B and
mon among individuals with the disease versus E. Acamprosate was not associated with a signifi-
controls. Advantages of GWAS include the ability cant reduction in drinking compared with placebo,
to identify multiple risk alleles with individual either alone or in combination with naltrexone at
small effect sizes, although studies typically require the end of the 16-week treatment period. The treat-
very large sample sizes. ment group that showed the best outcome received
MM with naltrexone without the CBI.
REFERENCE
Ducci, F., & Goldman, D. (2012). The genetic basis of addictive
4. D. Patients receiving medication management
disorders. Psychiatric Clinics of North America, 35(2), 495–519. (naltrexone) + no behavioral intervention
https://doi.org/10.1016/j.psc.2012.03.010. Of the nine treatment groups compared in the
COMBINE study, the groups performing best at
CHAPTER 4 the end of the treatment period received medi-
cation management (with naltrexone) with and
1. A. Her alcohol use is considered at-risk because without behavioral intervention, and behavioral
she is drinking more than the accepted weekly intervention with MM (with placebo).
amount for women.
This question is testing your knowledge of the 5. F. No group showed statistically superior outcomes
NIAAA standards of alcohol consumption for men at 1-year follow-up.
and women. According to the NIAAA, women According to results from the COMBINE study,
should drink no more than 7 standard drinks per all between-group effects observed following the
week and no more than 3 standard drinks per 16-week treatment period were no longer signifi-
occasion; men should drink no more than 14 stan- cant at 1-year follow-up.
dard drinks per week and no more than 4 drinks 6. B. Fomepizole prevents build-up of formaldehyde
per occasion. Drinking more than these limits is through inhibition of alcohol dehydrogenase.
considered to put individuals at risk for negative
health and social outcomes. To answer this question, you need to know both
methanol’s metabolic pathway and fomepizole’s
2. F. Inpatient; he has no social supports involved in mechanism of action. Methanol is metabolized to
his treatment. formaldehyde by alcohol dehydrogenase. Ethylene
This question is testing your application of the glycol is metabolized to glycolic acid, and ethanol
ASAM placement criteria to determine whether is metabolized to acetaldehyde; neither of these
this patient can be managed as an outpatient. In byproducts are in the metabolic pathway of metha-
general, patients who present with CIWA greater nol. Fomepizole competitively inhibits ADH, not
than 15 (indicating severe withdrawal), have ALDH, and prevents metabolization of methanol
unstable medical or psychiatric conditions that can to the highly toxic formaldehyde.
complicate treatment of withdrawal, have high lev- 7. A. Production of calcium oxalate crystals, leading
els of recent alcohol consumption, have a history of to renal injury
withdrawal seizures or DTs, who do not have any
social supports at home or cannot reliably present Ethylene glycol is metabolized to glycolic acid,
to an outpatient clinic daily should be managed in which is further broken down to calcium oxa-
the inpatient setting. late. Calcium oxalate crystals can then form in the
kidneys, leading to renal injury. Answer choice B
3. D. Acamprosate did not outperform either placebo refers to the effects of methanol; answer choice
or naltrexone. C refers to the effects of ethanol; answer choice
REVIEW QUESTIONS ANSWER KEY 151
D refers to the effects of isopropyl alcohol, which individuals with AUD. However, verbal learning
causes ketosis without metabolic acidosis. impairments are most quick to reverse following
cessation of use with some studies demonstrating
8. D. EtG
recovery within the first 2 weeks of abstinence. As
Indirect biomarkers do not contain ethanol or its longer-term abstinence is reached, learning and
direct metabolites and include MCV, CDT, GGT, memory issues are still present in some individuals.
AST, and alanine aminotranferase. Direct mea-
sures include EtG, EtS, BAC, and breath alcohol 13. D. Liver disease
concentrations. In the United States, alcohol-related mortality has
alarmingly doubled between 1999 and 2017. This
9. B. Women have a lower expression of ADH in the
has been the subject of extensive national discus-
gastric mucosa, reducing first-pass metabolism.
sion and was highlighted by recent publications
Gastric ADH is responsible for first-pass metabo- by economists Anne Case and Angus Deaton.
lism of alcohol. Women express less gastric ADH, These deaths, also coined “deaths of despair,” are
which allows more alcohol to be absorbed into primarily a result of liver disease (31%), followed
systemic circulation. Gastric ADH expression also by overdoses (18%) either with alcohol alone or
declines with advancing age. There is no evidence in combination with other drugs, followed by
for answer choice A. Answer choices C and D are cardiovascular disease (11%) based on mortality
true for individuals who express the ALDH*2 and data from 2017. Globally, however, alcohol-related
ADH1B*2 isotype, common among those with injuries (traffic accidents, self-harm, or violence)
East Asian heritage. are the most common cause of alcohol-related
10. B. Elevated heart rate mortality.
Vital signs are not factored into overall scoring 14. B. Naltrexone
but are typically recorded each time the CIWA-Ar Naltrexone is the only medication listed that pri-
is administered. The decision not to include vital marily acts by reducing the euphoric and pleas-
signs was based on data showing that pulse and ant effects of alcohol by blocking opioid pathways
blood pressure did not correlate as well with sever- projecting to the reward areas of the brain, thereby
ity of alcohol withdrawal compared with the other reducing alcohol-induced dopamine release and
signs and symptoms included in the CIWA-Ar. the positive reinforcing effects of alcohol. Disulfi-
11. C. ALDH; acetaldehyde ram predominantly works by creating a highly aver-
sive reaction (i.e., a punishment) in the presence of
Alcohol flush reaction is common among indi- alcohol, thereby incentivizing the patient to avoid it
viduals of East Asian descent because of a higher completely. Acamprosate reduces symptoms of pro-
frequency of both the ADH1B*2 and ALDH2*2 tracted withdrawal during abstinence and has no
alleles, both of which lead to accumulation of effect on the positive reinforcing effects of alcohol.
acetaldehyde. The ADH1B*2 allele metabolizes Sertraline has little efficacy for use in AUD beyond
ethanol more rapidly to acetaldehyde, whereas treating a co-occurring psychiatric disorder. Gaba-
the ALDH2*2 allele metabolizes acetaldehyde pentin has no effects on the positive reinforcing
more slowly; the presence of one or both of these properties of alcohol. Naltrexone is the best choice.
isotypes would lead to accumulation of acetalde-
hyde. Increased amounts of acetaldehyde lead to 15. C. Acamprosate
the release of histamines, which then causes vessel Acamprosate should only be started when the
dilatation and skin redness. patient has achieved abstinence; because of its
GABAergic and antiglutamatergic activity, it is
12. D. Verbal learning
thought to attenuate the symptoms of protracted
Some or all of the above cognitive domains have alcohol withdrawal that place patients at risk for
been found to become impaired in 50% to 70% of relapse during early abstinence.
152 REVIEW QUESTIONS ANSWER KEY
16. A. It is a positive allosteric modulator of GABA-A or otherwise) and migraines are not contraindica-
receptors. tions for using disulfiram.
Acamprosate is a positive allosteric modulator 20. B. High neuroticism
of the GABA-A receptor and an NMDA recep-
AUD is significantly associated with higher neu-
tor antagonist; it reduces glutamatergic activity
roticism and lower conscientiousness. It is not
and increases GABAergic activity during early
associated with differences in extraversion, open-
abstinence and can counteract the symptoms of
ness to experience, or agreeableness.
protracted withdrawal. Answer choices B and D
describe the mechanism of action for baclofen and 21. D. Acamprosate
gabapentin, respectively. The FDA approved three medications to treat AUD:
17. D. Alpha-2-receptor agonist disulfiram, naltrexone, and acamprosate. Of these,
Gabapentin was designed to be a structural analog acamprosate would be the preferred medication
of GABA and closely resembles endogenous amino to use in patients with hepatic insufficiency. It is
acids; despite this and despite its name, gabapentin excreted in urine as an unmetabolized drug. As a
does not directly agonize or antagonize the GABA- result, it should not be used as a first-line agent for
A or GABA-B receptor and does not modulate renal impairment patients. Naltrexone is metabo-
GABA activity. Gabapentin inhibits the alpha-2- lized hepatically via noncytochrome-mediated
delta-1 subunit of voltage-gated calcium channels. dehydrogenase and is subject to the extensive first-
pass effect. Its use can be associated with increase
18. C. ARND serum transaminases, and it is contraindicated
This question is testing your knowledge of the in patients with acute hepatitis or hepatic failure.
diagnostic criteria for FASD. This patient meets Disulfiram is metabolized hepatically via glucuroni-
criteria for ARND. Generally, the IQ of individuals dation. Its side effects include cholestatic hepatitis,
with FASD is above the threshold for intellectual fulminant hepatitis, and hepatic failure. It is rela-
disability (IQ >70). If this same patient had two or tively contraindicated in patients with hepatic cir-
more facial features of FAS, he would meet criteria rhosis. Gabapentin and ondansetron are used as
for pFAS. The diagnosis of FAS requires four cri- second-line treatment options for AUD but are not
teria: (1) two or more characteristic facial features FDA approved for this condition.
(short palpebral fissures, thin vermillion border, or
smooth philtrum), (2) growth retardation, (3) evi-
dence of brain involvement, and (4) neurobehav- CHAPTER 5
ioral impairments. Documented alcohol exposure
1. A. Alpha-1 subunit of GABA receptor
is not required for the FAS diagnosis.
Z-drugs have the greatest effect at the alpha-1 sub-
19. C. Coronary occlusion unit which mediates sleep and amnestic effects,
Severe myocardial disease or coronary occlusion is and weaker activity at alpha-2 and -3 subunits
an absolute contraindication to disulfiram use; the which mediates anxiolytic and anticonvulsant
disulfiram–alcohol reaction can cause coronary effects. Benzodiazepines exert their effects at
vasospasm, chest pain, and in rare cases can cause alpha-1 through -3 and -5 subunits (answer choice
cardiorespiratory death. Hepatic impairment is a C). Z-drugs do not interact with the NMDA or
relative but not absolute contraindication; disul- AMPA glutamate receptor.
firam can cause a fatal drug-induced hepatitis in
1 in 30,000 patients, and increases risk for choles- 2. D. Increase the duration of GABA channel
tatic and fulminant hepatitis. Physiological depen- opening
dence on opioids is a contraindication for using Barbiturates increase the duration of GABA
naltrexone but not disulfiram. Depression (severe channel opening (answer choice D), whereas
REVIEW QUESTIONS ANSWER KEY 153
benzodiazepines increase the frequency (answer their metabolic byproducts. This includes chlordi-
choice E). Barbiturates are antagonists, not ago- azepoxide, oxazepam, temazepam, and diazepam.
nists, at the AMPA and kainate receptor, ruling out However, some of the most commonly prescribed
answer choices A and B. Barbiturates are indirect benzodiazepines (e.g., clonazepam, lorazepam,
agonists and allosteric modulators of the GABA and alprazolam) do not share this metabolic path-
receptor, ruling out answer choice C. way and are therefore not detected by routine urine
drug tests.
3. E. There are no significant drug–drug interactions.
Most benzodiazepines are metabolized by CYP3A4 7. B. Complex sleep-related behaviors
oxidation followed by glucuronide conjugation. Following a number of reports serious injuries and
The so-called “LOT” (lorazepam, oxazepam, and deaths from sleep behaviors such as sleepwalking
temazepam) benzodiazepines do not undergo oxi- and sleep driving, the FDA placed a black box
dation and only undergo conjugation. Therefore, warning stating that use of zolpidem, zaleplon and
lorazepam does not have CYP-related drug–drug eszopiclone is contraindicated in patients who have
interactions, and answer choice E is correct. This experienced an episode of a complex sleep behav-
characteristic of LOT benzodiazepines makes ior while taking one of these medications. Benzo-
them the medication of choice in patients with diazepines, not Z-drugs, have a black box warning
hepatic impairment or where you wish to avoid stating that taking benzodiazepines together with
drug–drug interactions. opioids can be fatal (choice A).
4. D. Cleft palate 8. C. Memory impairment
Although the absolute risk remains very low, ben- Patients prescribed chronic benzodiazepines
zodiazepines have been found to increase the risk quickly build tolerance to their hypnotic, sedative,
of cleft palate from 6 in 10,000 to 11 in 10,000 based and anticonvulsant effects. Driving impairments
on retrospective case–control studies (Bellantu- and falls largely occur as a result of sedative side
ono et al., 2013). Neonatal abstinence syndrome effects. However, individuals do not tend to build
(answer choice A) can occur following chronic tolerance to the anxiolytic or amnesic effects of
maternal exposure to opioids. Placenta previa, still- benzodiazepines (choice C) (Vinkers & Olivier,
birth, and preeclampsia (answer choices B, C, and 2012).
E) are not known risks of benzodiazepine exposure
9. A. Transition patient from lorazepam to clonaz-
during pregnancy.
epam and perform a slow taper
5. B. High lipophilicity For patients who have been on benzodiazepines
The three pharmacologic characteristics that can longer term, it is generally recommended to per-
help predict all of the clinical effects of benzodi- form a gradual taper after cross-tapering to a ben-
azepines are lipophilicity, elimination half-life, and zodiazepine with a longer duration of effect, such
potency. Alprazolam has a long elimination half- as clonazepam. Ancillary medications may be used
life (11–15 hours) but is highly lipophilic (answer to help with anxiety and sleep during the taper, for
choice B), which accounts for its rapid onset of example, trazodone or hydroxyzine. Alprazolam
action and short duration of clinical effect. While has a short duration of effect and would not be
alprazolam does not have active metabolites, this a good choice for performing a taper (choice B).
does not explain why it has a short duration of Starting an SSRI may be reasonable if clinically
effect. Alprazolam does not have activity at the indicated, but answer choice A is still the next best
GABA-B or NMDA receptor. step.
6. B. Lorazepam 10. C. Naloxone
Standard urine drug tests detect oxazepam, and all It is rare that a patient presenting to the emergency
benzodiazepines that include oxazepam as one of room with a solitary benzodiazepine overdose can
154 REVIEW QUESTIONS ANSWER KEY
reach a stuporous or comatose state; in these cases, medications, antifungals, and antibiotics, can lead
benzodiazepines are typically co-ingested with to increased serum levels of methadone.
other substances, such as alcohol or opioids. Recall
3. B. 2.5 to 4 hours
that approximately 75% of benzodiazepine-related
fatal overdoses involve co-ingestion with opioids. Methadone differs from other opioids in the drug’s
In this clinical scenario, the immediate next best absorption, metabolism, and relative analgesic
step would be to administer naloxone (choice C). potency. Particular vigilance is necessary during
Flumazenil can be used in benzodiazepine over- treatment initiation and titration. Methadone can
dose, but it can precipitate withdrawal seizures in be detected in the blood 15 to 45 minutes after
patients who are benzodiazepine dependent. Flu- its oral administration and reaches its peak levels
mazenil is frequently used to reverse benzodiaze- between 2 and 4 hours.
pine-induced sedation for anesthesia or in cases of 4. D. Dextromethorphan
pediatric ingestion in which benzodiazepine
Many routinely prescribed medications are asso-
dependence is less of an issue. Flurazepam is a
ciated with false-positive urine toxicology results.
lipophilic benzodiazepine with a long elimination
If in doubt, such tests should be confirmed using
half-life that has no role for treatment in this sce-
gas chromatography-mass spectrometry. The “opi-
nario (choice B). Activated charcoal is generally
ates” immunoassay used in standard urine drug
not recommended in cases of benzodiazepine
screens uses antibodies specific for morphine
overdose because of the risk of aspiration (choice
metabolites and detects the presence of natural
D). Hemodialysis is not effective as a rapid elimi-
and some semisynthetic opioids, such as heroin,
nation technique (choice E).
morphine, and codeine. Opiate false-positive
REFERENCES results have been reported for dextromethorphan,
Bellantuono, C., Tofani, S., Di Sciascio, G., & Santone, G. (2013).
quinolones, diphenhydramine, and rifampin.
Benzodiazepine exposure in pregnancy and risk of major malfor- Dextromethorphan has a chemical structure that
mations: A critical overview. General Hospital Psychiatry, 35(1), is similar to morphine and can lead to a urine
3–8. https://doi.org/10.1016/j.genhosppsych.2012.09.003. drug screen that is false-positive for opiates or
Vinkers, C. H., & Olivier, B. (2012). Mechanisms underlying toler-
phencyclidine.
ance after long-term benzodiazepine use: A future for subtype-
selective GABA(A) receptor modulators? Advances in Pharmaco- 5. B. Meperidine
logical Sciences, 2012, Article 416864.
Meperidine is known to induce seizures because
CHAPTER 6 of the accumulation of meperidine’s metabolite,
normeperidine, norpethidine. The use of meperi-
1. C. 60 to 80 mg dine in the elderly and patients with underlying
Illicit opioid use was directly related to methadone hepatic or renal disease should be avoided. The use
dosage: in patients on doses above 70 mg per day, of meperidine, in combination with serotonergic
no heroin use was detected, whereas patients on medications or herbs, has also been linked with the
doses below 50 mg per day were at higher risk of development of serotonin syndrome.
using heroin than those receiving higher doses. A
6. C. The requirement for 7 days of abstinence from
meta-analysis of 21 studies concluded that metha-
opioids before naltrexone initiation
done dosages ranging from 60 to 100 mg per day
were more effective than lower dosages in retain- Extended-release naltrexone is an FDA-approved
ing patients and in reducing the use of heroin and treatment for OUD. Initiation requires detoxifi-
cocaine use during treatment (Faggiano et al., 2003). cation from opioids first because it precipitates
withdrawal symptoms in individuals dependent
2. C. Fluconazole on opioids. It is recommended for patients to
Methadone is metabolized by CYP3A4 and co-use abstain from opioids for 7 to 10 days before ini-
of potent CYP3A inhibitors, such as certain HIV tiation. This can be a significant barrier for many
REVIEW QUESTIONS ANSWER KEY 155
patients, particularly those being induced in the the person is smoking less than 10 cigarettes per
outpatient setting, but naltrexone is still a practi- day, it is recommended to start with the 14 mg/
cal treatment approach for many individuals with day patch for 6 weeks, followed by 7 mg/day for
OUD. 2 weeks.
7. D. OUD, moderate 4. A. Not wearing the patch during nighttime
In the Diagnostic and Statistical Manual of Mental Leaving the patch on overnight can cause side
Disorders, Fifth Edition, a patient must meet at effects such as insomnia and vivid dreams. If
least two diagnostic criteria to qualify for an OUD. patients experience this, they can remove the patch
Severity is characterized as “mild” if two or three before bedtime and replace it in the morning.
criteria are met, “moderate” if four or five criteria Smoking cessation rates are similar whether the
are met, and “severe” if six or more criteria are met. patch is left on for 24 hours or taken off at night. It
The patient above meets at least four criteria, which is not recommended to wear a patch for more than
qualifies him for a moderate OUD. He demon- 24 hours as it can cause an adverse reaction to the
strates tolerance to opioids, is using more and for skin. It is also not recommended for people to wear
longer than intended, has increased time spent more than one patch at a time.
obtaining opioids, and has failed to fulfill work
5. B. Nasal spray
obligations.
Nicotine nasal spray tends to be used for a longer
REFERENCE time and has been linked with a higher potential
Faggiano, F., Vigna-Taglianti, F., Versino, E., & Lemma, P. (2003). abuse than other forms of nicotine replacement.
Methadone maintenance at different dosages for opioid depen-
dence. Cochrane Database Syst Rev(3), Article CD002208. 6. C. Nortriptyline
A Cochrane systematic review found multiple con-
trolled trials of nortriptyline as smoking cessation.
CHAPTER 7 The data shows that nortriptyline can double the
1. B. Varenicline odds of quitting smoking (Hughes et al., 2005).
Among the available pharmacotherapies, which
REFERENCES
include NRT, bupropion, and varenicline, vareni-
Anthenelli, R. M., Benowitz, N. L., West, R., St Aubin, L., McRae,
cline has the highest efficacy in achieving absti-
T., Lawrence, D., Ascher, J., Russ, C., Krishen, A., & Evins, A. E.
nence. These findings have been demonstrated (2016). Neuropsychiatric safety and efficacy of varenicline, bupro-
in several large studies and were one of the main pion, and nicotine patch in smokers with and without psychiat-
findings of the Evaluating Adverse Events in a ric disorders (EAGLES): A double-blind, randomised, placebo-
Global Smoking Cessation Study (EAGLES) trial controlled clinical trial. Lancet (London, England), 387(10037),
2507–2520.
(Anthenelli et al., 2016). Hughes, J. R., Stead, L. F., & Lancaster, T. (2005). Antidepressants for
smoking cessation. Cochrane Syst Rev.
2. D. Decreased heart rate
Nicotine withdrawal symptoms usually reach their CHAPTER 8
peak 2 to 3 days after discontinuation. Smoking
1. B. Short-term memory impairment
cessation may lead to decreased heart rate, trem-
ors, irritability, insomnia, and increased appetite. Of the answer choices provided, answer choices B
and D are well-known effects of cannabis intoxi-
3. C. 14 mg every 24 hours cation. Cannabis intoxication does not cause
Dosing is determined by the number of cigarettes long-term memory impairments, respiratory
a person smokes daily when the patch is started. depression, or dementia (answer choices A, C, and
If the person smokes more than 10 cigarettes per E). Answer choice B is the correct answer; short-
day, it is recommended to begin with a 21 mg/ term memory impairments can lead to difficulties
day patch for the first 6 weeks, then 14 mg/day with learning and academic performance which is
for 2 weeks, and then 7 mg/day for 2 weeks. If most relevant to his presenting problem.
156 REVIEW QUESTIONS ANSWER KEY
2. B. Higher affinity for the CB1 receptor voucher CM + CBT; this combination is more
Synthetic cannabinoids are full agonists at the CB1 effective than either intervention alone.
receptor and have a much higher receptor affin- 6. D. The majority of individuals who try cannabis
ity (approximately 100 ×) for CB1 compared to once will not develop a cannabis use disorder
plant-based THC. Unlike plant-based cannabis,
Although there is a positive association between
synthetic cannabinoids do not contain CBD which
cannabis use and schizophrenia, the direction
would modulate the psychotomimetic effects of
of causality has not been established, ruling out
CB1 agonism.
answer choices A, B, and C. Among individuals
3. A. The perceived risk of marijuana use has declined who have tried cannabis once, approximately 9%
among this age group will eventually develop a cannabis use disorder,
There has been a sea change in public policy and which rules out answer choice E.
opinion regarding the safety and efficacy of canna- 7. C. Butane hash oil
bis; this has accompanied the widespread legaliza-
The boards may ask about the relative THC potency
tion and decriminalization of cannabis use in many
of different products derived from the cannabis
U.S. states. There is an inverse relationship between
plant. In general: cannabis leaves and stems (0.5%–
perceived risk and substance use; as the perceived
5%); sinsemilla, or the cannabis flowering tops of
risk of cannabis has been declining among high
unpollinated female plants (7%–14%); hashish oil
schoolers, cannabis use has increased. Over the
(15%–50%); “dab” or butane hash oil (up to 90%).
past several decades, the average proportion of
Hashish is dried cannabis resin with compressed
THC has increased while CBD has decreased, rul-
flowers and variable THC potency (2–8%).
ing out answer choices C and D. Despite being
legalized in a number of states, marijuana remains 8. B. Dronabinol
a Schedule I drug, ruling out answer choice E. Dronabinol is synthetically produced delta-9-THC
4. A. Tachypnea and will cause a positive UDS for THC because it is
structurally identical. Sativex is 1:1 delta-9-THC:CBD
Because of the co-localization of CB1 receptors
and will also cause a positive toxicology test. Of
with beta-adrenergic receptors, cannabis intoxica-
the two, this patient is most likely being prescribed
tion can lead to adrenergic-like effects including
dronabinol, which is commonly used for cancer-
tachycardia, tachypnea (answer choice A), and
induced nausea; Sativex is approved for the treatment
blood pressure variability, ruling out answer choice
of spasticity in multiple sclerosis. Nabilone is a syn-
B. Cannabis typically causes dry mouth, not hyper-
thetic analog of THC with distinct metabolites and
salivation (answer choice D). Piloerection com-
will not cause a positive test on a standard urine drug
monly occurs with opioid withdrawal, not cannabis
screen. CBD is not detected on standard urine drug
intoxication (answer choice C). Cannabis intoxica-
screens, ruling out answer choices D and E.
tion typically causes time dilatation or a sense of
time slowing down, ruling out answer choice E. 9. C. Increased risk of preterm birth
5. B. CM Cannabis use during pregnancy is associated with
an increased risk of preterm birth and infants who
This is an example of CM, which is based on the
are small for gestational age or require transfer to
principle of operant conditioning. In contingency
the NICU. It is not associated with increased risk
management, a patient with a substance use disor-
of preeclampsia, neonatal abstinence syndrome, or
der would perform a toxicology test, and if negative,
stillbirth (Corsi et al., 2019).
the patient would get to draw from a bucket of tick-
ets with prizes of variable monetary value. The most 10. C. CB1 antagonism; suicidality
effective evidence-based psychosocial interven- Rimonabant is a CB1 receptor antagonist and a
tion for cannabis use disorder is abstinence-based weight-loss drug whose use was limited due to the
REVIEW QUESTIONS ANSWER KEY 157
adverse effect of suicidality. Rimonabant is no lon- psychotherapy or medications, and from the avail-
ger available in the United States. able options is the only treatment that is based on
the principles of operant conditioning
11. C. Cannabis use has no effect on the risk of driving
accidents. 4. A. It induces intracellular dopamine-containing
Cannabis intoxication increases the risk of car vesicles to release dopamine into the synaptic cleft
accidents by a factor of 2; for comparison, alcohol and blocks the reuptake of dopamine.
with a BAL greater than 0.08% increases risk of car Cocaine acts primarily by blocking the reuptake
accidents by a factor of 5 (Sewell et al., 2009). Can- of released dopamine in the synaptic clefts of the
nabis intoxication leads to delayed reaction time, mesolimbic dopamine neurons. Caffeine competi-
poorer hand–eye coordination, and impairs auto- tively binds to adenosine receptors affecting the
matic motor behaviors that are important in driv- release of catecholamines and other neuropep-
ing. States that have legalized recreational tides. Methamphetamine blocks the dopamine
marijuana have seen a significant increase in can- transporter from pumping dopamine back into the
nabis-related car accidents. transmitting neuron and causes the intracellular
release of neurotransmitters to the synaptic cleft.
REFERENCES From the options, methamphetamine is the only
Corsi, D. J., Walsh, L., Weiss, D., Hsu, H., El-Chaar, D., Hawken, one that is commonly manufactured in illegal hid-
S., Fell, D. B., & Walker, M. (2019). Association between self- den laboratories by mixing various chemicals and
reported prenatal cannabis use and maternal, perinatal, and
neonatal outcomes. JAMA, 322(2), 145–152.
over-the-counter medications.
Sewell, R. A., Poling, J., & Sofuoglu, M. (2009). The effect of cannabis
5. D. Mirtazapine
compared with alcohol on driving. The American Journal on Ad-
dictions, 18(3), 185–193. Mirtazapine is a medication currently used to
treat depression, which antagonizes noradrener-
gic alpha-2 receptors and the serotonin 5-HT2A/C
CHAPTER 9 and 5-HT3 receptors. It has demonstrated efficacy
1. D. Bupropion in reducing the rewarding effect of cocaine.
Combined with CM, bupropion has been proven 6. C. Amphetamine
to reduce cocaine use more effectively than either Stimulant intoxication presents with hyperten-
treatment alone or placebo (Poling et al., 2006). sion, chest pain, anxiety, and diaphoresis. Tobacco
2. E. Rivastigmine intoxication would involve some gastrointestinal
Medications enhancing cholinergic transmission, symptoms. Alcohol intoxication would present
especially cholinesterase inhibitors, have shown with disinhibition and slurred speech. Opioid use
promise for the treatment of stimulant use disorder. would present with decreased respiration, lethargy,
Rivastigmine has reduced methamphetamine-asso- and constricted pupils.
ciated increases in diastolic blood pressure and self- 7. D. CAMK4
reported feelings of anxiety as well as the desire for the The CAMK4 and the presence of CHRNB3-A6 are
use of methamphetamine (De La Garza et al., 2012). linked with a higher susceptibility for cocaine use
3. A. CM disorder. VNTRs in the DAT1 gene are also associ-
Several behavioral treatments have been used for ated with cocaine and methamphetamine-induced
cocaine use disorder treatment, including CBT psychosis.
and CM. The goal of CBT is to teach strategies 8. D. Bruxism
and enhance coping abilities to prevent drug use Bruxism is frequently seen in MDMA intoxication.
behavior. The efficacy of CBT for cocaine use dis- The grinding of teeth usually goes away within 24
order has been demonstrated in multiple stud- to 48 hours after ingestion of the drug.
ies. CM can be combined with other forms of
158 REVIEW QUESTIONS ANSWER KEY
■ Maintenance: The goals for this stage include 3. D. Using point-of-care testing at home three times
sustaining changes, consolidating gains, learn- a week and rewarding her with random gift certifi-
ing alternative coping strategies, and recogniz- cates occasionally when she tests negative.
ing triggers. CM is based on the principles of introduc-
■ Relapse and recycling: This stage is not inevi- ing alternative reinforcers that are valued more
table. During this stage, patients’ main clinical than using drugs or alcohol (such as monetary
risk is to feel stuck in their old ways. As such, the rewards). CM as therapy involves positive rein-
primary goal of treatment is avoiding becoming forcement by rewarding patients with incentives
stuck and redefining the relapse as an opportu- for achieving their treatment goals such as absti-
nity for further self-improvement. nence. CM is most effective when the rewards
■ Termination: Patients in this stage have reached are given immediately following a negative test
their ultimate goals and can exit the cycle of (rather than days later if the drug screen is per-
change without fear of relapse. formed at a laboratory) and delivered with a
2. C. It sounds like you are interested in quitting but variable frequency (rather than following every
are worried about how quitting may affect your negative test).
performance.
Reflections are likely the most important aspect of 4. B. Commitment to participating in 12-step–based
MI. Different types of reflections include simple mutual-support groups and to abstinence.
reflections (for example: “So you are saying that Multiple studies investigating the effectiveness of
alcohol is causing a drift in your relationship.”), 12-step–based mutual-support groups have con-
double reflections (e.g., “So on the one hand, alco- sistently found that group participants who are
hol helps you cope with stress caused by work, committed to the group process and to the goal of
and on the other hand, it is causing a drift in your achieving abstinence tend to fare better and have
relationship.”), or amplified reflections (for exam- better long-term abstinence outcomes. Twelve-
ple, after a patient tells you that they enjoy drink- step-based mutual-support groups meetings
ing alcohol, you overshoot and respond: “I see, involve group members sharing their experiences
drinking alcohol has been a very positive experi- and are encouraged to “work through” the 12 steps.
ence in your life and you have no interest in cut- Twelve-step-based mutual-support group princi-
ting down.”). In this clinical vignette, the patient is ples include taking responsibility, finding humility,
expressing a desire to address her cocaine use but is and using honesty. The process uses tools based on
identifying potential barriers. Reflecting her state- psychological principles, including stimulus con-
ments will allow her to expand on her motivating trol and cue reactivity, reconditioning, and positive
factors and will lead to her using more change talk. reinforcement.
To protect the rights of the author(s) and publisher we inform you that this PDF is an uncorrected proof for internal business use only by the author(s), editor(s),
reviewer(s), Elsevier and typesetter Aptara. It is not allowed to publish this proof online or in print. This proof copy is the copyright property of the publisher and
is confidential until formal publication.
INDEX
Page numbers followed by ‘f ’ indicate figures, ‘t’ indicate tables, ‘b’ indicate boxes.
191
00015-Colon-Rivera-780323754866
These proofs may contain color figures. Those figures may print black and white in the final printed book if a color print product has not been planned. The color
figures will appear in color in all electronic versions of this book.
To protect the rights of the author(s) and publisher we inform you that this PDF is an uncorrected proof for internal business use only by the author(s), editor(s),
reviewer(s), Elsevier and typesetter Aptara. It is not allowed to publish this proof online or in print. This proof copy is the copyright property of the publisher and
is confidential until formal publication.
192 INDEX
Benzodiazepine, 5, 13, 35, 47, 135–136 Cannabis, 21, 123 CNR1 gene, 78
adverse effects of, 50 cardiovascular effects, 78 Cocaethylene assays, 104
cognitive impairment, 50 characteristics, 73 Cocaine, 5
half-lives and equipotent dosages, 48t chemical structure, 73 alcohol and, 104
immunoassays use antibodies, 104 comorbid medical conditions, 78 and angina, 13
increased mortality, 50 and driving, 79 chronic, 88
medical indications, 49 epidemiology of, 77 intrauterine exposure, 143
metabolic pathway of, 104f intoxication, 75, 76t and ischemia, 128
metabolism, 49 long-term effects of, 77 Cocaine use disorder, 122
overdose deaths, 127 neurobiology, 73 pharmacological treatment options, 87
paradoxical disinhibition, 50 and obesity, 129 Codeine (3-Methylmorphine), 61
physiologic dependence and withdrawal, and opioid use, 79 Coercion, 3
50 perceived risk of, 77 Cognitive behavioral therapy (CBT), 87,
pregnancy effects, 50 pharmacotherapy, 80 114b
psychomotor impairment, 50 potency, 73 psychological theory, 121
rebound insomnia, 50 pregnancy and, 79 Combined Pharmacotherapies and
respiratory depression, 50 psychiatric comorbidities, 78 Behavioral Interventions
sedation, 50 psychosocial interventions, 80 (COMBINE) study, 43, 120
suicide risk, 50 pulmonary effects, 79 Community reinforcement approach, 115
toxicology testing, 49 risk factors, 77 Complete blood count (CBC), 105–106
Benzoylecgonine (BE), 104 route of administration, 76t Comprehensive Addiction and Recovery
Beta-endorphin, 99 with schizophrenia, 78 Act (CARA) of 2016, 58
Binge/intoxication, 20, 36 treatments for, 80 Compulsivity, 19
animal models, 22 use disorder, 77 Conditioned place aversion, 23–24
neurobiological processes, 20 withdrawal, 77 Conditioned (learned) tolerance, 2
Bipolar disorder, 12 Cannabis family, 73 Consequentialism theory, 106
Black box, 70 Cannabis hyperemesis syndrome (CHS), Constipation, opioid use disorder, 60
warning, 50 76 Contemplation, 114
Blackouts, 33 Cannabis indica, 130 Contingency management (CM), 87, 114
Blood alcohol concentration (BAC), 32 Cannabis sativa, 130 principles, 125
Board of Education v. Earls, 107 Cannabis use disorder, 4 Controlled Substance Act, 80, 107
Body dysmorphic disorder, 94b Carbohydrate-deficient transferrin Co-occurring psychiatric illness, 53
Boggs Act of 1952, 80 (%CDT), 105–106 primary disorders, 11
Bradykinin, 99 Carcinogen, 31 Cue-induced reinstatement, 24–25
Breath alcohol testing, 32 Cardiomyopathy and alcohol use, 13 Current Opioid Misuse Measure (COMM),
Breathalyzer, 32 Cardiovascular effects 60
Buprenorphine, 28, 61, 64 cannabis, 78 Cyclic adenosine monophosphate
Bupropion, 70 of substance use, 12 phosphorylation (cAMP) signaling
Burgeoning use disorder, 53 Carfentanil, 62b pathways, 62–63
CB1 receptor, 129 Cytisine, 70
C 42 CFR (Code of Federal Regulation), 107 Cytokines, 99
Caffeine (1,3,7-trimethylxanthine), 4, 95 Chemotherapy-related nausea, 119
Calcium/calmodulin kinase (CAMK4), Child and adolescent levels of care D
85 utilization services (CALOCUS), 3 DARN-C, 113
cAMP response element-binding (CREB), Childhood antisocial behaviors, 38 DEARS, 113
26 Cirrhosis and alcohol use, 13 Defensive burying test, 24
Cancer, 79 Classical conditioning, 19 Delayed discounting, 19
Cannabidiol (CBD), 73 Clinical Institute Withdrawal Assessment Delta-9-tetrahydrocannabinol (THC),
Cannabinoid, 73, 99 for Alcohol (CIWA) protocol, 73, 122
chemical structure of, 74f 133–134 mechanisms of action, 74
endogenous, 73–74 Clinical Institute Withdrawal Assessment potency, 73
mechanism of action, 74 for Alcohol-Revised (CIWA-Ar), toxicology testing, 75
pharmacology, 74 34–35 Deontological theory, 106
plant-based, 73–74 Clinical Opiate Withdrawal Scale (COWS), Depression, 11
receptors, 75 63 Deviance proneness, 38
synthetic, 74 Clonidine, 70 Dextromethorphan (DXM), 91–92
00015-Colon-Rivera-780323754866
These proofs may contain color figures. Those figures may print black and white in the final printed book if a color print product has not been planned. The color
figures will appear in color in all electronic versions of this book.
To protect the rights of the author(s) and publisher we inform you that this PDF is an uncorrected proof for internal business use only by the author(s), editor(s),
reviewer(s), Elsevier and typesetter Aptara. It is not allowed to publish this proof online or in print. This proof copy is the copyright property of the publisher and
is confidential until formal publication.
INDEX 193
00015-Colon-Rivera-780323754866
These proofs may contain color figures. Those figures may print black and white in the final printed book if a color print product has not been planned. The color
figures will appear in color in all electronic versions of this book.
To protect the rights of the author(s) and publisher we inform you that this PDF is an uncorrected proof for internal business use only by the author(s), editor(s),
reviewer(s), Elsevier and typesetter Aptara. It is not allowed to publish this proof online or in print. This proof copy is the copyright property of the publisher and
is confidential until formal publication.
194 INDEX
00015-Colon-Rivera-780323754866
These proofs may contain color figures. Those figures may print black and white in the final printed book if a color print product has not been planned. The color
figures will appear in color in all electronic versions of this book.
To protect the rights of the author(s) and publisher we inform you that this PDF is an uncorrected proof for internal business use only by the author(s), editor(s),
reviewer(s), Elsevier and typesetter Aptara. It is not allowed to publish this proof online or in print. This proof copy is the copyright property of the publisher and
is confidential until formal publication.
INDEX 195
Opioid use disorder (Continued) Powell v. Texas, 107 Ryan Haight Online Pharmacy Consumer
respiratory depression, 59–60 Precontemplation, 113 Protection Act of 2008, 58
screening tools for, 60 Prefrontal cortex (PFC), 21
treatment of, 57 Pregnancy S
federal guidelines for, 59 and cannabis, 79 Saliva drug testing, 103
OPRM1 (μ-opioid receptor) gene, 41 opioid use disorder and, 64 SAMHSA-5 assays, 103–104
Osteonecrosis, 15 and stimulant use disorder, 88 Sativex, 81
Overdose, 121 teratogenic effects in, 39 Schema-focused therapy, 115
Oxycodone, 61 The Pregnant and the Reduction of Schizoaffective disorder, 128, 133–134
Opiates: Medication Intervention Schizophrenia, 12
P Safety and Efficacy (PROMISE) cannabis with, 78
Pain study, 65b Scopolamine, 93
clinical considerations in, 99 Preoccupation/anticipation, 20 Screener and Opioid Assessment for
gabapentin, 101 animal models, 24 Patients with Pain-Revised
mediators of, 99 stage of addiction, 24 (SOAPP-R), 60
meperidine, 101 Prescription drug monitoring programs Screening and Brief Intervention (SBI),
neurobiology of, 99 (PDMPs), 59 142–143
neuropathic, 99 Project MATCH (Matching Alcoholism Screening, brief intervention, and referral
nociceptive, 99 Treatment to Client Heterogeneity), to treatment (SBIRT), 116, 128
nonsteroidal antiinflammatory drugs, 42, 120 Secondary psychiatric disorders, 11
100 Prostaglandins, 99 Secondhand smoke, 69
opioids, 101 Pruritus, opioid use disorder, 60 Sedative-ataxic response, 38
paracetamol (acetaminophen), 101 Pseudoaddiction, 53 Sedatives
pathways, 99 Pseudo-withdrawal, 2 adverse effects, 50
pharmacotherapies for, 100 Psilocybin, 92 characteristics, 47
scales, 100 Psychiatric illness, co-occurring, 53 chemical structure, 47
tricyclic antidepressants (TCAs), 101 Psychiatric manifestations, substance use, differential diagnosis, 51
types of, 99 11 duration of action, 48
Panic attacks, 12 Psychosocial approaches elimination half-life, 48
Paracetamol (acetaminophen), 101 centralized model of care, 109 epidemiology of, 52
Partial fetal alcohol syndrome (pFAS), distributive model of care, 109 lipophilicity, 48
39–40 family-based programs, 109 management of, 51
Patient Medication Questionnaire (PMQ), indicated programs, 109 medical indications, 49
60 long-term care, 109 neurobiology, 47
Patient placement criteria (PPC-2R) selective programs, 109 onset, 48
model, 2–3 universal programs, 109 pharmacodynamics, 47
Pentoxifylline, 14 Pulmonary effects pharmacokinetics, 48
People v. Saille, 107, 136 cannabis, 79 potency, 48
Period circadian regulator (PER1) gene, of substance use, 15 presentation of, 50
85 rehabilitation for, 124
Persistent psychosis, 12 R structure of prototypical, 48f
Personality disorders, 133 Racial and ethnic minorities, 4 use disorder, 12, 52
Pharmacokinetic considerations, 5 Rapid eye movement (REM) sleep, 12 withdrawal, 51
Pharmacotherapeutic approaches, 27 Rebound symptoms, 2 Self-Management and Recovery Training
Phencyclidine (PCP), 91, 142 Renal comorbidities, substance use, 14 (SMART) Recovery, 111
Phosphatidylethanol (PETH), 105–106 Renal failure, acute, 14–15 Self-medication hypothesis, 38
Phosphodiesterase-5-inhibitors, 95b Residential rehabilitation program, 109 Semisynthetic opioids, 15
Physiological dependence, 2 Respiratory depression, 59–60 Sensitivity, 3
Placement matching, 3 Reward deficiency syndrome theory, 23 Serotonin, 32
Polycyclic aromatic hydrocarbons, 67 Reward pathway, 20 Sertraline, 42
Poppers, 95 Rhabdomyolysis, 14–15 Sleep impairments, alcohol use and, 12
Poppy seeds, 104 Richmond Agitation-Sedation Scale Smoking cessation, 69, 132
and false-positive tests, 137–138 (RASS), 35 Somatic symptom disorder, 100
Positive predictive value (PPV), 3–4 Rimonabant, 28, 75 Specificity, 3
Positive reinforcement, 19 Robinson v. California, 107 Standard drink, 36
Posstraumatic stress disorder (PTSD), 11 Roofies, 93 Steatosis, alcohol use and, 13
00015-Colon-Rivera-780323754866
These proofs may contain color figures. Those figures may print black and white in the final printed book if a color print product has not been planned. The color
figures will appear in color in all electronic versions of this book.
To protect the rights of the author(s) and publisher we inform you that this PDF is an uncorrected proof for internal business use only by the author(s), editor(s),
reviewer(s), Elsevier and typesetter Aptara. It is not allowed to publish this proof online or in print. This proof copy is the copyright property of the publisher and
is confidential until formal publication.
196 INDEX
00015-Colon-Rivera-780323754866
These proofs may contain color figures. Those figures may print black and white in the final printed book if a color print product has not been planned. The color
figures will appear in color in all electronic versions of this book.
To protect the rights of the author(s) and publisher we inform you that this PDF is an uncorrected proof for internal business use only by the author(s), editor(s),
reviewer(s), Elsevier and typesetter Aptara. It is not allowed to publish this proof online or in print. This proof copy is the copyright property of the publisher and
is confidential until formal publication.
INDEX 197
00015-Colon-Rivera-780323754866
These proofs may contain color figures. Those figures may print black and white in the final printed book if a color print product has not been planned. The color
figures will appear in color in all electronic versions of this book.